Download as pdf or txt
Download as pdf or txt
You are on page 1of 26

FOCUS 17 that depend on the collisions between reactants.

The second
gives insight into the formation of polymers and shows how
the kinetics of their formation affects their properties. The
ticularly interesting as it is the basis for a method of estimating
the distance between certain groups in large molecules.
17G.1 Photochemical processes; 17G.2 The primary quantum yield;
third examines the general mechanism of action of ‘enzymes’, 17G.3 Mechanism of decay of excited singlet states; 17G.4 Quenching;
17G.5 Resonance energy transfer

Chemical kinetics
which are biological catalysts.
17F.1 Unimolecular reactions; 17A.2 Polymerization kinetics;
17F.3 Enzyme-catalysed reactions

Web resource What is an application


of this material?
17G Photochemistry
Plants, algae, and some species of bacteria have evolved appa-
‘Photochemistry’ is the study of reactions that are initiated by ratus that performs ‘photosynthesis’, the capture of visible and
This Focus introduces the principles of ‘chemical kinetics’, results of the analysis are relations, which can be explored ex- light. This Topic explores the fate of the electronically excited near-infrared radiation for the purpose of synthesizing com-
the study of reaction rates. The rate of a chemical reaction perimentally, between the equilibrium constant of the overall molecules formed by the absorption of photons. One possible plex molecules in the cell. Impact 26 introduces the reaction
might depend on variables that can be controlled, such as the process and the rate constants of the forward and reverse reac- fate is energy transfer to another molecule; this process is par- steps involved.
pressure, the temperature, and the presence of a catalyst, and tions in the proposed mechanism.
it is possible to optimize the rate by the appropriate choice of 17C.1 First-order reactions approaching equilibrium;
conditions. 17C.2 Relaxation methods

17A The rates of chemical reactions 17D The Arrhenius equation


This Topic discusses the definition of reaction rate and out- The rate constants of most reactions increase with increasing
lines the techniques for its measurement. The results of such temperature. This Topic introduces the ‘Arrhenius equation’,
measurements show that reaction rates depend on the concen- which captures this temperature dependence by using only
tration of reactants (and sometimes products) and ‘rate con- two parameters that can be determined experimentally.
stants’ that are characteristic of the reaction. This dependence 17D.1 The temperature dependence of reaction rates;
can be expressed in terms of differential equations known as 17D.2 The interpretation of the Arrhenius parameters
‘rate laws’.
17A.1 Monitoring the progress of a reaction; 17A.2 The rates of reactions

17E Reaction mechanisms


17B Integrated rate laws The study of reaction rates also leads to an understanding of
the ‘mechanisms’ of reactions, their analysis into a sequence of
‘Integrated rate laws’ are the solutions of the differential equa- elementary steps. This Topic shows how to construct rate laws
tions that describe rate laws. They are used to predict the con- from a proposed mechanism. The elementary steps themselves
centrations of species at any time after the start of the reaction have simple rate laws which can be combined into an overall
and to provide procedures for measuring rate constants. This rate law by invoking the concept of the ‘rate-determining step’
Topic explores some simple yet useful integrated rate laws that of a reaction, by making the ‘steady-state approximation’, or
appear throughout the Focus. by supposing that a ‘pre-equilibrium’ exists.
17B.1 Zeroth-order reactions; 17B.2 First-order reactions; 17E.1 Elementary reactions; 17E.2 Consecutive elementary reactions;
17B.3 Second-order reactions 17E.3 The steady-state approximation; 17E.4 The rate-determining
step; 17E.5 Pre-equilibria; 17E.6 Kinetic and thermodynamic control
of reactions

17C Reactions approaching equilibrium


17F Examples of reaction mechanisms
In general, rate laws must take into account both the forward
and reverse reactions and describe the approach to equi- This Topic develops three examples of reaction mechanisms.
librium, when the forward and reverse rates are equal. The The first describes a special class of reactions in the gas phase

724 17 Chemical kinetics

TOPIC 17A The rates of chemical reactions reaction vessel held at constant temperature. If the initial
pressure is p0, and is due solely to N2O5, and the total pressure
at any later time is p, derive expressions for the partial pres- Driving
Mixing
chamber Movable
spectrometer
sures of all three species in terms of p0 and p. pistons
Collect your thoughts You can assume perfect-gas behaviour,
in which case the partial pressure of a gas is proportional to its Fixed
spectrometer
amount. Then imagine that a certain amount of N2O5 decom-
ment. Gas-phase reactions, for instance, are often carried out poses such that its partial pressure falls from p0 to p0 − Δp. Figure 17A.1 The arrangement used in the flow technique for
➤ Why do you need to know this material? in a vessel held in contact with a substantial block of metal. Because 1 mol N2O5 decomposes to give 2 mol NO2, the partial studying reaction rates. The reactants are injected into the mixing
Liquid-phase reactions must be carried out in an efficient ther- pressure of NO2 increases from 0 to 2Δp. Likewise, the partial chamber at a steady rate. The location of the spectrometer
Studies of the rates of consumption of reactants and for-
mostat. Special efforts have to be made to study reactions at pressure of O2 increases from 0 to 12 Δp. The total pressure p corresponds to different times after initiation of the reaction.
mation of products make it possible to predict how quickly
low temperatures, as in the study of the kinds of reactions that is the sum of the partial pressures of the three components.
a reaction mixture approaches equilibrium. They also lead
take place in interstellar clouds. Thus, supersonic expansion
to detailed descriptions of the molecular events that trans- The solution Draw up the following table:
of the reacting gas can be used to attain temperatures as low Mixing
form reactants into products.
as 10 K. For work in the liquid phase and the solid phase, very p(N2O5) p(NO2) p(O2) p chamber

➤ What is the key idea? low temperatures are often reached by flowing cold liquid or Initially p0 0 0 p0 Driving Stopping
cold gas around the reaction vessel. Alternatively, the entire 1 p0 + 32 Δp pistons piston
Reaction rates are expressed as rate laws, which are empir- Later p0 − Δp 2Δp 2 Δp
reaction vessel is immersed in a thermally insulated container
ical summaries of the rates in terms of the concentrations Equivalent to 5p − 32 p 4 (p − p0) 1 (p − p0)
filled with a cryogenic liquid, such as liquid helium (for work 3 0 3 3
Fixed
of reactants and, in some cases, products.
at around 4 K) or liquid nitrogen (for work at around 77 K). spectrometer
Non-isothermal conditions are sometimes employed. For in- The bottom line follows from p = p0 + 32 Δp, rearranged into
➤ What do you need to know already? Δp = 23 (p − p0), and then substituted into the line above. Figure 17A.2 In the stopped-flow technique the reagents are
stance, the shelf life of an expensive pharmaceutical may be driven quickly into the mixing chamber by the driving pistons
This introductory Topic is the foundation of a sequence:
explored by slowly raising the temperature of a single sample. Comment. A check of the calculation is to note that when all and then the time dependence of the concentrations
all you need to be aware of initially is the significance of
Spectroscopy is widely applicable to the study of reac- the N2O5 has been consumed its partial pressure is zero, which is monitored.
stoichiometric numbers (Topic 2C). For more background
tion kinetics, and is especially useful when one substance in implies that 53 p0 − 23 pfinal = 0 and hence pfinal = 52 p0. This result
on the spectroscopic determination of concentration,
the reaction mixture has a strong characteristic absorption is expected because 1 mol N2O5 is replaced by 2 mol NO2 and
refer to Topic 11A.
2 mol O2: the total amount of molecules therefore goes from
1
in a conveniently accessible region of the electromagnetic
spectrum. For example, the progress of the reaction H2(g) + 1 mol to 2 12 mol, resulting in the pressure increasing by a conventional flow techniques is that a large volume of reactant
Br2(g) → 2 HBr(g) can be followed by measuring the absorp- factor of 2 12 = 52 . solution is necessary. This requirement makes the study of fast
Chemical kinetics is the study of reaction rates. Experiments tion of visible radiation by bromine. A reaction that changes Self-test 17A.1 Repeat the calculation of the partial pressures reactions particularly difficult because to spread the reaction
show that reaction rates depend on the concentration of reac- the number or type of ions present in a solution may be for the species in the reaction 2 NOBr(g) → 2 NO(g) + Br2(g), over a length of tube the flow must be rapid. This disadvan-
tants (and in some cases products) in characteristic ways that followed by monitoring the electrical conductivity of the assuming that the initial pressure is p0 and is due to NOBr tage is avoided by the stopped-flow technique, in which the
can be expressed in terms of differential equations known as solution. The replacement of neutral molecules by ionic prod- alone. reagents are mixed very quickly in a small chamber fitted with
‘rate laws’. ucts can result in dramatic changes in the conductivity, as Answer: pNOBr = 23 p0 − 12 p, pNO = p − p0 , pBr2 = 12 ( p − p0 ) a movable piston instead of an outlet tube (Fig. 17A.2). The
in the reaction (CH3)3CCl(aq) + H2O(l) → (CH3)3COH(aq) + flow pushes the piston back and ceases when it reaches a stop;
H+(aq) + Cl−(aq). If hydrogen ions are produced or consumed, the reaction continues in the mixed solutions. Observations,
the reaction may be followed by monitoring the pH of the commonly using spectroscopic techniques such as ultraviolet–
Monitoring the progress of a
17A.1 solution.
(b) Special techniques
visible absorption and fluorescence emission, are made on the
reaction Other methods of determining composition include emis- The method used to monitor concentrations depends on the sample as a function of time. The technique permits the study
sion spectroscopy (Topic 11F), mass spectrometry, gas chro- species involved and the rapidity with which their concentra- of reactions that occur on the millisecond to second timescale.
The first step in the kinetic analysis of reactions is to establish the matography, nuclear magnetic resonance (Topics 12B and tions change. Many reactions reach equilibrium over periods The suitability of the stopped-flow method for the study of
stoichiometry of the reaction and identify any side reactions. 12C), and electron paramagnetic resonance (for reactions in- of minutes or hours, and several techniques may then be used small samples means that it is appropriate for many biochemi-
The basic data of chemical kinetics are then the concentrations volving radicals or paramagnetic d-metal ions; Topic 12D). A to follow the changing concentrations. In a real-time analysis cal reactions; it has been widely used to study the kinetics of
of the reactants and products at different times after a reaction reaction in which at least one component is a gas might result the composition of the system is analysed while the reaction protein folding and enzyme action.
has been initiated. in an overall change in pressure in a system of constant vol- is in progress. Either a small sample is withdrawn or the bulk Very fast reactions can be studied by flash photolysis, in
ume, so its progress may be followed by recording the varia- solution is monitored. which the sample is exposed to a brief flash of light, which
tion of pressure with time. In the flow method the reactants are mixed as they flow initiates the reaction, and then the contents of the reaction
(a) General considerations together into a chamber (Fig. 17A.1). The reaction continues chamber are monitored by electronic absorption or emission,
The rates of most chemical reactions are sensitive to the Example 17A.1 as the thoroughly mixed solutions flow through the outlet infrared absorption, or Raman scattering. In the arrangement
Relating the variation in the total
temperature (as described in Topic 17D), so in conventional tube, and observation of the composition at different positions shown in Fig. 17A.3 a strong and short laser pulse, the pump,
pressure to the partial pressures of the species present
experiments the temperature of the reaction mixture must along the tube is equivalent to the observation of the reaction promotes a molecule A to an excited electronic state A* which
be held constant throughout the course of the reaction. This The decomposition N2O5(g) → 2 NO2(g) + 12 O2(g) is moni- mixture at different times after mixing. The disadvantage of can either emit a photon (as fluorescence or phosphorescence)
requirement puts severe demands on the design of an experi- tored by measuring the total pressure in a constant-volume
17A The rates of chemical reactions 725 726 17 Chemical kinetics

travelled along a fixed length of the outlet tube. Different reac- It follows from the stoichiometry of the reaction A + 2 B → v = 12 d[NO]/dt = 0.080 mmol dm−3 s −1. Because νNOBr = −2 it
Detector Monochromator tion times can be selected by varying the flow rate along the 3 C + D that follows that the rate of reaction can be written in terms of
outlet tube. An advantage of the chemical quench flow method [NOBr] as v = − 12 d[NOBr]/dt; hence d[NOBr]/dt = −2v =
Beamsplitter d[D] 1 d[C] d[A] d[B] −0.16 mmol dm−3 s −1. The rate of consumption of NOBr is
Laser over the stopped-flow method is that rapid spectroscopic =3 =− = − 12
dt dt dt dt therefore 0.16 mmol dm−3 s −1, or 9.6 × 1016 molecules cm−3 s −1.
measurements are not needed in order to measure the concen-
Lens Lens Sample tration of reactants and products. Once the reaction has been so there are several rates connected with the reaction. The un-
cell quenched, the solution may be examined by ‘slow’ techniques, desirability of having different rates to describe the same reac-
Prisms on such as mass spectrometry and chromatography. In the freeze tion is avoided by introducing the extent of reaction, ξ (xi), (b) Rate laws and rate constants
Continuum
generation motorized stage quench method, the reaction is quenched by cooling the mix- which is defined so that for each species J in the reaction, the
ture within milliseconds and the concentrations of reactants, change in amount of J, dnJ, is The rate of reaction is often found to be proportional to the
Figure 17A.3 A configuration used for flash photolysis, in which
intermediates, and products are measured spectroscopically. concentrations of the reactants raised to a power. For example,
the same pulsed laser is used to generate a monochromatic Extent of reaction
dnJ = ν Jdξ [definition] (17A.1) the rate of a reaction might be found to be proportional to the
pump pulse and, after continuum generation, a ‘white’ light
probe pulse. The time delay between the pump and probe pulses
molar concentrations of two reactants A and B, so
may be varied. where νJ is the stoichiometric number of the species (Topic
17A.2 The rates of reactions 2C; remember that νJ is negative for reactants and positive for v = kr[A][B] (17A.4)
products). The unique rate of reaction, v, is then defined as
or react with another species B to form first the intermediate Reaction rates depend on the composition and the tempera- The constant of proportionality kr is called the rate constant
1 dξ Rate of reaction
AB and then the product C: ture of the reaction mixture. The next few sections look at v= (17A.2) for the reaction; it is independent of the concentrations but
V dt [definition]
these observations in more detail. depends on the temperature. An experimentally determined
A + hν → A* (absorption) where V is the volume of the system. For any species J, dξ = equation of this kind is called the rate law of the reaction.
dnJ/νJ, so More formally, a rate law is an equation that expresses the rate
A* → A (emission) (a) The definition of rate of reaction in terms of the concentrations of all the species
1 1 dnJ
A* + B → AB → C (reaction) Consider a reaction of the form A + 2 B → 3 C + D, in which at v= × (17A.3a) present in the overall chemical equation for the reaction at the
ν J V dt
some instant the molar concentration of a participant J is [J] time of interest:
The rates of appearance and disappearance of the various spe- and the volume of the system is constant. The instantaneous For a homogeneous reaction in a constant-volume system
Rate law in terms of concentrations
cies are determined by observing time-dependent changes in rate of consumption of a reactant or formation of a product is the volume V can be taken inside the differential and nJ/V is v = f([A],[B], …) [general form] (17A.5a)
the absorption spectrum of the sample during the course of the slope of the tangent to the graph of concentration against written as the molar concentration [J] to give
the reaction. This monitoring is done by passing a weak pulse time (expressed as a positive quantity). It follows that the For homogeneous gas-phase reactions, it is often more con-
1 d[J]
of white light, the probe, through the sample at different times instantaneous rate of consumption of one of the reactants at v= (17A.3b) venient to express the rate law in terms of partial pressures,
ν J dt
after the laser pulse. Pulsed ‘white’ light can be generated di- a given time is −d[R]/dt, where R is A or B. This rate is a posi- which for perfect gases are related to molar concentrations by
rectly from the laser pulse by the phenomenon of continuum tive quantity (Fig. 17A.4). The rate of formation of one of the For a heterogeneous reaction the (constant) surface area, A, pJ = RT[J]. In this case,
generation, in which focusing a short laser pulse on sapphire products (C or D, denoted P) is d[P]/dt (note the difference in occupied by the species is used in place of V. Then, because the
Rate law in terms of partial pressures
or a vessel containing water or carbon tetrachloride results sign). This rate is also positive. surface concentration is σJ = nJ/A it follows that v = f(pA,pB, …) [general form] (17A.5b)
in an outgoing beam with a wide distribution of frequencies.
1 dσ J
A time delay between the strong laser pulse and the ‘white’ v= (17A.3c) The rate law of a reaction is determined experimentally, and
ν J dt
light pulse can be introduced by allowing one of the beams to cannot in general be inferred from the chemical equation
travel a longer distance before reaching the sample. For exam- (a) Tangent, rate = slope
In each case there is now a single rate for the reaction (for the for the reaction. The reaction of hydrogen and bromine, for
ple, a difference in travel distance of Δd = 3 mm corresponds Product chemical equation as written). With molar concentrations example, has a very simple stoichiometry, H2(g) + Br2(g) →
Molar concentration, [J]

to a time delay Δt = Δd/c ≈10 ps between two beams, where c is in moles per cubic decimetre and time in seconds, reaction 2 HBr(g), but its rate law is complicated:
the speed of light. The relative distances travelled by the two rates of homogeneous reactions are reported in moles per ka[H 2 ][Br2 ]3/2
beams in Fig. 17A.3 are controlled by directing the ‘white’ light (b) Tangent, rate = –slope cubic decimetre per second (mol dm−3 s−1) or related units. For v= (17A.6)
[Br2 ] + kb[HBr]
beam to a motorized stage carrying a pair of mirrors. gas-phase reactions, such as those taking place in the atmos-
In contrast to real-time analysis, quenching methods are phere, concentrations are often expressed in molecules per In certain cases the rate law does reflect the stoichiometry
based on ‘quenching’, or stopping, the reaction after it has cubic centimetre (molecules cm−3) and rates in molecules of the reaction; but that is either a coincidence or reflects a
been allowed to proceed for a certain time. In this way the Reactant per cubic centimetre per second (molecules cm−3 s−1). For het- feature of the underlying reaction mechanism (Topic 17E).
composition is analysed at leisure and reaction intermediates erogeneous reactions, rates are expressed in moles per square
A note on good (or, at least, our) practice A general rate constant
may be trapped. These methods are suitable only for reactions Time, t metre per second (mol m−2 s−1) or related units.
is denoted kr to distinguish it from the Boltzmann constant k. In
slow enough for there to be little reaction during the time it Figure 17A.4 The definition of (instantaneous) rate as the some texts k is used for the former and kB for the latter. When
takes to quench the mixture. In the chemical quench flow slope of the tangent drawn to the curve showing the variation Brief illustration 17A.1 expressing the rate constants in a more complicated rate law, such
method, the reactants are mixed in much the same way as in of concentration of (a) products, (b) reactants with time. For as that in eqn 17A.6, we use k a, k b, and so on.
the flow method but the reaction is quenched by another rea- negative slopes, the sign is changed when reporting the rate, The rate of formation of NO, d[NO]/dt, in the reaction
gent, such as a solution of acid or base, after the mixture has so all reaction rates are positive. 2 NOBr(g) → 2 NO(g) + Br2(g) is reported as 0.16 mmol dm−3 s −1. The units of kr are always such as to convert the product of
Because νNO = +2, the rate of the reaction is reported as concentrations, each raised to the appropriate power, into a

17A The rates of chemical reactions 727 728 17 Chemical kinetics

rate expressed as a change in concentration divided by time. The power to which the concentration of a species (a product Taking (common) logarithms gives
(d) The determination of the rate law
For example, if the rate law is the one shown in eqn 17A.4, with or a reactant) is raised in a rate law of this kind is the order of
concentrations expressed in mol dm−3, then the units of kr will the reaction with respect to that species. A reaction with the The determination of a rate law is simplified by the isolation log(xy) = log x + log y
be dm3 mol−1 s−1 because rate law in eqn 17A.4 is first order in A and first order in B. method, in which all the reactants except one are present in
The overall order of a reaction with a rate law like that in eqn large excess. The dependence of the rate on each of the re- log v0 = log (kr ,eff [A]0a ) = log kr ,eff + log[A]0a (17A.11b)
kr [A] [B] 17A.7 is the sum of the individual orders, a + b + … . The over- actants can be found by isolating each of them in turn—by
!##"## $ !#"# $ !#"# $
dm3 mol −1 s −1 × moldm −3 × moldm −3 = moldm −3 s −1 all order of the rate law in eqn 17A.4 is 1 + 1 = 2; the rate law is having all the other substances present in large excess—and log xa = a log x
therefore said to be second-order overall. piecing together a picture of the overall rate law.
A reaction need not have an integral order, and many If a reactant B is in large excess its concentration is nearly = log kr ,eff + a log [A]0
which are the units of v. If the concentrations are expressed gas-phase reactions do not. For example, a reaction with the constant throughout the reaction. Then, although the true
in molecules cm−3, and the rate in molecules cm−3 s−1, then the rate law rate law might be v = kr[A][B]2, the current value of [B] can This equation has the form of the equation for a straight line:
rate constant is expressed in cm3 molecule−1 s−1. The approach be approximated by its initial value [B]0 (from which it hardly
just developed can be used to determine the units of the rate v = kr[A]1/2[B] (17A.8) changes in the course of the reaction) to give % !"
y
# #
intercept
$ !#"# $
slope × x

constant from rate laws of any form. log v0 = log kr ,eff + a log [A]0 (17A.11c)
is half order in A, first order in B, and three-halves order 2
A pseudofirst-
v = kr,eff [A], with kr,eff = kr[B] order reaction, (17A.10a)
Brief illustration 17A.2 overall. 0
B in excess It follows that, for a series of initial concentrations, a plot
of the logarithms of the initial rates against the logarithms of
The rate constant for the reaction O(g) + O3(g) → 2 O2(g) is Brief illustration 17A.3 Because the true rate law has been forced into first-order form the initial concentrations of A should be a straight line, and
8.0 × 10−15 cm3 molecule−1 s −1 at 298 K. To express this rate con- by assuming a constant B concentration, the effective rate law that the slope of the graph is a, the order of the reaction with
stant in dm3 mol−1 s −1, make use of the relation 1 cm = 10−1 dm The experimentally determined rate law for the gas-phase is classified as a pseudofirst-order rate law and kr,eff is called respect to A.
to convert the volume: reaction H2(g) + Br2(g) → 2 HBr(g) is given by eqn 17A.6. the effective rate constant for a given, fixed concentration of B.
In the rate law the concentration of H2 appears raised to the If, instead, the concentration of A is in large excess, and hence Example 17A.2
(10−1 dm)3 Using the method of initial rates
! power +1, so the reaction is first order in H2. However, the effectively constant, then the original rate law simplifies to
kr = 8.0 × 10−15 cm3 molecule−1 s −1 concentrations of Br2 and HBr do not appear as a single term The recombination of I atoms in the gas phase in the presence
−18 3
= 8.0 × 10 dm molecule s −1 −1 raised to a power, so the reaction has an indefinite order with v = k′r,eff [B]2, now with k′r,eff = kr[A]0 of argon was investigated and the order of the reaction was
respect to both Br2 and HBr, and an indefinite order overall. A pseudosecond-order reaction, A in excess (17A.10b) determined by the method of initial rates. The initial rates of
Now note that the number of molecules can be expressed reaction of 2 I(g) + Ar(g) → I2(g) + Ar(g) were as follows:
as an amount in moles by division by Avogadro’s constant This pseudosecond-order rate law is also much easier to ana-
[I]0/(10−5 mol dm−3) 1.0 2.0 4.0 6.0
expressed as molecules per mole: Some reactions obey a zeroth-order rate law, and therefore lyse and identify than the complete law. Note that the order of
v0/(mol dm−3 s−1) (a) 8.70 × 10−4 3.48 × 10−3 1.39 × 10−2 3.13 × 10−2
have a rate that is independent of the concentration of the the reaction and the form of the effective rate constant change
−18 3 −1 −1
kr = 8.0 × 10 dm molecule s reactant (so long as some is present). Thus, the catalytic according to whether A or B is in excess. In a similar manner, (b) 4.35 × 10−3 1.74 × 10−2 6.96 × 10−2 1.57 × 10−1

 1 molecule  −1
−1
decomposition of phosphine (PH3) on hot tungsten at high a reaction may even appear to be zeroth order. Many reactions (c) 8.69 × 10−3 3.47 × 10−2 1.38 × 10−1 3.13 × 10−1
= 8.0 × 10−18 dm3 ×  s
 6.022 × 10 molecules mol  pressures has the rate law in aqueous solution that are reported as first or second order
23 −1
The Ar concentrations are (a) 1.0 × 10−3 mol dm−3, (b) 5.0 ×
−18 23 3 −1 −1 are actually pseudofirst or pseudosecond order: the solvent 10−3 mol dm−3, and (c) 1.0 × 10−2 mol dm−3. Find the orders of
= 8.0 × 10 × 6.022 × 10 dm mol s
v = kr (17A.9) water, for instance, might participate in a reaction but it is in reaction with respect to I and Ar, and the rate constant.
= 4.8 × 106 dm3 mol −1 s −1 such large excess that its concentration remains constant.
This law means that PH3 decomposes at a constant rate until it In the method of initial rates, which is often used in con- Collect your thoughts You need to identify sets of data in
has entirely disappeared. junction with the isolation method, the instantaneous rate is which only one reactant is changing (such as each row of data
for constant [Ar]). The identification of order from such data
A practical application of a rate law is that once the law and As seen in Brief illustration 17A.3, when a rate law is not of measured at the beginning of the reaction for several different
involves the application of eqn 17A.11c, plotting the logarithm
the value of the rate constant are known, it is possible to pre- the form in eqn 17A.7, the reaction does not have an overall initial concentrations of the isolated reactant. If the initial rate
of the rate against the logarithm of a concentration of one
dict the rate of reaction from the composition of the mixture. order and might not even have definite orders with respect to is doubled when the concentration of an isolated reactant A is
of the reactants (in this case, arbitrarily chosen to be I). So,
Moreover, as demonstrated in Topic 17B, by knowing the rate each participant. doubled, then the reaction is first order in A; if the initial rate
first tabulate the logarithms of the concentrations of I and
law, it is also possible to predict the composition of the reac- These remarks point to three important tasks: is quadrupled, then the reaction is second order in A. More the rates for the values at constant [Ar]0 (i.e. for each row of
tion mixture at a later stage of the reaction. A rate law also formally, with an eye on developing a graphical method for data). The slope gives the order with respect to [I] and the
• To identify the rate law and obtain the rate constant from
provides evidence used to assess the plausibility of a proposed determining the order, suppose the rate law for a reaction with intercept at log [I]0 = 0 gives log kr,eff, with a different value for
the experimental data. This aspect is discussed in this
mechanism of the reaction. This application is developed in A isolated is each [Ar]0. The effective rate constant obtained in this way is
Topic.
Topic 17E. kr,eff = kr[Ar]0b, so to extract kr and b, take logarithms, as in the
• To account for the values of the rate constants and explain v = kr[A]a text, to obtain
their temperature dependence. This task is undertaken in
(c) Reaction order Topic 17D. Then the initial rate of the reaction, v 0, is given by the initial
log kr,eff = log kr + b log [Ar]0
Many reactions are found to have rate laws of the form • To construct reaction mechanisms consistent with the concentration of A: Now realize that you need to plot the log kr,eff found in the first
rate law. The techniques for doing so are introduced in part of the solution against log [Ar]0. Then the slope gives b
v = kr[A]a[B]b … (17A.7) Topic 17E. v 0 = kr,eff [A]0a Initial rate of an ath-order reaction (17A.11a) and the intercept at log [Ar]0 = 0 gives log kr.
17A The rates of chemical reactions 729 730 17 Chemical kinetics

0 [Ar]0/(mol dm−3) 1.0 × 10−3 5.0 × 10−3 1.0 × 10−2 ☐ 3. The rate of reaction is defined in terms of the extent ☐ 5. The order of a reaction is the power to which a partici-
log([Ar]0/mol dm−3) −3.00 −2.30 −2.00
of reaction in such a way that it is independent of the pant is raised in the rate law; the overall order is the
species being considered. sum of these powers.
log(kr,eff/dm3 mol−1 s−1) 6.94 7.64 7.93
log(v0/mol dm–3 s–1)

–1 ☐ 4. A rate law is an expression for the reaction rate in


Figure 17A.5b shows the plot of log{kr,eff/(dm3 mol−1 s −1)} terms of the concentrations of the species that occur
against log{[Ar]0/(mol dm−3)}. The slope is 1, so b = 1 and the in the overall chemical reaction.
–2
reaction is first order with respect to Ar. The intercept at
7.93
7.64 log{[Ar]0/(mol dm−3)} = 0 is log{kr,eff/(dm3 mol−1 s −1)} = 9.94, so
6.94 kr = 8.7 × 109 dm6 mol−2 s −1. The overall (initial) rate law is
0
therefore v = kr[I]02[Ar]0.
–3
(a)
–5 –4.8 –4.6 –4.4
log([I]0/mol dm ) –3
–4.2 –4
A note on good practice When taking the common logarithm Checklist of equations
of a number of the form x.xx × 10n (with n < 10) there are four
8
significant figures in the answer (for instance, log 1.23 × 104 = Property Equation Comment Equation number
4.090): the figure before the decimal point is simply the power of
log(kr,eff/dm3 mol–1 s–1)

Rate of a reaction v = (1/V)(dξ/dt) Definition 17A.2


10. Conversely, when taking the common antilogarithm of y.yyy,
v = (1/ν J )(d[J]/dt) Constant-volume system 17A.3b
7.5 there are three significant figures in the answer (for instance,
105.678 = 4.76 × 105). Rate law (in some cases) v = kr[A]a[B]b… a, b, … : orders; a + b + … : overall order 17A.7
Method of initial rates log v0 = log kr ,eff + a log[A]0 Reactant A isolated 17A.11c
9.91
7 Self-test 17A.2 The initial rate of a certain reaction depended
0 on the concentration of a substance J as follows:

[J]0/(10−3 mol dm−3) 5.0 10.2 17 30


6.5
–3 –2.8 –2.6 –2.4 –2.2 –2 v0/(10−7 mol dm−3 s−1) 3.6 9.6 4 130
(b) log([Ar]0/mol dm–3)
Find the order of the reaction with respect to J and the rate
Figure 17A.5 Analysis of the data in Example 17A.2. (a) Plots for constant.
finding the order with respect to I. The intercepts at log [I]0 = 0 Answer: 2, 1.6 × 10 −2 dm3 mol−1 s −1
are far to the right, and are shown in the inset. (b) The plots for
finding the order with respect to Ar and the rate constant kr.
The intercept at log [Ar]0 = 0 is far to the right, and is shown in
the inset.
The method of initial rates might not reveal the full rate
law because once the products have started to be generated
they might participate in the reaction and affect its rate. For
The solution The data give the following points for the graph: example, in the reaction between H2 and Br2, the rate law in
eqn 17A.6 shows that the rate depends on the concentration
log([I]0/mol dm−3) −5.00 −4.70 −4.40 −4.22 of the product HBr. To avoid this difficulty, the rate law should
log(v0/mol dm−3 s−1) (a) −3.060 −2.458 −1.857 −1.504 be fitted to the data throughout the reaction. The fitting may
(b) −2.362 −1.759 −1.157 −0.804 be done, in simple cases at least, by using a proposed rate law
(c) −2.061 −1.460 −0.860 −0.504 to predict the concentration of any component at any time,
and comparing it with the data; methods based on this pro-
The graph of the data for varying [I] but constant [Ar] is cedure are described in Topic 17B. A rate law should also be
shown in Fig. 17A.5a. The slopes of the lines are 2, so the tested by observing whether the addition of products or, for
reaction is second order with respect to I. The effective rate gas-phase reactions, a change in the surface-to-volume ratio in
constants kr,eff are as follows: the reaction chamber affects the rate.

Checklist of concepts
☐ 1. The rates of chemical reactions are measured by using ☐ 2. The instantaneous rate of consumption of a reactant
techniques that monitor the concentrations of species or formation of a product is the slope of the tangent to
present in the reaction mixture. Examples include real- the graph of concentration against time (expressed as a
time and quenching procedures, flow and stopped- positive quantity).
flow techniques, and flash photolysis.

732 17 Chemical kinetics

TOPIC 17B Integrated rate laws [A], so make these values the matching limits of the integrals
and write
The integrated rate law in eqn 17B.2b can be expressed in
terms of the concentration of the product P by noting that for a
reaction A → P the increase in the concentration of P matches
d[A]
[A] t
∫[A]0 [A] = − kr ∫0 dt the decrease in concentration of A. If it is assumed that there
is no P present at the start of the reaction, then [P] = [A]0 − [A],
Because the integral of 1/x is ln x + constant, the left-hand side and hence [A] = [A]0 − [P]. This expression for [A] can be sub-
of this expression is stituted into eqn 17B.2b to give
1 [A]0 −[P]
➤ Why do you need to know this material? ln = − krt , [P] = [A]0 (1− e − k t )
r
(17B.2c)
Integral A.2
!#"# $ [A]0
You need the integrated rate law if you want to predict [A] d[A] [A] [A]
the composition of a reaction mixture as it approaches ∫[A]0 [A] = ln [A] + constant [A]0 = ln [A] − ln [A]0 = ln [A]0 A useful indication of the rate of a first-order chemical reac-
Increasing kr tion is the half-life, t1/2, of a substance, the time taken for the
[A]t /[A]0

equilibrium. The integrated rate law is also the basis of


concentration of a reactant to fall to half its initial value. This
determining the order and rate constants of a reaction, The right-hand side integrates to −krt, and so
quantity is readily obtained from the integrated rate law. Thus,
which is a necessary step in the formulation of the mecha-
the time for the concentration of A to decrease from [A]0 to
nism of the reaction. [A] (17B.2b)
2 [A]0 in a first-order reaction is given by eqn 17B.2b as
1
ln = − krt , [A] = [A]0e − krt
[A]0 Integrated first-
➤ What is the key idea? order rate law
0
A rate law is a differential equation that can be integrated 0 Time, t
1
[A]0
krt1/2 = − ln 2
= − ln 12 = ln2
to find how the concentrations of reactants and products [A]0
Figure 17B.1 The linear decay of the reactant in a zeroth-order Equation 17B.2b shows that if ln([A]/[A]0) is plotted against
change with time.
reaction. t, then a first-order reaction will give a straight line of slope Hence
➤ What do you need to know already? −kr. Some rate constants determined in this way are given ln2
t1/2 = Half-life
in Table 17B.1. The second expression in eqn 17B.2b shows kr (17B.3)
You need to be familiar with the concepts of rate law, [first-order reaction]
where [A] is the concentration of A at t and [A]0 is the initial that in a first-order reaction the reactant concentration de-
reaction order, and rate constant (Topic 17A). The manipu-
concentration of A. This expression rearranges to creases exponentially with time with a rate determined by kr (Note that ln 2 = 0.693.) The main point to note about this
lation of simple rate laws requires only elementary tech-
(Fig. 17B.2). result is that for a first-order reaction, the half-life of a reac-
niques of integration (see the Resource section for standard
[A] = [A]0 − krt Integrated zeroth-order rate law (17B.1) tant is independent of its initial concentration. Therefore, if
integrals).
the concentration of A at some arbitrary stage of the reaction
This expression applies until all the reactant has been used up is [A], then it will have fallen to 12 [A] after a further interval of
Table 17B.1 Kinetic data for first-order reactions*
at t = [A]0/kr, after which [A] remains zero (Fig. 17B.1). (ln 2)/kr. Some half-lives are given in Table 17B.1.
Rate laws (Topic 17A) are differential equations, which can be Reaction Phase θ/°C kr/s−1 t1/2
integrated to predict how the concentrations of the reactants
2 N2O5 → 4 NO2 + O2 g 25 3.38 × 10−5 5.70 h Example 17B.1
and products change with time. Even the most complex rate Analysing a first-order reaction
laws may be integrated numerically. However, in a number of 17B.2 First-order reactions Br2(l) 25 4.27 × 10−5 4.51 h
The variation in the partial pressure of azomethane with time
simple cases analytical solutions, known as integrated rate C2H6 → 2 CH3 g 700 5.36 × 10−4 21.6 min
was followed at 600 K, with the results given below. Confirm
laws, are easily obtained and prove to be very useful. Consider the first-order rate law * More values are given in the Resource section.
that the decomposition CH3N2CH3(g) → CH3CH3(g) + N2(g)
d[A] is first-order in azomethane, and find the rate constant and
= − kr [A] (17B.2a) half-life at 600 K.
dt
1
17B.1 Zeroth-order reactions This equation can be integrated to show how the concentra- t/s 0 1000 2000 3000 4000
tion of A changes with time. 0.8 p/Pa 10.9 7.63 5.32 3.71 2.59
The rate of a zeroth-order reaction of the type A → P is con-
stant (so long as reactant remains), so How is that done? 17B.1 Deriving the first-order 0.6 Collect your thoughts To confirm that a reaction is first order,
[A]/[A]0

kr,small
plot ln([A]/[A]0) against time and expect a straight line.
d[A] integrated rate law Because the partial pressure of a gas is proportional to its con-
= − kr 0.4
dt centration, an equivalent procedure is to plot ln(p/p0) against
First, rearrange eqn 17B.2a into
It follows that the change in concentration of A is simply its 0.2 t. If a straight line is obtained, its slope can be identified with
d[A] kr,large −kr. The half-life is then calculated from kr by using eqn 17B.3.
rate of consumption (which is −kr) multiplied by the time t for = − kr dt
[A] 0
which the reaction has been in progress: 0 1 2 3 The solution Draw up the following table by using p0 = 10.9 Pa:
kr,smallt
and recognize that kr is a constant independent of t. Initially
[A] − [A]0 = − krt (at t = 0) the concentration of A is [A]0, and at a later time t it is Figure 17B.2 The exponential decay of the reactant in a first-
t/s 0 1000 2000 3000 4000

order reaction. The larger the rate constant, the more rapid is the p/p0 1 0.700 0.488 0.340 0.0238
decay: here kr,large = 3kr,small. ln(p/p0) 0 −0.357 −0.717 −1.078 −1.437
17B Integrated rate laws 733 734 17 Chemical kinetics

Figure 17B.3 shows the plot of ln(p/p0) against t/s. The plot The concentration is [A]0 at t = 0 and at a later time t it is [A]. As in the case of the first-order reactions, eqn 17B.4b can pears entails the disappearance of one B). Then eqn 17B.7a
is straight, confirming a first-order reaction, and its slope is Therefore, be rewritten in terms of the product P given the stoichiometry becomes
−3.6 × 10−4. Therefore, kr = 3.6 × 10−4 s −1. It follows from eqn A → P by noting that [A] = [A]0 − [P]. With this substitution,
17B.3 that the half-life is d[A]
Integral A.1
!
#"# $ and after some rearrangement, = − kr ([A]0 − x )([B]0 − x )
dt
[A] d[A] t
ln2 −∫
t1/2 = = 1.9 ×103 s 2 = kr ∫ dt
3.6 ×10−4 s −1 [A]0 [A] 0 [P] k t[A]02 Because [A] = [A]0 − x, it follows that d[A]/dt = −dx/dt and the
= krt , [P] = r (17B.4c)
([A]0 −[P])[A]0 1+[A]0 krt rate law may be written
The integral on the left-hand side (including the minus sign) is
0 dx
It follows from eqn 17B.4b by substituting t = t1/2 and = kr ([A]0 − x )([B]0 − x )
[A]
[A] = 12 [A]0 that the half-life of a species A that is consumed dt
1 1 1
+ constant = − in a second-order reaction is
[A] [A]
[A] [A]0 Step 2 Integrate the rate law
0
–0.5
The initial condition is that x = 0 when t = 0; so the integra-
1
ln(p/p0)

and that of the right-hand side is krt. It follows that Half-life


t1/2 = (17B.5) tions required are
kr [A]0 [second-order reaction]
x dx t
–1
1 1 [A]0 (17B.4b) Therefore, unlike a first-order reaction, the half-life of a sub- ∫ 0 ([A]0 − x )([B]0 − x ) r ∫0
= k dt
− = krt , [A] =
[A] [A]0 1+ krt[A]0 Integrated second-
stance in a second-order reaction depends on the initial con-
order rate law The right-hand side evaluates to krt. The integral on the left
centration. A practical consequence of this dependence is that
–1.5 is evaluated by using the method of partial fractions (see The
0 1 2 3 4 species that decay by second-order reactions (which includes
t/(103 s) chemist’s toolkit 30 and the list of integrals in the Resource
Equation 17B.4b shows that for a second-order reaction a some environmentally harmful substances) may persist in low
section):
Figure 17B.3 The determination of the rate constant of a plot of 1/[A] against t is expected to be a straight line. The slope concentrations for long periods because their half-lives are
first-order reaction: a straight line is obtained when ln [A] (or, of the graph is kr. Some rate constants determined in this way long when their concentrations are low. In general, for an nth-
Integral A.3
as here, ln p/p0) is plotted against t; the slope is −kr. The data are given in Table 17B.2. The alternative form of the equation order reaction (with n > 1) of the form A → P, the half-life is !###"###$
plotted are from Example 17B. can be used to predict the concentration of A at any time after
the start of the reaction. It shows that the concentration of A
related to the rate constant and the initial concentration of A
by (see Problem P17B.15)
x dx 1 [A]
{ [B]
∫0 ([A]0 − x )([B]0 − x ) = [B]0 −[A]0 ln [A]0 −0 x − ln [B]0 −0 x }
Self-test 17B.1 In a particular experiment, it was found that approaches zero more slowly than in a first-order reaction
n−1
the concentration of N2O5 in liquid bromine varied with time with the same initial rate (Fig. 17B.4). 2 −1 Half-life
The two logarithms can be combined as follows:
t1/2 = (17B.6)
as follows: (n −1)kr [A]n0−1 [nth-order reaction, n > 1]
[A]0 [B]0 [A] [B]
t/s 0 200 400 600 1000 Table 17B.2 Kinetic data for second-order reactions* ln − ln = ln 0 − ln 0
Another type of second-order reaction is one that is first [A]0 − x [B]0 − x [A] [B]
!#"# $ !"
# # $
[N2O5]/(mol dm−3) 0.110 0.073 0.048 0.032 0.014 order in each of two reactants A and B:
Reaction Phase θ/°C kr/(dm3 mol−1 s−1) [A] [B]
Confirm that the reaction is first order in N2O5 and determine 2 NOBr → 2 NO + Br2 g 10 0.80 1 1
the rate constant. d[A] = ln − ln
2 I → I2 g 23 7 × 109 = − kr [A][B] (17B.7a) [A]/[A]0 [B]/[B]0
Answer: kr = 2.1 × 10 −3 s −1 dt
* More values are given in the Resource section.
[B]/[B]0
An example of a reaction that may have this rate law is A + = ln
[A]/[A]0
B → P. This rate law can be integrated to find the variation of
1
the concentrations [A] and [B] with time. Step 3 Finalize the expression
17B.3 Second-order reactions 0.8 Combining all the results so far gives

The integrated form of the second-order rate law, How is that done? 17B.3 Deriving a second-order
0.6 [B]/[B]0 (17B.7b)
[A]/[A]0

d[A] kr,small integrated rate law for A + B → P ln = ([B]0 − [A]0 )krt


[A]/[A]0 Integrated rate law
= − kr [A]2 (17B.4a) [second-order reaction of
dt 0.4 Before integrating eqn 17B.7a, it is necessary to know how the the type A + B → P, with
kr,large concentration of B is related to that of A, which can be found [A]0 ≠ [B]0]
can be found by much the same method as for first-order 0.2 from the reaction stoichiometry and the initial concentrations
reactions.
[A]0 and [B]0 which in this derivation are taken to be unequal.
Therefore, a plot of the expression on the left against t should
0 Follow these steps.
How is that done? 17B.2 Deriving a second-order 0 1 2 3 be a straight line from which kr can be obtained. As shown in
kr,small[A]0t Step 1 Rewrite the rate law by considering the reaction the following Brief illustration, the rate constant may be esti-
integrated rate law stoichiometry
Figure 17B.4 The variation with time of the concentration of mated quickly by using data from only two measurements.
To integrate eqn 17B.4a, first rearrange it into a reactant in a second-order reaction. The grey lines are the It follows from the reaction stoichiometry that when the Similar calculations may be carried out to find the
d[A] corresponding decays in a first-order reaction with the same concentration of A has fallen to [A]0 − x, the concentration integrated rate laws for other orders, and some are listed in
= − kr dt initial rate. For this illustration, kr,large = 3kr,small. of B will have fallen to [B]0 − x (because each A that disap- Table 17B.3.
[A]2

17B Integrated rate laws 735 736 17 Chemical kinetics

The chemist’s toolkit 30 Brief illustration 17B.1


Integration by the method of Checklist of concepts
partial fractions
Consider a second-order reaction of the type A + B → P car-
To solve an integral of the form ried out in a solution. Initially, the concentrations of reactants ☐ 1. An integrated rate law is an expression for the concen- ☐ 3. Analysis of experimental data using integrated rate
1 are [A]0 = 0.075 mol dm−3 and [B]0 = 0.050 mol dm−3. After tration of a reactant or product as a function of time laws allow for the prediction of the composition of a
I=∫ dx 1.0 h the concentration of B has fallen to [B] = 0.020 mol dm−3. (Table 17B.3). reaction system at any stage, the verification of the rate
(a − x )(b − x )
where a and b are constants with a ≠ b, use the method of par- Because the change in the concentration of B is the same as ☐ 2. The half-life of a reactant is the time it takes for its law, and the determination of the rate constant.
tial fractions in which a fraction that is the product of terms that of A (and equal to x), it follows that during this time concentration to fall to half its initial value.
(as in the denominator of this integrand) is written as a sum of interval
fractions. To implement this procedure write the integrand as x = (0.050 − 0.020) mol dm−3 = 0.030 mol dm−3
1 1  1 1 
= −
(a − x )(b − x ) b − a  a − x b − x  Therefore, the concentration of A after 1.0 h is

Then integrate each term on the right. It follows that [A] = [A]0 − x = (0.075 − 0.030) mol dm−3 = 0.045 mol dm−3
Checklist of equations
Integral Integral
and you are given that [B] = 0.020 mol dm−3. It follows from Property Equation Comment Equation number
A.2 A.2
!"# !"# eqn 17B.7b that
( )
Integrated rate law [A] = [A]0 −krt Zeroth order, A → P 17B.1
1 dx dx
b−a ∫ a− x ∫ b− x
I= − Integrated rate law ln([A]/[A]0) = −krt or [A] = [A]0 e−krt First order, A → P 17B.2b
1 0.020/0.050
kr = ln
1  1 1  ((0.050 − 0.075)mol dm −3 ) × (3600 s) 0.045/0.075 Half-life t1/2 = (ln 2)/kr First order, A → P 17B.3
= ln − ln + constant
b − a  a − x b − x  −3
= 4.5 ×10 dm mol s 3 −1 −1 Integrated rate law 1/[A] − 1/[A]0 = krt or [A] = [A]0/(1 + krt[A]0) Second order, A → P 17B.4b
Half-life t1/2 = 1/kr[A]0 Second order, A → P 17B.5
t1/2 = (2n−1 − 1)/(n − 1)kr[A]0n−1 nth order, n > 1 17B.6
Table 17B.3 Integrated rate laws Integrated rate law ln{([B]/[B]0)/([A]/[A]0)} = ([B]0 − [A]0)krt Second order, A + B → P 17B.7b

Order Reaction Rate law and its integrated form* t1/2


0 A→P v = kr [A]0/2kr
krt = [P] for 0 ≤ [P] ≤ [A]0,
[A] = [A]0 − krt for 0 ≤ [A] ≤ [A]0
1 A→P v = kr[A] (ln 2)/kr
[A]0 −k t
krt = ln , [A] = [A]0e − krt , [P] = [A]0 (1 − e r )
[A]
2 A→P v = kr[A] 2
1/kr[A]0

krt =
[P] , [A] =
[A]0
, [P] = r
k t[A]02
[A]0 ([A]0 −[P]) 1 + krt[A]0 1 +[A]0 krt

A+B→P v = kr[A][B]
1 [A] ([B]0 −[P]) ,
krt = ln 0
[B]0 −[A]0 ([A]0 −[P])[B]0

[B]/[B]0 [A]0[B]0 (1 − e([B]0 −[A]0 )krt )


ln = ([B]0 − [A]0 )krt , [P] =
[A]/[A]0 [A]0 −[B]0 e([B]0 −[A]0 )krt
A + 2B → P v = kr[A][B]
[A]0[B]0 (1 − e( 0 0 ) r )
[B] − 2[A] k t
1 [A] ([B]0 − 2[P]) ,
krt = ln 0 [P] =
[B]0 − 2[A]0 ([A]0 −[P])[ B]0 2[A]0 −[B]0 e([B]0 −2[A]0 )krt
3 A + 2B → P v = kr[A][B]2
2[P] 1 [A] ([B]0 − 2[P])
krt = + ln 0
(2[A]0 −[B]0 )([B]0 − 2[P])[B]0 (2[A]0 −[B]0 )2 ([A]0 −[P])[ B]0

[P] must be determined graphically or numerically


2n−1 −1
n≥2 A→P v = kr[A]n
(n −1)kr [A]n0−1
1  1 1 
krt = −
n −1  ([A]0 −[P])n−1 [A]n0−1 

No simple general solution for [P] for n > 3


* v = d[P]/dt
738 17 Chemical kinetics

TOPIC 17C Reactions approaching (As explained in Topic 6A, the replacement of activities by
the numerical values of molar concentrations is justified if the
system is treated as ideal.) Exactly the same conclusion can be
Final
equilibrium

Concentration, [A]
equilibrium reached—more simply, in fact—by noting that, at equilibrium,
the forward and reverse rates must be the same, so Exponential
relaxation

kr [A]eq = kr′[B]eq (17C.7) Initial


equilibrium
This relation rearranges into eqn 17C.6. The theoretical im-
portance of eqn 17C.6 is that it relates a thermodynamic quan- T1 T2

d[A] tity, the equilibrium constant, to quantities relating to rates. Time, t


➤ Why do you need to know this material? = − kr [A] + kr′[B] (17C.2) Its practical importance is that if one of the rate constants can
dt Figure 17C.2 The relaxation to the new equilibrium composition
All reactions tend towards equilibrium and the rate laws be measured, then the other may be obtained if the equilib-
when a reaction initially at equilibrium at a temperature T1 is
can be used to describe the changing concentrations as If the initial concentration of A is [A]0, and no B is present ini- rium constant is known.
subjected to a sudden change of temperature, which takes it to T2.
they approach that composition. The analysis of the time- tially, then at all times [A] + [B] = [A]0. Therefore, Equation 17C.6 is valid even if the forward and reverse
dependence also reveals the connection between rate reactions have different orders, but in that case more care
d[A] needs to be taken with units. For instance, if the reaction A +
constants and equilibrium constants. = − kr [A] + kr′([A]0 − [A])
dt (17C.3) B → C is second-order in the forward direction and first-order 17C.2 Relaxation methods
➤ What is the key idea? = − (kr + kr′ )[A] + kr′[A]0 in the reverse direction, then the condition for equilibrium is
Both forward and reverse reactions must be incorpo- kr[A]eq[B]eq = kr′[C]eq and the dimensionless equilibrium con- The term relaxation denotes the return of a system to equilib-
rated into a reaction scheme in order to account for the The solution of this first-order differential equation (as may be stant in full dress is rium. It is used in chemical kinetics to indicate that an exter-
approach to equilibrium. checked by differentiation, Problem P17C.1) is nally applied influence has shifted the equilibrium position of
[C]eq / c −○−  [C]  −○− kr −○− a reaction, often suddenly, and that the concentrations of the
kr′ + kr e − ( k + k′ )t K= = c = ×c
([A]eq / c −○− )([B]eq / c −○− )  [A][B]  eq
r r

➤ What do you need to know already? [A] = [A]0 , [B] = [A]0 − [A] (17C.4) kr′ species involved then adjust towards the equilibrium values
kr + kr′
You need to be familiar with the concepts of rate law, reac- characteristic of the new conditions (Fig. 17C.2).
Figure 17C.1 shows the time dependence predicted by this The presence of c = 1 mol dm−3 in the last term ensures that the Consider the response of reaction rates to a tempera-

tion order, and rate constant (Topic 17A), integrated rate


laws (Topic 17B), and equilibrium constants (Topic 6A). As equation. ratio of second-order to first-order rate constants, with their ture jump, a sudden change in temperature. As explained in
in Topic 17B, the manipulation of simple rate laws requires As t → ∞, the exponential term in eqn 17C.4 decreases to different units, is turned into a dimensionless quantity. Topic 6B, provided ΔrH is non-zero the equilibrium compo-

only elementary techniques of integration. zero and the concentrations reach their equilibrium values, sition of a reaction depends on the temperature, so a sudden
which are therefore shift in temperature acts as a perturbation on the system. One
Brief illustration 17C.1 way of achieving a temperature jump is to subject the sample
kr′[A]0 k [A]
[A]eq = , [B]eq = [A]0 − [A]eq = r 0 (17C.5) that has been made conducting by the addition of ions to an
kr + kr′ kr + kr′
In practice, most kinetic studies are made on reactions that are The rate constants of the forward and reverse reactions for a electric discharge; bursts of microwave radiation or intense
far from equilibrium and if products are in low concentration It follows that the equilibrium constant of the reaction is dimerization reaction were found to be 8.0 × 108 dm3 mol−1 s −1 electromagnetic pulses from lasers can also be used. Electrical
the reverse reactions are unimportant. Close to equilibrium, (second order) and 2.0 × 106 s −1 (first order). The equilibrium discharges can achieve temperature jumps of between 5 and
[B]eq kr
however, the products might be so abundant that the reverse K= = (17C.6) constant for the dimerization is therefore 10 K in about 1 μs. The high energy output of a pulsed laser
[A]eq kr′
reaction must be taken into account. is sufficient to generate temperature jumps of between 10 and
8.0 ×108 dm3 mol −1 s −1
K= ×1moldm −3 = 4.0 ×102 30 K within nanoseconds in aqueous samples.
2.0 ×106 s −1
1 The response of a system to a sudden temperature increase
A
can be analysed by considering the rate laws for the forward
First-order reactions
17C.1 0.8 and reverse reactions and the temperature dependence of the
approaching equilibrium For a more general reaction, the overall equilibrium con- rate constants.
0.6
stant can be expressed in terms of the rate constants for all the
[J]/[J]0

Considering a reaction in which A forms B and both forward intermediate stages of the reaction mechanism (see Problem
0.4 How is that done? 17C.1 Exploring the response to a
and reverse reactions are first order (as in some isomerizations): P17C.4):
temperature jump
d[A] 0.2 B
A→B = − kr [A] ka kb The equilibrium constant in First consider a simple A ! B equilibrium that is first order in
dt K= × ×! (17C.8)
d[A]
(17C.1)
0
ka′ kb′ terms of the rate constants each direction and then an equilibrium A ! B + C that is first
B→ A = kr′[B] 0 1 2 3 order forward and second order reverse. In each case, when the
dt (kr + kr’)t
where the kr are the rate constants for the individual steps and temperature is increased suddenly, the rate constants change
The concentration of A is reduced by the forward reaction (at Figure 17C.1 The approach of concentrations to their equilibrium the kr′ are those for the corresponding reverse steps. The ap- from their original values to the new values kr and kr′ charac-
a rate kr[A]) but it is increased by the reverse reaction (at a rate propriate powers of c should be included in each factor if the

values as predicted by eqn 17C.4 for a reaction A ! B that is first teristic of the new temperature, but the concentrations of A
kr′[B]). The net rate of change at any stage is therefore order in each direction, and for which kr = 2kr′. orders of the forward and reverse reactions are different. and B remain for an instant at their old equilibrium values.

17C Reactions approaching equilibrium 739 740 17 Chemical kinetics

(a) The equilibrium A ! B (first order forward and reverse) As before, d[A]/dt = dx/dt; the solution of this differential
As the system immediately after the temperature jump is no equation is an exponential decay proportional to e − t/τ with Checklist of concepts
longer at equilibrium, it readjusts to the new equilibrium τ given by
concentrations, which are now given by kr[A]eq = kr′[B]eq, and ☐ 1. There is a relation between the equilibrium constant, ☐ 2. In relaxation methods of kinetic analysis, the equi-
1 a thermodynamic quantity, and the rate constants of librium position of a reaction is shifted suddenly and
it does so at a rate that depends on the new rate constants. Let = kr + kr′([B]eq +[C]eq )
τ
the deviation of [A] from its new equilibrium value be x, so the forward and reverse reactions (see below). then the time-dependence of the concentration of the
[A] = [A]eq + x; then the reaction stoichiometry implies that species involved is followed.
Step 2 Relate the equilibrium concentrations by introducing an
[B] = [B]eq − x. At the new temperature the concentration of A
equilibrium constant
changes as follows:
The equilibrium constant for the reaction (assuming ideal
d[A] solutions) is
dt
= − kr [A] + kr′[B] Checklist of equations
([B]eq / c−○− )([C]eq / c−○− ) [B]eq [C]eq
= − kr ([A]eq + x ) + kr′([B]eq − x ) K= = Property Equation Comment Equation number
([A]eq / c−○− ) [A]eq c−○−
Cancel Equilibrium constant in terms of rate constants K = ka / ka′ × kb / kb′ × ! Include c⦵ as appropriate 17C.8
!##"##$
= − kr [A]eq + kr′[B]eq − (kr + kr′ )x The reaction stoichiometry implies that the concentrations of Relaxation of an equilibrium A ! B after a temperature jump x = x 0e − t /τ
First order in each direction 17C.9a
B and C are the same, so τ = 1/(kr + kr′ )
= − (kr + kr′ )x
a
!# #"## $
Because d[A]/dt = dx/dt, this equation is a first-order dif- [B]eq = [C]eq = ( K[A]eq c −○− )1/2
ferential equation with a solution that resembles eqn 17B.2b.
If x0 is the deviation from equilibrium immediately after the and the time constant becomes
temperature jump, the time-dependence of x is
1 k 
1 (17C.9a) = k + 2akr′ = kr′  r + 2a
x = x 0e − t /τ τ= τ r  kr′ 
kr + kr′ Relaxation after a
temperature jump
[first-order reactions] Step 3 Identify the equilibrium constant of the reaction
You should now recognize that the ratio of the rate constants
(b) The equilibrium A ! B + C (first order forward and is a form of the equilibrium constant at the new temperature.
second order reverse) Specifically:
As in the previous derivation, the system immediately after
the temperature jump is no longer at equilibrium, so it read- [B]eq [C]eq
kr [A]eq = kr′[B]eq [C]eq and K =
justs to the new equilibrium concentrations, which are now [A]eq c −○−
given by kr[A]eq = kr′[B]eq[C]eq, and it does so at a rate that
depends on the new rate constants. Let the deviation of [A] implying that
from its new equilibrium value be x, so [A] = [A]eq + x; then
the reaction stoichiometry implies that [B] = [B]eq − x and kr [B]eq [C]eq
[C] = [C]eq − x. = = Kc−○−
kr′ [A]eq
Step 1 Set up and solve the rate equations
and therefore
At the new temperature the concentration of A changes as
follows: 1 −−
= kr′( Kc ○ + 2a)
τ
d[A]
= − kr [A]+ kr′[B][C]
dt
The time dependence of x is therefore
= − kr ([A]eq + x ) + kr′([B]eq − x )([C]eq − x )

!### #"####
0
$ %
Neglect 1
x = x 0e − t /τ τ = −− (17C.9b)
= − {kr + kr′([B]eq + [C]eq )}x − kr [A]eq + kr′ [ B ]eq [ C ]eq + kr′x 2 kr′( Kc ○ + 2a)
Relaxation after a
−−
= − {kr + kr′([B]eq + [C]eq )}x a = ( K[A]eq c ○ )1/2 temperature jump
[mixed-order reaction]
742 17 Chemical kinetics

TOPIC 17D The Arrhenius equation is ln(A/dm3 mol−1 s −1). Use a least-squares procedure to deter-
mine the slope and the intercept.
Brief illustration 17D.1

For a reaction with an activation energy of 50 kJ mol−1, an


The solution Draw up the following table:
increase in the temperature from 25 °C to 37 °C (body tem-
(103 K)/T 1.43 1.37 1.32 1.27 1.23 1.19 1.10 1.00 perature) corresponds to
ln(kr/dm3 mol−1 s−1) −4.51 −3.35 −2.25 −1.07 −0.24 0.77 3.00 4.98 kr ,2 50 ×103 Jmol −1  1 1 
ln = −
Now plot ln kr against (103 K)/T (Fig. 17D.2). The least-squares kr ,1 8.3145JK −1 mol −1  298K 310K 
ln A fit results in a line with slope −22.7 and intercept 27.7. 50 ×103  1 1 
➤ Why do you need to know this material? = − = 0.781…
Therefore, 8.3145  298 310 
Exploration of the dependence of reaction rates on tem-
perature leads to the formulation of theories that reveal Ea = −(−22.7) × (8.3145 J K−1 mol−1) × (103 K) = 189 kJ mol−1 By taking natural antilogarithms (that is, by forming ex),
Slope = –Ea/R kr,2 = 2.18kr,1. This result corresponds to slightly more than a
the details of the processes that occur when reactant mol-

ln kr
A = e27.7 dm3 mol−1 s −1 = 1.1 × 1012 dm3 mol−1 s −1 doubling of the rate constant as the temperature is increased
ecules meet and undergo reaction.
from 298 K to 310 K.
Note that A has the same units as kr.
➤ What is the key idea?
5 The fact that Ea is given by the slope of the plot of ln kr
The temperature dependence of the rate of a reaction
against 1/T leads to the following conclusions:
depends on the activation energy, the minimum energy
needed for reaction to occur in an encounter between 1/T
• A high activation energy signifies that the rate con-

ln{kr/(dm3 mol–1 s–1)}

Physical interpretation
reactants. Figure 17D.1 An Arrhenius plot, a plot of ln kr against 1/T, is a stant depends strongly on temperature.
straight line when the reaction follows the behaviour described
➤ What do you need to know already? • If a reaction has zero activation energy, its rate is
by the Arrhenius equation (eqn 17D.1). The slope is −Ea/R and the 0
You need to know that the rate of a chemical reaction is intercept at 1/T = 0 is ln A. independent of temperature.
expressed by a rate constant (Topic 17A). • A negative activation energy indicates that the rate
decreases as the temperature is raised.
Table 17D.1 Arrhenius parameters*
–5
For some reactions it is found that a plot of ln kr against
Chemical reactions usually go faster as the temperature is (1) First-order reactions Phase A/s−1 Ea/(kJ mol−1) 1 1.1 1.2 1.3 1.4 1/T does not give a straight line. It is still possible to define
(103 K)/T
raised. It is found experimentally for many reactions that a CH3NC → CH3CN gas 3.98 × 1013 160 an activation energy for these ‘non-Arrhenius reactions’ at a
plot of ln kr against 1/T gives a straight line with a negative 2 N2O5 → 4 NO2 + O2 gas 4.94 × 10 13
103.4 Figure 17D.2 The Arrhenius plot using the data in Example particular temperature by writing
slope, indicating that an increase in ln kr (and therefore an 17D.1.
(2) Second-order reactions Phase A/(dm3 mol−1 s−1) Ea/(kJ mol−1) d(1/T)/dT = −1/T 2
increase in kr) results from a decrease in 1/T (i.e. an increase
in T). OH + H2 → H2O + H gas 8.0 × 1010 42 Self-test 17D.1 Determine A and E a from the following data: Ea d ln kr d ln kr
− = = −T2
NaC2H5O + CH3I in ethanol 2.42 × 1011 81.6 R d(1/T ) dT
T/K 300 350 400 450 500
* More values are given in the Resource section. and therefore
kr/(dm3 mol−1 s−1) 7.9 × 106 3.0 × 107 7.9 × 107 1.7 × 108 3.2 × 108
17D.1The temperature dependence  d ln kr 
Ea = RT 2 
Activation energy
 dT 
Answer: 8 × 1010 dm3 mol−1 s −1, 23 kJ mol−1
[definition] (17D.3)
of reaction rates
Example 17D.1 Determining the Arrhenius parameters This expression is the formal definition of activation energy. It
The temperature dependence characteristic of a reaction Once the activation energy of a reaction is known, the value reduces to the earlier one (as the slope of a straight line) for a tem-
is normally expressed mathematically by introducing two The rate of the second-order decomposition of ethanal (acetal- of a rate constant kr,2 at a temperature T2 can be predicted from perature-independent activation energy (see Problem P17D.1).
parameters, one representing the intercept and the other the dehyde, CH3CHO) was measured over the temperature range its value kr,1 at another temperature T1. To do so, write Non-Arrhenius behaviour is sometimes a sign that quantum
slope of the straight line of a so-called ‘Arrhenius plot’ of 700–1000 K, and the rate constants are reported below. Find mechanical tunnelling is playing a significant role in the reaction
Ea and A. Ea Ea
ln kr against 1/T and writing the Arrhenius equation ln kr ,1 = ln A − ln kr ,2 = ln A − (Topic 7D). In biological reactions it might signal that an enzyme
RT1 RT2 has undergone a structural change and has become less efficient.
T/K 700 730 760 790 810 840 910 1000
Ea
ln kr = ln A − Arrhenius equation (17D.1) kr/
RT 0.011 0.035 0.105 0.343 0.789 2.17 20.0 145
(dm3 mol−1 s−1) and then subtract the first from the second to obtain
The parameter A, which is obtained from the intercept of the The interpretation of the
17D.2
Collect your thoughts According to eqn 17D.1, the data can Ea E
line at 1/T = 0 (at infinite temperature, Fig. 17D.1), is called
be analysed by plotting ln(kr/dm3 mol−1 s −1) against 1/(T/K), or
ln kr ,2 − ln kr ,1 = − + a
RT2 RT1 Arrhenius parameters
the frequency factor (and still commonly the pre-exponential
more conveniently (103 K)/T, expecting to get a straight line.
factor). The parameter Ea, which is obtained from the slope The activation energy is found from the dimensionless slope which can be rearranged into For the present Topic the Arrhenius parameters are regarded
of the line (which is equal to −Ea/R), is called the activation by writing −Ea/R = slope/units, where in this case ‘units’ = as purely empirical quantities which summarize the variation
energy. Collectively the two quantities are called the k E 1 1 of rate constants with temperature. Focus 18 provides a more
1/(103 K), so Ea = −slope × R × 103 K. The intercept at 1/T = 0 ln r ,2 = a  −  (17D.2)
Arrhenius parameters (Table 17D.1). kr ,1 R  T1 T2  elaborate interpretation.

17D The Arrhenius equation 743 744 17 Chemical kinetics

(a)A first look at the energy requirements of maximum; a transition state is a conformation of the atoms in of molecules in states with energy greater than or equal to
the activated complex that, after a small further distortion, leads iminε is
reactions Ea(uncatalysed)
inevitably to products. Ea(catalysed)
Ni = (N/q )e−iεβ
To interpret Ea, consider how the molecular potential energy Potential energy
changes in the course of a chemical reaction that begins The conclusion from the preceding discussion is that ∞ ∞ imin −1
N imin −1 Reactants
with a collision between A and B molecules (Fig. 17D.3). In ∑ Ni = ∑Ni − ∑ N i = N − ∑ e−iεβ q
The activation energy is the minimum energy reactants i =imin i =0 i =0 i =0
the gas phase that step is an actual collision; in solution it
must have in order to form products. N imin −1
is best regarded as a close encounter, possibly with excess = N − ∑ (e − εβ )i
energy, which might involve the solvent too. As the reac- For example, in a reaction mixture there are numerous molec- q i =0

tion event proceeds, A and B come into contact, distort, and ular encounters each second, but only very few are sufficiently Products
begin to exchange or discard atoms. The reaction coordi- energetic to lead to reaction. The fraction of close encounters The sum in blue is a finite geometrical series of the form 1 +
Reaction coordinate
nate summarizes the collection of motions, such as changes between reactants with energy in excess of Ea is given by the r + r2 + … , with r = e − εβ . The sum of n − 1 terms of such a series
in interatomic distances and bond angles, that are directly Boltzmann distribution (Prologue and Topic 13A) as e − E /RT . a is (1− r n )/(1− r ). The term in blue can therefore be written Figure 17D.5 A catalyst provides a different path with a lower
activation energy. The result is an increase in the rate of the
involved in the formation of products from reactants. The This interpretation is confirmed by comparing this expression imin −1
1− e − iminεβ reaction (in both directions).
reaction coordinate is essentially a geometrical concept with the Arrhenius equation written in the form ∑ (e − εβ i
)=
1− e − εβ
and quite distinct from the extent of reaction. The potential i =0
Arrhenius equation
energy rises to a maximum and the cluster of atoms that kr = Ae − E /RT
a
[alternative form] (17D.4) 1/(1 − e−εβ) = q
corresponds to the region close to the maximum is called the (b)The effect of a catalyst on the
activated complex. which is obtained by taking antilogarithms of both sides of = q (1− e − iminεβ )
activation energy
After the maximum, the potential energy falls as the atoms eqn 17D.1. Insight into this expression can be obtained by con-
Therefore, the fraction of molecules in states with energy of at
in the cluster rearrange and eventually it reaches a value sidering the role of the Boltzmann distribution for a simple least εmin = iminε is
The Arrhenius equation predicts that the rate constant of a re-
characteristic of the products. The climax of the reaction is model system. action can be increased by increasing the temperature or by
at the peak of the potential energy curve, which corresponds 1 ∞ 1 N  decreasing the activation energy. Changing the temperature
N i∑
to the activation energy Ea. Here two reactant molecules have N i =  N − × q (1− e − iminεβ ) of a reaction mixture is easy to do. Reducing the activation
N q 
How is that done? 17D.1 Interpreting the exponential =imin
come to such a degree of closeness and distortion that a small energy is more challenging, but is possible if a reaction takes
factor in the Arrhenius equation = 1− (1− e − iminεβ ) = e − iminεβ = e − ε min /kT
further distortion will send them in the direction of products. place in the presence of a suitable catalyst, a substance that
This crucial configuration is called the transition state of the Suppose the energy levels available to the system form a uni- which has the form of eqn 17D.4. accelerates a reaction but undergoes no net chemical change.
reaction. Although some molecules entering the transition form array of separation ε such that the energy levels are i ε , The catalyst lowers the activation energy of the reaction by
state might revert to reactants, if they pass through this con- with i = 0, 1, 2, … (Fig. 17D.4). providing an alternative path (Fig. 17D.5).
figuration then it is inevitable that products will emerge from
Brief illustration 17D.2
the encounter. Brief illustration 17D.3
e–εmin/kT
A note on good practice The terms ‘activated complex’ and The fraction of molecules with energy at least εmin is e − ε min /kT .
By multiplying εmin and k by NA, Avogadro’s constant, and Enzymes are biological catalysts. Suppose that an enzyme
‘transition state’ are often used as synonyms; however, there is
identifying NAεmin with Ea, then the fraction f of molecular reduces both the activation energy and the frequency factor
a distinction, which is best kept in mind. An activated complex
of a reaction by a factor of ten. Letting the activation energy
Energy

εmin
is a cluster of atoms that corresponds to the region close to the collisions that occur with at least a molar kinetic energy Ea
becomes f = e − Ea /RT . With Ea = 50 kJ mol−1 = 5.0 × 104 J mol−1 change from 80 kJ mol−1 to 8 kJ mol−1, and using eqn 17D.4, the
and T = 298 K: ratio of rate constants at 298 K is
Population
4
Jmol −1 )/(8.3145JK −1 mol −1 × 298K )

f = e − (5.0 × 10 = 1.7 ×10−9 −E / RT
ε kr,catalysed Acatalysed e a,catalysed Acatalysed − ( Ea,catalysed −Ea,uncatalysed )/RT
or about one or two in a billion. = = e
0 kr,uncatalysed Auncatalysed e − Ea,uncatalysed /RT Auncatalysed
Population
1 − (8 × 103 Jmol−1 −80 × 103 Jmol−1 )/(8.3145JK −1 mol−1 ) × (298K )
Potential energy

Figure 17D.4 Equally spaced energy levels of a model system. If the activation energy is zero, each collision leads to = ×e
Ea 10
Reactants As shown in the text, the fraction of molecules with energy of reaction and, according to the Arrhenius equation, the rate = 4.2 ×1011
at least εmin is e − εmin / kT . constant is equal to the frequency factor A. This factor can
therefore be identified as the rate constant in the limit that The calculation shows that a decrease in the activation energy
Products The Boltzmann distribution for this system is each collision is successful. The exponential factor, e − E /RT , a
by an order of magnitude has a much greater impact on the
gives the fraction of collisions that are sufficiently energetic rate constant than a decrease by the same order of magnitude
N i e − iεβ − εβ − iεβ
=
N q = (1− e )e to be successful, so the rate constant is reduced from A to in the frequency factor.
Reaction coordinate Ae − E /RT . a

Figure 17D.3 A potential energy profile for an exothermic where Ni is the number of molecules in state i, N is the total
reaction. The height of the barrier between the reactants and number of molecules, β = 1/kT, and the partition function
products is the activation energy of the forward reaction. q comes from the result in eqn 13B.2a. The total number
17D The Arrhenius equation 745

Checklist of concepts TOPIC 17E Reaction mechanisms


☐ 1. The activation energy, the parameter Ea in the Arrhenius ☐ 3. The frequency factor is the rate constant in the limit
equation, is the minimum energy that a molecular that all encounters, irrespective of their energy, lead to
encounter needs in order to result in reaction. reaction.
☐ 2. The higher the activation energy, the more sensitive the ☐ 4. A catalyst lowers the activation energy of a reaction.
rate constant is to the temperature. tion of cyclopropane to propene. In a bimolecular reaction,
➤ Why do you need to know this material? a pair of molecules collide and exchange energy, atoms, or
groups of atoms, or undergo some other kind of change. It is
The ability to construct the rate law for a reaction that
important to distinguish molecularity from order:
takes place by a sequence of steps provides insight into
chemical reactions at the molecular level and also sug-
Checklist of equations gests how the yield of desired products can be optimized.
• reaction order is an empirical quantity, and obtained
from the experimentally determined rate law;
Property or process Equation Comment Equation number ➤ What is the key idea? • molecularity refers to an elementary reaction proposed as
Arrhenius equation ln kr = ln A − Ea/RT 17D.1 Many chemical reactions occur as a sequence of simpler an individual step in a mechanism.
Activation energy Ea = RT 2 (d ln kr /dT ) General definition 17D.3 steps, with corresponding rate laws that can be com-
The rate law of a unimolecular elementary reaction is first-
Arrhenius equation kr = Ae − Ea /RT Alternative form 17D.4 bined into an overall rate law by applying a variety of
order in the reactant:
approximations.
d[A] Unimolecular
➤ What do you need to know already? A→P = − kr [A] (17E.1)
dt elementary reaction
You need to be familiar with the concept of rate laws (Topic
where P denotes products (several different species may
17A), and how to integrate them (Topics 17B and 17C). You
be formed). A unimolecular reaction is first order because
also need to be familiar with the Arrhenius equation for
the number of A molecules that decay in a short interval is
the effect of temperature on the rate constant (Topic 17D).
proportional to the number available to decay. For instance,
ten times as many decay in the same interval when there are
initially 1000 A molecules as when there are only 100 present.
The study of reaction rates leads to an understanding of the Therefore, the rate of decomposition of A is proportional to its
mechanism of a reaction, its analysis into a sequence of ele- concentration at any moment during the reaction.
mentary steps. Simple elementary steps have simple rate laws, An elementary bimolecular reaction has a second-order
which can be combined into an overall rate law by invoking rate law:
one or more approximations.
d[A] Bimolecular
A + B→ P = − kr [A][B] (17E.2)
dt elementary reaction

A bimolecular reaction is second order because its rate is


17E.1 Elementary reactions proportional to the rate at which the reactant species meet,
which in turn is proportional to both their concentrations.
Many reactions occur in a sequence of steps called elementary Therefore, if there is evidence that a reaction is a single-step,
reactions, each of which involves only a small number of mol- bimolecular process, the rate law can simply be written down
ecules or ions. A typical elementary reaction is as in eqn 17E.2 (and then tested against experimental data).

H + Br2 → HBr + Br
Brief illustration 17E.1
Note that the phase of the species is not specified in the chemi-
cal equation for an elementary reaction and the equation rep- Bimolecular elementary reactions are believed to account for
many homogeneous reactions, such as the dimerizations of
resents the specific process occurring to individual molecules.
alkenes and dienes and reactions such as
This equation, for instance, signifies that an H atom attacks a
Br2 molecule to produce an HBr molecule and a Br atom. CH3I(alc) + CH3CH2O−(alc) → CH3OCH2CH3(alc) + I−(alc)
The molecularity of an elementary reaction is the number of
(where ‘alc’ signifies alcohol solution). There is evidence that
molecules coming together to react in an elementary reaction.
the mechanism of this reaction is a single elementary step:
In a unimolecular reaction, a single molecule shakes itself
apart or its atoms into a new arrangement, as in the isomeriza- CH3I + CH3CH2O− → CH3OCH2CH3 + I−

17E Reaction mechanisms 747 748 17 Chemical kinetics

This mechanism is consistent with the observed rate law The product P is formed by the unimolecular decay of I: Example 17E.1 Analysing consecutive reactions Products
Reactants
v = kr[CH3I][CH3CH2O−] d[P]
= kb[I] (17E.3c) Suppose that in an industrial batch process a substance A
dt
produces the desired compound I which goes on to decay to a Concentrationi, [J]
The following sections describe how a series of simple steps If it is assumed that initially the molar concentration of A is worthless product P, each step of the reaction being first order.
can be combined into a mechanism and how the correspond- [A]0, the first-order rate law of eqn 17E.3a can be integrated (as At what time will I be present in greatest concentration?
ing overall rate law can be derived. For the present it is impor- in Topic 17B) to give Collect your thoughts The time dependence of the concentra-
tant to note that, if the reaction is an elementary bimolecular tion of I is given by eqn 17E.4b. Find the time at which [I]
process, then it has second-order kinetics, but if the kinetics is [A] = [A]0 e − k t a (17E.4a) passes through a maximum, tmax, by calculating the derivative
Intermediates
second order, then the reaction might be complex. The postu- d[I]/dt and then setting it equal to zero.
lated mechanism can be explored only by detailed detective When this equation is substituted into eqn 17E.3b, the result TIme, t
work on the system and by investigating whether side prod- is, after rearrangement, The solution It follows from eqn 17E.4b that
ucts or intermediates appear during the course of the reaction. Figure 17E.2 The basis of the steady-state approximation. It is
d[I] d[I] k (k e − kat − kbe − kbt )[A]0
Detailed analysis of this kind was one of the ways, for exam- + kb[I] = ka[A]0 e − k t =− a a supposed that the concentrations of intermediates remain small
dt
a
dt kb − ka
and hardly change during most of the course of the reaction.
ple, in which the reaction H2(g) + I2(g) → 2 HI(g) was shown
to proceed by a complex mechanism. For many years the reac- This differential equation has a standard form in the sense that This derivative is equal to zero when t = tmax and ka e − katmax =
tion had been accepted on good but insufficiently meticulous it has been studied and its solution listed. With the initial con- kbe − kbtmax . Therefore, taking natural logarithms of both sides
evidence as a fine example of a simple bimolecular reaction, dition [I]0 = 0, because no intermediate is present initially, the gives constant concentration. More specifically, after an initial in-
H2 + I2 → HI + HI, in which atoms exchanged partners during solution of the differential equation (provided ka ≠ k b) is ln ka − kat max = ln kb − kbt max duction period, an interval during which the concentrations
a collision. of intermediates rise from zero, the rates of change of the con-
ka which rearranges to centrations of all reaction intermediates are negligibly small
[I] = (e − k t − e − k t )[A]0
a b
(17E.4b)
kb − ka during the major part of the reaction (Fig. 17E.2):
ln (k /k )
Consecutive elementary !#"a# b
17E.2 $ d[I]
At all times [A] + [I] + [P] = [A]0, so it follows that ln ka − ln kb = kat max − kbt max = (ka − kb )t max ≈0 Steady-state approximation (17E.5)
reactions dt
 k e − k t − kbe − k t  b a
It then follows that This approximation greatly simplifies the discussion of reac-
Some reactions proceed through the formation of an interme- [P] = 1+ a [A]0 (17E.4c)
tion schemes. For example, to apply the approximation to the
 kb − ka  1 k
diate (denoted I), as in the consecutive unimolecular reactions t max = ln a consecutive first-order mechanism, d[I]/dt is set equal to 0 in
ka − kb kb
The concentration of the intermediate I rises to a maximum eqn 17E.3b, which then becomes ka[A]− kb[I]= 0. It then follows
ka
A → I→ P
kb and then falls to zero (Fig. 17E.1). The concentration of the Comment. For a given value of ka, as k b increases both the that
product P rises from zero towards [A]0, when all A has been time at which [I] is a maximum and the yield of I decrease.
ka
Note that the intermediate occurs in the reaction steps but converted to P. [I] = [A] (17E.6)
Self-test 17E.1 Calculate the maximum concentration of I and kb
does not appear in the overall reaction, which in this case is justify the last remark.
A → P. Any reverse reactions are ignored here, so the reaction Answer: [I]max/[A]0 = (k a/k b)c, c = k b/(k b − k a) The steady-state approximation requires the concentration
proceeds from all A to all P, not to an equilibrium mixture of of the intermediate to be low relative to that of the reactants,
the two. An example of this type of mechanism is the decay of which is the case when ka << k b. Equation 17E.6 implies that
1
a radioactive family, such as the concentration of the intermediate changes as the concen-
tration of A changes, but if ka/k b << 1 it changes very little.
23.5 min 239 2.35 days 239 0.8
The steady-state approximation
Concentration, [J]/[A]0

239
U →
 Np →
 Pu P 17E.3 Both requirements of the steady-state approximation, the low
concentration of intermediate and its slow change, are there-
0.6
(The times are half-lives.) The characteristics of this type of One feature of the calculation so far has probably not gone fore satisfied.
reaction are discovered by setting up the rate laws for the net unnoticed: there is a considerable increase in mathematical On substituting the value of [I] in eqn 17E.6 into eqn 17E.3c,
0.4
rate of change of the concentration of each substance and then I complexity as soon as the reaction mechanism has more than that equation becomes
combining them in the appropriate manner. a couple of steps or reverse reactions are taken into account. A
0.2
The rate of unimolecular decomposition of A is reaction scheme involving many steps is nearly always unsolv- [I] = (ka/kb)[A]
A able analytically, and alternative methods of solution are nec-
d[A] 0 d[P]
= − ka[A] (17E.3a) 0 5 10 15 20 25 30 essary. One approach is to integrate the rate laws numerically. = kb[I] ≈ ka[A] (17E.7)
dt dt
Time, kat An alternative approach, which continues to be widely used
The intermediate I is formed from A (at a rate ka[A]) but decays Figure 17E.1 The concentrations of A, I, and P in the consecutive because it leads to convenient expressions and more readily di- It follows that the rate of formation of P is the same as the rate
to P (at a rate k b[I]). The net rate of formation of I is therefore reaction scheme A → I → P. The curves are plots of eqns 17E.4a–c gestible results, is to make an approximation. of loss of A (as given by eqn 17E.3a), and that both processes
with ka = 10kb. If the intermediate I is in fact the desired product, The steady-state approximation (which is also widely called are governed by the rate constant ka. In effect, the reactant
d[I] it is important to be able to predict when its concentration is the quasi-steady-state approximation to distinguish it from a A flows directly through to becoming P without any I accu-
= ka[A] − kb[I] (17E.3b)
dt greatest; see Example 17E.1. true steady state) assumes that the intermediate, I, is in a low, mulating on the way. The solution of eqn 17E.7 is found by
17E Reaction mechanisms 749 750 17 Chemical kinetics

1 net rates to zero. You can then solve the resulting equations Small kr Large kr The rate law of a reaction that has a rate-determining
A P algebraically to obtain expressions for the concentrations of step can often—but certainly not always—be written down
(a)
0.8 the intermediates. Finally, use these solutions to obtain an almost by inspection. If the first step in a mechanism is rate-
Concentration, [J]/[A]0

expression for the overall rate of consumption of N2O5. determining, then the rate of the overall reaction is equal to

Reactants
Large kr Small kr

Products
0.6 the rate of that step because the rate constants of the subse-
The solution The intermediates are NO and NO3; the net rates
of change of their concentrations are (b) quent steps are such that the intermediates immediately
0.4 flow through these steps to give products. Moreover, be-
d[NO] Large kr Small kr cause the rate-determining step is the one with the small-
= kb[NO2 ][NO3 ] − kc [NO][N 2O5 ] ≈ 0
0.2 dt est rate constant, then it follows that the rate-determining
I (c) Large kr step is the one with the highest activation energy. Once over
d[NO3 ]
0 = ka [N 2O5 ] − ka′[NO2 ][NO3 ] − kb[NO2 ][NO3 ] ≈ 0 the initial barrier, the intermediates cascade into products
0 1 2 3 dt Figure 17E.4 In these diagrams of reaction schemes, heavy
Time, kat (Fig. 17E.5).
arrows represent steps with large rate constants and light arrows
The solutions of these two simultaneous equations (in blue)
Figure 17E.3 A comparison of the exact result for the represent steps with small rate constants. (a) The first step
are
concentrations of a consecutive reaction and the concentrations is rate-determining; (b) the second step is rate-determining; Brief illustration 17E.2
obtained by using the steady-state approximation (dotted lines) Use expression for [NO3] (c) although one step has a small rate constant, it is not rate-
for kb = 20ka. (The curve for [A] is unchanged.) determining because there is a route with a large rate constant The oxidation of NO to NO2, 2 NO(g) + O2(g) → 2 NO2(g),
ka [N 2O5 ] k [NO2 ][NO3 ] ka kb that circumvents it. proceeds by the following mechanism:
[NO3 ] = [NO] = b =
(ka′ + kb )[NO2 ] kc [N 2O5 ] (ka′ + kb )kc
NO + NO → N2O2 ka
substituting the solution for [A], eqn 17E.4a, and integrating
The net rate of change of concentration of N2O5 is then A note on good practice It is commonly said that ‘the first step is
the resulting expression: N2O2 → NO + NO ka′
slow and the second is fast, so the first step is rate-determining’.
d[N 2O5 ] Such a statement is incorrect: the two rates are equal; it is the rate N2O2 + O2 → NO2 + NO2 kb
[A] = [A]0e−kat = − ka [N 2O5 ] + ka′[NO2 ][NO3 ] − kc [NO][N 2O5 ] constants that are different.
lntegral E.1 dt
!#"# $ with rate law (see Problem P17E.6)
[P] t t
ka ka′[N 2O5 ] ka kb In general, the rate-determining step (RDS) is the step in a
∫ 0
d[P] = ∫ ka[A]dt = ka[A]0 ∫ e − k t dt
0 0
a
= − ka [N 2O5 ] +
ka′ + kb
− [N O ]
ka′ + kb 2 5 mechanism that controls the overall rate of the reaction (in the d[NO2 ] 2ka kb[NO]2[O2 ]
=
dt ka′ + kb[O2 ]
Hence present example, the first step governed by ka, with ka << k b).
2k k [N O ]
=− a b 2 5 The rate-determining step must be a crucial gateway for the
ka′ + kb When the concentration of O2 in the reaction mixture is so
[P] = [A]0 (1− e − k t ) a (17E.8) formation of products, and not just a reaction with a small rate high that ka′ << kb[O2 ], the rate law then simplifies to
That is, N2O5 decays with a first-order rate law with a rate con- constant. If another reaction with a larger rate constant can
This result is the same as in eqn 17E.4c when ka << k b; however, stant that depends on ka, ka′, and k b but not on kc. also lead to products, then the step with the small rate con- d[NO2 ]
= 2ka [NO]2
the use of the steady-state approximation is much simpler. stant is irrelevant because it can be sidestepped (Fig. 17E.4). In dt
Figure 17E.3 compares the approximate solutions found here Self-test 17E.2 Derive the rate law for the decomposition of some cases, when a first-order reaction is in competition with
ozone in the reaction 2 O3(g) → 3 O2(g) on the basis of the which shows that the formation of N2O2 in the first step is
with the exact solutions found earlier: ka does not have to be a second-order reaction, the criterion has to be expressed in
(incomplete) mechanism rate-determining. The rate law could also have been written
very much smaller than k b for the approach to be reasonably terms of the relative sizes of the first-order (for one step) and a
by inspection of the mechanism, because the rate law for the
accurate. O3 → O2 + O ka
pseudofirst-order (for the second step) rate constants, for only
overall reaction is simply the rate law of that rate-determining
then can their magnitudes be compared. This point is illus-
O2 + O → O3 ka′ step.
trated in the next Brief illustration.
Example 17E.2 Using the steady-state approximation O + O3 → O2 + O2 kb

Devise the rate law for the decomposition of N2O5, 2 N2O5(g) Answer: d[O3]/dt = −2k a k b[O3]2/(k′[O
a 2] + k b[O3])

→ 4 NO2(g) + O2(g) on the basis of the following mechanism:


17E.5 Pre-equilibria
N2O5 → NO2 + NO3 ka
NO2 + NO3 → N2O5 k′a Now consider a slightly more complicated mechanism in

Potential energy
NO2 + NO3 → NO2 + O2 + NO kb 17E.4 The rate-determining step which an intermediate I reaches an equilibrium with the re-
NO + N2O5 → NO2 + NO2 + NO2 kc actants A and B:
When the steady-state approximation is valid, which is when
Small Large Large k
A note on good practice Note that when writing the equation for ka << k b in the reaction A → I → P, the decrease in the concen- !!
A+B #!"
a k
! I  →P b
Pre-equilibrium (17E.9)
rate rate rate k′ a

an elementary reaction all the species are displayed individually; tration of A is matched by an increase in the concentration of constant constant constant
so write A → B + B, for instance, not A → 2 B. P. It is important to realize that the rates of the steps A → I and This scheme involves a pre-equilibrium, in which an interme-
I → P are the same: the concentration of I is so low compared RDS diate is in equilibrium with the reactants. A pre-equilibrium
Progress of reaction
Collect your thoughts First identify the intermediates and for to the concentration of A that even though ka << k b (and there- can arise when the rate of decay of the intermediate back
each of them write an expression for the net rate of forma- fore k b >> ka) the rate of the second step, k b[I], matches that of Figure 17E.5 The reaction profile for a mechanism in which the into reactants is much faster than the rate at which it forms
tion. Then apply the steady-state approximation and set these the first, ka[A]. first step (RDS) is rate-determining. products; this condition is satisfied if ka′ >> k b. Because A, B,

17E Reaction mechanisms 751 752 17 Chemical kinetics

and I are assumed to be in equilibrium, it follows that (see Self-test 17E.3 Show that the pre-equilibrium mechanism in equilibrium) is given by the ratio of the two rates, and there-
Topic 17C) which 2 A ! I (K) followed by I + B → P (k b) results in an Kinetic and thermodynamic
17E.6 fore by the ratio of the two rate constants:
overall third-order reaction. control of reactions [P2 ] kr ,2
K = kac ⦵/ka′ Answer: d[P]/dt = (k bK/c −○ )[A]2[B]
= Kinetic control (17E.14)
[P1 ] kr ,1
[I]c −

K k In some cases reactants can give rise to a variety of products,
K= and [I] = − [A][B] = a [A][B] (17E.10) as in nitrations of mono-substituted benzene, when various This ratio represents the kinetic control over the proportions
[A][B] c ka′ ○

One feature to note is that although each of the rate con- proportions of the ortho-, meta-, and para-substituted prod- of products, control that stems from relative rates rather than
In writing these equations, the rate of reaction of I to form P is stants in eqn 17E.12 increases with temperature, this might ucts are obtained, depending on the directing power of the thermodynamic considerations about equilibrium. Kinetic
presumed to be too slow to affect the maintenance of the pre- not be true of kr itself. Thus, if the rate constant ka′ increases original substituent. Suppose two products, P1 and P2, are pro- control is a common feature of the reactions encountered
equilibrium (see the following Example). The rate of formation more rapidly than the product kak b increases, then kr = kak b/ka′ duced by the following competing reactions: in organic chemistry, where reactants are chosen that facili-
of P may now be written decreases with increasing temperature and the reaction goes tate pathways favouring the formation of a desired product.
more slowly as the temperature is raised. Mathematically, the A + B → P1 v(P1) = kr,1[A][B] If a reaction is allowed to reach equilibrium, then the
[I] = (ka/ka′)[A][B] overall reaction is said to have a ‘negative activation energy’. proportion of products is determined by thermodynamic
A + B → P2 v(P2) = kr,2[A][B]
d[P] k For example, suppose that each rate constant in eqn 17E.12 rather than kinetic considerations, and the ratio of concentra-
= kb[I] = kb a [A][B] (17E.11) has an Arrhenius temperature dependence (Topic 17D). It fol- The relative proportion in which the two products have been tions is controlled by considerations of the standard reaction
dt ka′
lows from the Arrhenius equation (eqn 17D.4, kr = Ae − E /RT ) a
produced at a given stage of the reaction (before it has reached Gibbs energy.
This rate law has the form of a second-order rate law with a that
composite rate constant:
ex+y = exey
d[P] kk ex−y = ex/ey
= kr [A][B] with kr = b a (17E.12)
dt ka′
( A e − E /RT )( Abe − E
a,a a,b / RT
) AA Checklist of concepts
In this pre-equilibrium mechanism the final step, I → P, is kr = a = a b e−( E a,a + Ea,b − Ea,a ′ )/ RT

Aa′e − E /RT a,a ′ Aa′


rate-determining. The preceding steps control the steady con- ☐ 1. The mechanism of reaction is the sequence of elemen- ☐ 5. In the steady-state approximation, it is assumed that
centration of the intermediate. The effective activation energy of the reaction is therefore tary steps that leads from reactants to products. the concentrations of all reaction intermediates remain
☐ 2. The molecularity of an elementary reaction is the num- constant and small throughout the reaction.
Example 17E.3 Ea = Ea,a + Ea,b − Ea,a′ (17E. 13) ber of molecules coming together to react. ☐ 6. Pre-equilibrium is a state in which an intermediate
Analysing a pre-equilibrium using the
☐ 3. An elementary unimolecular reaction has first-order is in equilibrium with the reactants and which arises
steady-state assumption
This activation energy is positive if Ea,a + Ea,b > Ea,a′ (Fig. 17E.6a) kinetics; an elementary bimolecular reaction has sec- when the rate of decay of the intermediate back to reac-
Analyse the scheme shown in eqn 17E.9 using the steady-state but negative if Ea,a′ > Ea,a + Ea,b (Fig. 17E.6b). An important con- ond-order kinetics. tants is much faster than the rate at which products are
approximation. sequence of this discussion is that it is necessary to be cautious formed from the intermediate.
☐ 4. The rate-determining step is the step in a reac-
Collect your thoughts Begin by writing the net rates of change
when making predictions about the effect of temperature on ☐ 7. Kinetic control over the proportions of products stems
tion mechanism that controls the rate of the overall
of the concentrations of P and I, and then invoke the steady- reactions that are the outcome of several steps. from relative rates rather than thermodynamic consid-
reaction.
state approximation for the intermediate I. Use the resulting erations about equilibrium.
expression to obtain the rate of change of the concentration
of P.
The solution The net rates of change of P and I are
d[P]
= kb[I] Steady-state approximation
Checklist of equations
dt Ea,a Ea,a’
Potential energy

d[I] Property Equation Comment Equation number


= ka [A][B] − ka′[I] − kb[I] ≈ 0 (a)
dt Unimolecular reaction d[A]/dt = − kr [A] A→P 17E.1
The second equation implies that (b) Bimolecular reaction d[A]/dt = − kr [A][B] A+B→P 17E.2
ka kb
ka [A][B] Ea,b Consecutive reactions [A] = [A]0 e − kat A → I→ P 17E.4
[I] ≈ [I] = (ka /(kb − ka ))(e − kat − e − kbt )[A]0
ka′ + kb
[P] = {1 + (ka e − kbt − kbe − kat )/(kb − ka )}[A]0
Now substitute this result into the expression for the rate of
Reaction coordinate Steady-state approximation d[I]/dt ≈ 0 I is an intermediate 17E.5
formation of P:
d[P] k [A][B] kk Figure 17E.6 For a reaction with a pre-equilibrium, there are
= kb[I] ≈ kb a = kr [A][B] with kr = a b
dt ka′ + kb ka′ + kb three activation energies to take into account: two referring
to the reversible steps of the pre-equilibrium and one for the
This expression reduces to that in eqn 17E.12 when the rate final step. The relative magnitudes of the activation energies
constant for the decay of I into products is much smaller than determine whether the overall activation energy is (a) positive or
that for its decay into reactants, kb << ka′ . (b) negative.
754 17 Chemical kinetics

TOPIC 17F Examples of reaction the steady-state approximation to the net rate of formation
of A*:
further to take into account experimental results over a range
of concentrations and pressures.
d[A*]
mechanisms dt
= ka[A]2 − ka′[A][A*] − kb[A*] ≈ 0

Rearrangement of this equation gives


(17F.2) Example 17F.1Analysing data in terms of the
Lindemann–Hinshelwood mechanism
At 300 K the effective rate constant for a gaseous reaction
ka[A]2
[A*] = (17F.3) A → P, which has a Lindemann–Hinshelwood mechanism,
kb + ka′[A]
is kr,1 = 2.50 × 10−4 s −1 at [A]1 = 5.21 × 10−4 mol dm−3 and kr,2 =
so the rate law for the formation of P is 2.10 × 10−5 s −1 at [A]2 = 4.81 × 10−6 mol dm−3. Calculate the rate
constant ka for the activation step in the mechanism.
A d[P] k k [A]2
➤ Why do you need to know this material? = kb[A*] = a b (17F.4) Collect your thoughts Use eqn 17F.8 to write an expression
dt kb + ka′[A]
Some important reactions have complex mechanisms and A for the difference 1/kr,2 − 1/kr,1, rearrange the expression for ka,
need special treatment, so you need to see how to make At this stage the rate law is not first-order. However, if the rate and then insert the data.
and implement assumptions about the relative rates of the of deactivation by (A*,A) collisions is much greater than the
rate of unimolecular decay, in the sense that ka′[A][A*]>> kb[A*], The solution It follows from eqn 17F.8 that
steps in a mechanism. A*
or (after cancelling the [A*]), ka′[A]>> kb , then k b can be 1 1 1 1 1 
− = −
➤ What is the key idea?
Products
neglected in the denominator of eqn 17F.4 to obtain kr ,2 kr ,1 ka  [A]2 [A]1 
A
The steady-state approximation can often be used to
d[P] kk and so
derive rate laws for proposed mechanisms. Figure 17F.1 A representation of the Lindemann–Hinshelwood = kr [A] with kr = a b
dt ka′ 1/[A]2 −1/[A]1
mechanism of unimolecular reactions. The species A is excited by ka =
➤ What do you need to know already? collision with A, and the energized A molecule (A*) may either Lindemann–Hinshelwood rate law (17F.5) 1/ kr ,2 −1/ kr ,1
You need to be familiar with the concept of rate laws be deactivated by a collision with A or go on to decay by a
unimolecular process to form products. Equation 17F.5 is a first-order rate law, as required. 1/(4.81×10−6 moldm −3 ) −1/(5.21×10−4 moldm −3 )
(Topic 17A) and the formulation of an overall rate law from =
The Lindemann–Hinshelwood mechanism can be tested 1/(2.10 ×10−5 s −1 ) −1/(2.50 ×10−4 s −1 )
a mechanism by using the steady-state approximation
(Topic 17E). because it predicts that, as the concentration (and therefore the = 4.72 dm3 mol −1 s −1
partial pressure) of A is reduced, the reaction should switch to
tion, however, because the overall mechanism has bimolecular overall second-order kinetics. Thus, when ka′[A][A*]<< kb[A*] Self-test 17F.1 The effective rate constants for a gaseous
as well as unimolecular steps. or (after cancelling the [A*]) ka′[A]<< kb , the rate law in eqn reaction A → P, which has a Lindemann–Hinshelwood
Many reactions take place by mechanisms that involve sev- The first successful explanation of unimolecular reactions 17F.4 becomes mechanism, are 1.70 × 10−3 s −1 and 2.20 × 10−4 s −1 at [A] = 4.37 × 10−4
eral elementary steps. In each case it is possible to approach was provided by Frederick Lindemann in 1921 and then elabo- mol dm−3 and 1.00 × 10−5 mol dm−3, respectively. Calculate the
d[P]
the setting up (and testing) of a rate law by proposing a rated by Cyril Hinshelwood. In the Lindemann–Hinshelwood = ka[A]2 (17F.6) rate constant for the activation step in the mechanism.
dt
mechanism and, when appropriate, applying the steady-state mechanism it is supposed that a reactant molecule A becomes Answer: 24.7 dm3 mol−1 s −1
approximation. energetically excited by collision with another A molecule in a The physical reason for the change of order is that as the pres-
bimolecular step (Fig. 17F.1): sure is reduced the rate of the bimolecular process in which A*
loses its excess energy becomes negligible compared to the rate
d[A*]
A + A → A* + A
dt
= ka[A]2 (17F.1a) at which A* goes on to form products. The reaction mecha- 17F.2 Polymerization kinetics
17F.1 Unimolecular reactions nism is then a sequence of two steps, with the first step (which
The energized molecule (A*) might lose its excess energy by is bimolecular) being rate limiting. If the full rate law in eqn There are two major classes of polymerization processes and
A number of gas-phase reactions follow first-order kinetics, as collision with another molecule: 17F.4 is written as in each one the average molar mass of the product varies with
in the isomerization of cyclopropane to propene: time in a distinctive way. In stepwise polymerization any two
d[A*] d[P] k k [A]
A + A* → A + A = − ka′[A][A*] (17F.1b) = kr [A] with kr = a b (17F.7) monomers present in the reaction mixture can link together at
dt dt kb + ka′[A]
cyclo-C3H6(g) → CH3CH=CH2(g) v = kr[cyclo-C3H6] any time and growth of the polymer is not confined to chains
Alternatively, the excited molecule might shake itself apart or then the expression for the effective rate constant, kr, can be that are already forming (Fig. 17F.2). As a result, monomers
The problem with the interpretation of first-order rate laws is into a different arrangement of its atoms and form products P. rearranged (by inverting each side) to are consumed early in the reaction and, as will be seen, the
that presumably a molecule acquires enough energy to react as That is, it might undergo the unimolecular decay average molar mass of the product grows linearly with time.
a result of collisions with other molecules. However, collisions 1 k′ 1 Effective rate constant In chain polymerization a monomer, M, attacks another
d[A*] = a + [Lindemann–Hinshelwood (17F.8)
are simple bimolecular events, so how can they result in a A* → P = − kb[A*] (17F.1c) kr ka kb ka[A] mechanism] monomer, links to it, then that unit attacks another monomer,
dt
first-order rate law? First-order gas-phase reactions are widely and so on. The monomer is used up as it becomes linked to
called ‘unimolecular reactions’ because they also involve an If the unimolecular step is the rate-determining step, the Hence, a test of the theory is to plot 1/kr against 1/[A] and ex- the growing chains (Fig. 17F.3). Polymers built from numer-
elementary unimolecular step in which the reactant molecule overall reaction will have first-order kinetics, as observed. pect a straight line. This behaviour is observed often at low ous monomers are formed rapidly and only the yield, not the
changes into the product. This term must be used with cau- This conclusion can be demonstrated explicitly by applying concentrations but deviations are common at high concentra- average molar mass, of the polymer is increased by allowing
tions. Topic 18A develops the description of the mechanism long reaction times.

17F Examples of reaction mechanisms 755 756 17 Chemical kinetics

Polyesters and polyurethanes are formed similarly (the 25 Chain polymerization occurs by addition of monomers to
latter without elimination). A polyester, for example, can be a growing polymer, often by a radical chain process. It results
Average chain length, 〈N〉

regarded as the outcome of the stepwise condensation of a 20 in the rapid growth of an individual polymer chain for each
hydroxyacid HO−R−COOH. Consider the formation of a monomer available to react. Examples include the addition
polyester from such a monomer. Its progress can be measured 15 polymerizations of ethene, methyl methacrylate, and styrene,
in terms of the concentration of the −COOH groups in the as in
sample (denoted A), because these groups gradually disappear 10
−CH2ĊHX + CH2=CHX → −CH2CHXCH2ĊHX
as the condensation proceeds. Because the condensation reac-
tion can occur between molecules containing any number of 5 and subsequent reactions.
monomer units, chains of many different lengths can grow in There are several characteristic steps in a chain reaction.
the reaction mixture. 0 Initiation, the formation of active radicals, may be written as
0 0.2 0.4 0.6 0.8 1
In the absence of a catalyst, condensation is expected to Fraction condensed, p
be overall second-order in the concentration of the −OH and In → R· + R·
Figure 17F.4 The average chain length of a polymer as a function
−COOH (or A) groups, so M + R· → ·M1
of the fraction of reacted monomers, p. Note that p must be very
close to 1 for the chains to be long.
d[A] where In is the initiator, R· the radical that In forms, and ·M1
= − kr [OH][A] (17F.9a)
Figure 17F.2 In stepwise polymerization, growth can start at any dt is a monomer radical. In this reaction a radical is produced,
pair of monomers (in green), and so new chains (in red) begin to but in some polymerizations the initiation step leads to the
form throughout the reaction. However, because there is one −OH group for each −COOH This result is illustrated in Fig. 17F.4. When p is expressed formation of an ionic chain carrier. Initiation is followed by
group, this equation is the same as in terms of the rate constant kr (the second part of eqn 17F.11), propagation, the continuation of the chain reaction:
the result is
M + ·M1 → ·M2
d[A]
= − kr [A]2 (17F.9b) Degree of polymerization in M + ·M2 → ·M3
dt 〈 N 〉 = 1+ krt[A]0 terms of the rate constant (17F.12b)
[stepwise polymerization] !
If the rate constant for the condensation is independent of M + ·Mn−1 → ·Mn
the chain length, kr remains constant throughout the reac- The average length grows linearly with time. Therefore, the
tion. The solution of this rate law is then given by eqn 17B.4b, longer a stepwise polymerization proceeds, the higher the where Mn is a polymer consisting of n monomer units.
and is average molar mass of the product. Polymerization may terminate in a number of ways. For
example,
[A]0
[A] = (17F.10)
1+ krt[A]0 Brief illustration 17F.1 mutual termination: ·Mn + ·Mm → Mn+m
disproportionation: ·HMn + ·Mm → Mn + HMm
The fraction, p, of −COOH groups that have condensed at time Consider a polymer formed by a stepwise process with
t is therefore kr = 1.00 dm3 mol−1 s −1 and an initial monomer concentration chain transfer: M + ·Mn → ·M + Mn
of [A]0 = 4.00 × 10−3 mol dm−3. From eqn 17F.12b, the degree of
[A] = [A]0 /(1 + krt[A]0)
polymerization at t = 1.5 × 104 s is In mutual termination two growing radical chains combine.
〈N〉 = 1 + (1.00 dm3 mol−1 s −1) × (1.5 × 104 s) In termination by disproportionation a hydrogen atom trans-
Figure 17F.3 The process of chain polymerization. Chains (in red) [A]0 −[A] k t[A]0 Fraction of condensed groups
p= = r (17F.11) × (4.00 × 10−3 mol dm−3) = 61 fers from one chain to another, corresponding to the oxidation
grow as each chain acquires additional monomers (in green). [A]0 1+ krt[A]0 [stepwise polymerization] of the donor and the reduction of the acceptor. In chain trans-
From eqn 17F.12a, the fraction condensed, p, is fer, a new chain initiates at the expense of the one currently
The degree of polymerization, the average number of mon- 〈 N 〉−1 61−1 growing. As can be suspected, the mechanism is complicated,
(a) Stepwise polymerization omer residues per polymer molecule, can now be calculated. p= = = 0.98 but can be explored by using the steady-state approximation.
〈N 〉 61
Stepwise polymerization commonly proceeds by a conden- This quantity is the ratio of the initial concentration of A, [A]0,
sation reaction, in which a small molecule (typically H2O) is to the concentration of end groups, [A], at the time of interest, How is that done? 17F.1 Deriving an expression for the
eliminated in each step. Stepwise polymerization is the mech- because there is one A group per polymer molecule. For ex-
(b) Chain polymerization rate of chain polymerization
anism of production of polyamides, as in the formation of ample, if there were initially 1000 A groups and there are now
nylon-66: only 10, the average length of each polymer must be 100 units. Many gas-phase reactions and liquid-phase polymerization The kinetic analysis of chain polymerization must take into
Because [A] can be expressed in terms of p (the first part of reactions are chain reactions. In a chain reaction, a reaction account initiation, propagation, and termination.
H2N(CH2)6NH2 + HOOC(CH2)4COOH → eqn 17F.11), the average number of monomers per polymer intermediate produced in one step generates an intermediate Step 1 Write an expression for the rate of initiation of the process
H2N(CH2)6NHCO(CH2)4COOH + H2O molecule, 〈N〉, is in a subsequent step, then that intermediate generates another
If the initiation step is
intermediate, and so on. The intermediates in a chain reaction
continuing to
[A]0 1 Degree of polymerization are called chain carriers. In a radical chain reaction the chain In → R· + R· vi = ki[In]
〈N 〉 = = (17F.12a)
→ H−[HN(CH2)6NHCO(CH2)4CO]n−OH [A] 1− p [stepwise polymerization] carriers are radicals (species with unpaired electrons). M + R· → ·M1
17F Examples of reaction mechanisms 757 758 17 Chemical kinetics

If the rate constants of the chain propagation steps are large The overall rate of polymerization is therefore proportional to with kr given in eqn 17F.14c. That is, the slower the initiation S Active Active S
enough, the first of these two steps is rate-determining for the square root of the concentration of the initiator, In, and site site
of the chain (the smaller the initiator concentration and the
the overall polymerization process and the rate of initiation is given by smaller the initiation rate constant), the greater is the kinetic
is equal to vi. chain length, and therefore the higher is the average molar E E

 fk 
1/2 (17F.13) mass of the polymer.
Step 2 Write an expression for propagation v = kr [In]1/2[M] with kr = kp  i 
 kt  Rate of polymeri-
Lock
If the rate of propagation is independent of chain size for zation Induced
[chain polymeri- and fit
sufficiently large chains, then the rate of propagation, vp, may SE
zation] key
be written
17F.3 Enzyme-catalysed reactions
vp = kp[M][·M] The kinetic chain length, λ , is the ratio of the number of Figure 17F.6 Two models that explain the binding of a substrate
to the active site of an enzyme. In the lock-and-key model,
where ·M stands for a polymer of any length. It follows from monomer units consumed to the number of radicals produced A catalyst is a substance that accelerates a reaction but under-
the active site and substrate have complementary three-
the remark in Step 1 that in the initiation step: goes no net chemical change (Topic 17D): the catalyst lowers
dimensional structures and dock without the need for major
the activation energy of the reaction by providing an alter-
 d[⋅M]  number of monomer units consumed atomic rearrangements. In the induced fit model, binding of the
 dt  = 2 f ki[In] λ= native path to that of the uncatalysed reaction (Fig. 17F.5). substrate induces a conformational change in the active site.
production number of radicals produced Enzymes, which are homogeneous biological catalysts, are The substrate fits well in the active site after the conformational
where f is the fraction of radicals R· that successfully initiate Kinetic chain length very specific and can have a dramatic effect on the reactions change has taken place.
(17F.14a)
a chain. The factor 2 recognizes that two radicals are formed [definition] they control. For example, the enzyme catalase accelerates the
in each initiation step. reaction it catalyses by a factor of 1012 at 298 K.
The kinetic chain length can be imagined as the average Enzymes contain an active site, which is responsible for
Step 3 Consider the termination of the process
number of molecules in a chain produced by one initiat- binding the substrates, the reactants, and processing them fit model, in which binding of the substrate induces a confor-
For the present analysis, suppose that only mutual termina- ing radical. The kinetic chain length can be expressed in into products. As is true of any catalyst, the active site returns mational change in the active site. Only after the change does
tion occurs. If the rate of termination is assumed to be inde- terms of the rate expressions above. To do so, recognize that to its original state after the products are released. Many en- the substrate fit snugly in the active site.
pendent of the length of the chain, the rate law for termination monomers are consumed at the rate that chains propagate. zymes consist primarily of proteins, some featuring organic Experimental studies of enzyme kinetics are typically con-
is
Then, or inorganic co-factors in their active sites. However, cer- ducted by monitoring the initial rate of product formation
vt = kt[·M]2 tain RNA molecules can also be biological catalysts, forming in a solution in which the enzyme is present at very low con-
rate of propagation of chains ribozymes. centration. Indeed, enzymes are such efficient catalysts that
and the rate of change of radical concentration by this deple- λ=
rate of production of radicals The structure of the active site is specific to the reaction that significant accelerations may be observed even when their
tion process is
Kinetic chain length in terms of reaction rates (17F.14b) it catalyses, with groups in the substrate attached to groups in concentration is more than three orders of magnitude smaller
 d[⋅M]  the active site primarily by hydrogen bonding, electrostatic than that of the substrate.
 dt  = − 2kt[⋅M]2
depletion In applying the steady-state approximation, the rate of forces, and van der Waals interactions. Figure 17F.6 shows The principal features of many enzyme-catalysed reactions
In this case, the factor 2 recognizes that two radicals are production of radicals is set equal to the termination rate two models that explain the binding of a substrate to the ac- are as follows:
removed in each depletion step. (Step 4 in the discussion above). Therefore, the kinetic chain tive site of an enzyme. In the lock-and-key model, the active
length may be written as site and substrate have complementary three-dimensional • For a given initial concentration of substrate, [S]0, the
Step 4 Apply the steady-state approximation structures and dock without the need for major structural initial rate of product formation is proportional to the
The net rate of formation of ·M is k [⋅M][M] kp[M] change. Experimental evidence however favours the induced total concentration of enzyme, [E]0.
λ= p =
2kt[⋅M]2 2kt[⋅M] • For a given [E]0 and low values of [S]0, the rate of product
production depletion
d[⋅M] !"
# # $ !" # # $ formation is proportional to [S]0.
= 2fki[In] − 2kt[⋅M]2 ≈ 0 The steady-state expression for [·M], [⋅M]= (fki / kt )1/2[In]1/2 , is
dt • For a given [E]0 and high values of [S]0, the rate of product
substituted for the radical concentration, to obtain formation becomes independent of [S]0, reaching a maxi-
steady-state approximation
mum value known as the maximum velocity, vmax.
The steady-state concentration of radical chains is therefore λ = kr [M][In]−1/2 with kr = kp (4 fki kt )−1/2
Ea(uncatalysed)
Ea(catalysed) The Michaelis–Menten mechanism accounts for these fea-

Potential energy
1/2 Kinetic chain length
 fk  (17F.14c) tures. According to this mechanism, an enzyme–substrate
[⋅M]=  i  [In]1/2 [chain polymerization]
k  t Reactants complex is formed in the first step and then either the sub-
In mutual termination, the average number of monomers strate is released unchanged or, after modification, released as
In Step 2 it is established that the overall rate of polymeriza- in a polymer molecule, 〈N〉, produced by the reaction is the the product:
tion is equal to the rate of propagation, which is given by sum of the numbers of monomers in the two combining
vp = kp[M][·M]. The steady-state expression for [·M] can now polymer chains. The average number of units in each chain is ka
Products 
E +S ← 
→ ES

be inserted into this expression to give λ . Therefore, k′ a Michaelis–Menten
kb
mechanism
Reaction coordinate
1/2 ES 
→ P+E
 fk  〈 N 〉= 2λ = 2kr [M][In]−1/2 Degree of polymerization
vp = kp[⋅M][M]= kp  i  [In]1/2[M] (17F.15) Figure 17F.5 A catalyst provides a different path with a lower
 kt  [chain polymerization]
activation energy. The result is an increase in the rates of the Again, the mechanism may be analysed by using the steady-
forward (and reverse) reaction. state approximation.

17F Examples of reaction mechanisms 759 760 17 Chemical kinetics

How is that done? 17F.2 Deriving the Michaelis–Menten


1 and so KM = 10.0 mmol dm−3.

equation 40
Rate of reaction, v/vmax

1/v

The rate of product formation according to the Michaelis–


Menten mechanism is Slope = KM/vmax 30
1/(v/mmol dm–3 s–1)

v = k b[ES]
–1/KM 1/vmax 20
so the strategy is centred on finding an expression for the
concentration of the intermediate, ES.
10
Step 1 Apply the steady-state approximation 0
0 0 1/[S]0
The concentration of the enzyme–substrate complex is found Substrate concentration, [S]0
by invoking the steady-state approximation and writing 0
Figure 17F.7 The variation of the rate of an enzyme-catalysed Figure 17F.8 The structure of a Lineweaver–Burk plot for the 0 0.2 0.4 0.6 0.8 1
reaction with substrate concentration. The approach to a analysis of an enzyme-catalysed reaction that proceeds by 1/([CO2]/mmol dm–3)
d[ES] a Michaelis–Menten mechanism and the significance of the
= ka [E][S] − ka′[ES] − kb[ES] ≈ 0 maximum rate, vmax, for large [S]0 is explained by the Michaelis–
dt intercepts and the slope. Figure 17F.9 The Lineweaver–Burk plot of the data for
Menten mechanism.
Example 17F.2.
It follows that
A note on good practice The slope and the intercept are unit-less:
ka [E][S]
[ES] = all graphs should be plotted as pure numbers.
ka′ + kb Equation 17F.16 predicts that, in accord with experimental Example 17F.2 Analysing data by using a Lineweaver–
observations (Fig. 17F.7): Burk plot Self-test 17F.2 The enzyme α-chymotrypsin is secreted in
where [E] and [S] are the concentrations of free enzyme and
substrate, respectively. • When [S]0 << KM, the rate is proportional to [S]0: The enzyme carbonic anhydrase catalyses the hydration of the pancreas of mammals and cleaves peptide bonds between
CO2 in red blood cells to give hydrogencarbonate (bicarbon- certain amino acids. Several solutions containing the small
Step 2: Simplify the expression for [ES] k peptide N-glutaryl-l-phenylalanine-p-nitroanilide at differ-
v = b [S]0[E]0 (17F.17a) ate) ion: CO2(g) + H2O(l) → HCO3−(aq) + H+(aq). The follow-
Now define the Michaelis constant as KM ent concentrations were prepared and the same small amount
ing data were obtained for the reaction at pH = 7.1, 273.5 K,
and an enzyme concentration of 2.3 nmol dm−3: of α-chymotrypsin was added to each one. The following
ka′ + kb • When [S]0 >> KM, the rate reaches its maximum value and
KM = data were obtained on the initial rates of the formation of
ka is independent of [S]0:
[CO2]/(mmol dm−3) 1.25 2.5 5 20 product:
vmax = k b[E]0 (17F.17b) v/(mmol dm−3 s−1) 2.78 × 10−2 5.00 × 10−2 8.33 × 10−2 1.67 × 10−1
(Note that this constant has the dimensions of molar concen- [S]/(mmol dm−3) 0.334 0.450 0.667 1.00 1.33 1.67
tration.) To express the rate law in terms of the total concen- Determine the maximum velocity and the Michaelis constant v/(mmol dm−3 s−1) 0.150 0.199 0.285 0.406 0.516 0.619
trations of enzyme and the initial concentration of substrate
Substitution of this definition of vmax into eqn 17F.16 gives
for the reaction.
first added, note that the total concentration of the enzyme is vmax Determine the maximum velocity and the Michaelis constant
v= (17F.18a) Collect your thoughts Prepare a Lineweaver–Burk plot as for the reaction.
[E]0 = [E] + [ES]. It follows that [E] = [E]0 − [ES]. This expres- 1+ K M /[S]0
sion for [E] is inserted into the steady-state expression for [ES] explained in the text and determine the values of KM and vmax Answer: vmax = 2.80 mmol dm−3 s −1, K M = 5.89 mmol dm−3

above to give which can be rearranged into a form suitable for data analysis by linear regression analysis.
by linear regression by taking reciprocals of both sides: The solution Draw up the following table:
([E]0 − [ES])[S] The action of an enzyme may be partially suppressed by
[ES] =
KM
1 1  KM  1
= + the presence of a foreign substance, which is called an inhibi-
v vmax  vmax  [S]0 Lineweaver–Burk plot (17F.18b) 1/([CO2]/(mmol dm−3)) 0.800 0.400 0.200 0.0500
1/(v/(mmol dm−3 s−1)) 36.0 20.0 12.0 6.0 tor. An inhibitor may be a poison that has been administered
Because the substrate is typically in large excess relative to
A Lineweaver–Burk plot is a plot of 1/v against 1/[S]0. to the organism, or it may be a substance that is naturally
the enzyme, the free substrate concentration is approximately Figure 17F.9 shows the Lineweaver–Burk plot for the data. The
According to eqn 17F.18b, it should yield a straight line with present in a cell and involved in its regulatory mechanism.
equal to the initial substrate concentration: [S] ≈ [S]0. The slope is 40.0 and the y-intercept is 4.00. Hence,
slope of KM/vmax, a y-intercept at 1/vmax, and an x-intercept at An inhibitor typically works by blocking the active site or
solution of the resulting expression for [ES] is
−1/KM (Fig. 17F.8). The value of KM can also be obtained from 1 1 attaching elsewhere to the enzyme and forcing a change in
vmax /(mmoldm −3 s −1 ) = = = 0.250
[S]0[E]0 [E]0 the ratio of the slope to the y-intercept. The value of k b is then intercept 4.00 geometry at the site so that it can no longer accommodate
[ES] = = calculated from the y-intercept and eqn 17F.17b. However, the the substrate.1
K M +[S]0 1+ K M /[S]0
plot cannot give the individual rate constants ka and ka′ that so vmax = 0.250 mmol dm−3 s −1.
Step 3 Write an expression for the rate law appear in the definition of KM. The stopped-flow technique de- slope 40.00 1
The use of kinetic criteria to distinguish different types of inhibition is
scribed in Topic 17A can give the additional data needed, be- K M /(mmoldm −3 ) = = = 10.0
The expression for [ES] can now be substituted into v = k b[ES], intercept 4.00 described in our Physical chemistry for the life sciences (2012).
to give the Michaelis–Menten equation cause the rate of formation of the enzyme–substrate complex
can be found by monitoring the concentration after mixing
kb[E]0 (17F.16) the enzyme and substrate. This procedure gives a value for ka,
v= and ka′ is then found by combining this result with the values
1+ K M /[S]0 Michaelis–Menten equation
of k b and KM.
17F Examples of reaction mechanisms 761

Checklist of concepts TOPIC 17G Photochemistry


☐ 1. The Lindemann–Hinshelwood mechanism of ‘unimo- ☐ 5. The kinetic chain length is the ratio of the number of
lecular’ reactions accounts for the first-order kinetics of monomer units consumed to the number of radicals
some gas-phase reactions. produced in the initiation step.
☐ 2. In stepwise polymerization any two monomers in the ☐ 6. Enzymes are homogeneous biological catalysts.
reaction mixture can link together at any time. ☐ 7. The Michaelis–Menten mechanism of enzyme kinetics Table 17G.1 Examples of photochemical processes
☐ 3. The longer a stepwise polymerization proceeds, the accounts for the dependence of rate on the concentra- ➤ Why do you need to know this material? Process General form Example
higher the average molar mass of the product. tions of the substrate and the enzyme. Many chemical and biological processes, including pho-
Ionization A* → A+ + e− NO* → NO+ + e−
☐ 4. In chain polymerization an activated monomer attacks ☐ 8. A Lineweaver–Burk plot is used to determine the tosynthesis and vision, are initiated by the absorption of
Electron A* + B → A+ + B− Ru(bpy)32+* + Fe3+ → Ru(bpy)33+ + Fe2+
another monomer and links to it; the slower the initia- parameters that occur in the Michaelis–Menten electromagnetic radiation, so you need to know how to transfer or A− + B+
tion of the chain, the higher the average molar mass of mechanism. include its effect in rate laws. The quantitative analysis of
Dissociation A* → B + C O3* → O2 + O
the polymer. these processes provides insight into their mechanisms.
A* + B–C → Hg* + CH4 → Hg + CH3 + H
A+B+C
➤ What is the key idea?
Addition A* + A* → B
The mechanisms of many photochemical reactions lead * + * and isomers
to relatively simple rate laws that yield rate constants and
Checklist of equations quantitative measures of the efficiency with which radiant A* + B → AB Hg* + H2 → HgH + H
energy induces reactions. Abstraction A* + B−C → Hg* + CH3−H → Hg−CH3 + H
Property Equation Comment Equation number A−B + C
Lindemann–Hinshelwood rate law d[P]/dt = kr[A] with kr = ka kb / ka′ ka′[A]>> kb 17F.7 ➤ What do you need to know already? Isomerization A* → A′
* O
You need to be familiar with the concepts of singlet and or
Effective rate constant 1/ kr = ka′ / ka kb +1/ ka[A] Lindemann–Hinshelwood mechanism 17F.8 rearrange-
triplet states (Topic 11F), modes of radiative decay (fluo- ment O
Fraction of condensed groups p = krt[A]0 /(1 + krt[A]0 ) Stepwise polymerization 17F.11
rescence and phosphorescence, Topic 11G), concepts of
Degree of polymerization 〈 N 〉 = 1/(1 − p ) = 1 + krt[A]0 Stepwise polymerization 17F.12 electronic spectroscopy (Topic 11F), and the formulation of * Excited state.

Rate of polymerization v = kr[In]1/2[M] Chain polymerization 17F.13 a rate law from a proposed mechanism (Topic 17E).
Kinetic chain length λ = kr [M][In]−1/2 , kr = kp (4 f kik t )−1/2 Chain polymerization 17F.14c
Degree of polymerization 〈 N 〉 = 2kr [M][In]−1/2 Chain polymerization 17F.15
Michaelis–Menten equation v = vmax/(1 + KM/[S]0) 17F.18a Photochemical processes are initiated by the absorption of Competing with the formation of photochemical products
Lineweaver–Burk plot 1/v = 1/vmax + (KM/vmax)(1/[S]0) 17F.18b electromagnetic radiation. Among the most important of are numerous primary photophysical processes that can de-
these processes are the ones that capture the radiant energy activate the excited state (Table 17G.2). Therefore, it is impor-
of the Sun. Some of these reactions lead to the heating of the tant to consider the timescales of the formation and decay of
atmosphere during the daytime by absorption of ultraviolet excited states before describing the mechanisms of photo-
radiation. Others include the absorption of visible radiation chemical reactions.
during photosynthesis. Without photochemical processes, the Electronic transitions caused by absorption of ultravio-
Earth would probably be simply a warm, sterile, rock. let and visible radiation occur within 10−16–10−15 s. The upper
limit for the rate constant of a first-order photochemical
reaction is then expected to be about 1016 s−1. Fluorescence is
slower than absorption, with typical lifetimes of 10 −12–10−6 s.
17G.1 Photochemical processes Therefore, the excited singlet state can initiate very fast photo-
chemical reactions in the femtosecond (10−15 s) to picosec-
Table 17G.1 summarizes common photochemical reactions. ond (10−12 s) range. Examples of such ultrafast reactions are
Photochemical processes are initiated by the absorption of the initial events of vision and of photosynthesis. Internal
radiation by at least one component of a reaction mixture. conversion (IC) occurs on a timescale similar to that for the
In a primary process, products are formed directly from the release of vibrational energy in molecules, so can occur in less
excited state of a reactant. Examples include fluorescence than 10−12 s. Typical intersystem crossing (ISC, Topic 11G) and
(Topic 11G) and cis–trans photoisomerizations. Products of phosphorescence lifetimes for large organic molecules are
a secondary process originate from intermediates that are 10−12–10−4 s and 10−6–10−1 s, respectively. As a consequence of
formed directly from the excited state of a reactant, such as their long lifetimes, excited triplet states are photochemically
oxidative processes initiated by the oxygen atom formed by important. Indeed, because phosphorescence decay is sev-
the photodissociation of ozone. eral orders of magnitude slower than most typical reactions,

17G Photochemistry 763 764 17 Chemical kinetics

Table 17G.2 Common photophysical processes by the number of photons absorbed by the molecule in the With Ndecomposed = (3.44 × 10−3 mol) × (6.022 × 1023 mol−1) =
same interval: 2.07… × 1021, P = 1.00 W = 1.00 J s −1, t = 2700 s, λ = 490 nm = 17G.3Mechanism of decay of excited
Primary absorption S + hν → S*
number of events N
4.90 × 10−7 m, and f = 0.60 it follows that singlet states
Excited-state absorption S* + hν → S** φ= = events
number of photons absorbed N abs (2.07…×1021 ) × (6.626 ×10−34 Js) × (2.998 ×108 ms −1 )
T* + hν → T** φ= Consider the formation and decay of an excited singlet state in
Primary quantum yield 0.60 × (1.00Js −1 ) × (2700s) × (4.90 ×10−7 m)
Fluorescence S* → S + hν
[definition]
(17G.1a) the absence of a chemical reaction:
= 0.52
Stimulated emission S* + hν → S + 2hν
When both the numerator and denominator of this expression That is, about half the photons that are absorbed bring about Absorption: S + hνi → S* vabs = Iabs
Intersystem crossing (ISC) S* → T*
Phosphorescence T* → S + hν
are divided by the time interval over which the events occur, photodissociation. Fluorescence: S* → S + hνf v F = kF[S*]
the primary quantum yield is also seen to be the rate of Internal conversion: S* → S v IC = kIC[S*]
Internal conversion (IC) S* → S Self-test 17G.1 In an experiment to measure the quantum
radiation-induced primary events divided by the rate of pho-
Collision-induced emission S* + M → S + M + hν yield of a photochemical reaction, the absorbing substance Intersystem crossing: S* → T* v ISC = kISC[S*]
ton absorption, Iabs: was exposed to 320 nm radiation from an 87.5 mW laser source
Collisional deactivation S* + M → S + M in which S is an absorbing singlet-state species, S* an excited
rate of process v for 38 min. The intensity of the transmitted light was 0.35 that
T* + M → S + M φ= = singlet state, T* an excited triplet state, and hνi and hνf are the
rate of photon absorption I abs of the incident light. As a result of irradiation, 0.324 mmol of
Electronic energy transfer: the absorbing substance decomposed. Determine the primary energies of the incident and fluorescent photons, respectively.
Singlet–singlet S* + S → S + S* Primary quantum yield in terms of rates of processes (17G.1b) quantum yield. From the methods presented in Topic 17E, the rate of forma-
Triplet–triplet T* + T → T + T* Answer: ϕ = 0.93 tion of S* and its net rate of disappearance may be written as:
Excimer formation S* + S → (SS)*
Example 17G.1 Calculating a primary quantum yield Rate of formation of S* = Iabs
Energy pooling
A molecule in an excited state must either decay to the
Singlet–singlet S* + S* → S** + S In an experiment to determine the quantum yield of a photo- Rate of disappearance of S* = kF[S*] + kISC[S*] + kIC[S*]
ground state or form a photochemical product. Therefore, the
Triplet–triplet T* + T* → S** + S chemical reaction, the absorbing substance was exposed total number of molecules deactivated by radiative processes, = (kF + kISC + kIC)[S*]
to light of wavelength 490 nm from a 1.00 W laser source
* Denotes an excited state; S is a singlet state, T a triplet state, and M a ‘third body’. non-radiative processes, and photochemical reactions must be
for 2700 s, with 60 per cent of the incident light being
equal to the number of excited species produced by absorption It follows that the excited state decays by a first-order process
absorbed. As a result of irradiation, 3.44 mmol of the absorb-
of the incident radiation. It follows that the sum of primary so, when the light is turned off, the concentration of S* varies
ing substance decomposed. What is the primary quantum
species in excited triplet states can undergo a very large quantum yields ϕi for all photophysical and photochemical with time t as
yield?
number of collisions with other reactants before they lose their events i must be equal to 1, regardless of the number of reac-
energy radiatively. Collect your thoughts You need to calculate the quanti- tions involving the excited state: [S*](t ) = [S*]0 e − t /τ 0
(17G.3a)
ties in eqn 17G.1a. The number of photochemical events
vi 1
Brief illustration 17G.1
is simply the number of decomposed molecules, Nevents = ∑φi = ∑ I abs I abs ∑
= vi = 1 (17G.2a) where the observed lifetime, τ0, of the excited singlet state is
Ndecomposed. To calculate the number of absorbed photons Nabs, i i i
defined as
note that: Then, from eqn 17G.1b in the form φi = vi / I abs it follows that
To judge whether the excited singlet or triplet state of the reac-
tant is a suitable product precursor, the emission lifetimes are v 1 Observed lifetime of
• The energy absorbed by the substance is Eabs = fPt, where φi = i (17G.2b) τ0 = the excited singlet state (17G.3b)
compared with the half-life of the relevant chemical reaction
P is the incident power, t is the time of exposure, and the ∑ vi kF + kISC + kIC [definition]
(Topic 17B). Consider a unimolecular photochemical reaction i
factor f (in this case f = 0.60) is the fraction of incident
with rate constant kr = 1.7 × 104 s −1, and therefore a half-life Therefore, the primary quantum yield of a particular process This expression can be used in a kinetic analysis of the decay of
light that is absorbed.
of 41 μs. The observed fluorescence lifetime of the reactant may be determined directly from the experimental rates of all S* to find an expression for the quantum yield of fluorescence.
is 1.0 ns and the observed phosphorescence lifetime is 1.0 ms. • Eabs is also related to the number Nabs of absorbed photons photophysical and photochemical processes that deactivate
The excited singlet state is therefore too short-lived to be a through Eabs = Nabshν = Nabshc/λ, where hc/λ is the energy the excited state.
major source of product in this reaction. On the other hand, of a single photon of wavelength λ. How is that done? 17G.1 Deriving an expression for the
If it is assumed that the only photophysical processes for the
the relatively long-lived excited triplet state is a good candi- excited singlet state are fluorescence, internal conversion, and quantum yield of fluorescence
date for an intermediate. You can combine these two expressions for the absorbed
phosphorescence, then it follows that Most fluorescence measurements are conducted by illuminat-
energy to obtain Nabs. The primary quantum yield follows
from ϕ = Ndecomposed/Nabs. ϕF + ϕIC + ϕP = 1 ing a dilute sample with a continuous and intense beam of
visible or ultraviolet radiation. It follows that [S*] is small and
The solution From the two expressions for the absorbed where ϕF, ϕIC, and ϕP are the quantum yields of fluorescence, constant, so the steady-state approximation (Topic 17E) may
energy, it follows that internal conversion, and phosphorescence, respectively (inter- be used for [S*]:
17G.2 The primary quantum yield system crossing from the singlet to the triplet state is taken into
 hc  d[S*]
fPt = N abs   account by the presence of ϕP). The quantum yield of photon = I abs − kF[S*] − kISC[S*] − kIC[S*]
λ dt
The rates of deactivation of the excited state by radiative, emission by fluorescence and phosphorescence is ϕemission = ϕF +
= I abs − (kF + kISC + kIC )[S*] ≈ 0
non-radiative, and chemical processes determine the yield and therefore that N abs = fPt λ / hc . Now use eqn 17G.1a to write ϕP, which is less than 1. If the excited singlet state also participates
of product in a photochemical reaction. The primary quan- in a primary photochemical reaction with quantum yield ϕr, then Consequently,
tum yield, ϕ, is defined as the number of photophysical or N decomposed N decomposed hc
φ= =
photochemical events that lead to primary products divided N abs fPt λ ϕF + ϕIC + ϕP + ϕr = 1 I abs = (kF + kISC + kIC )[S*]
17G Photochemistry 765 766 17 Chemical kinetics

The rate of fluorescence, vF , is kF[S*], so it follows from eqn d[S*] Stern–Volmer plots can therefore be made by plotting either and collect terms:
= I abs − (kF + kISC + kIC + kQ[Q])[S*] ≈ 0
17G.1b that the quantum yield of fluorescence is dt IF,0/IF or τ0/τ against the quencher concentration. The slope and
v kF[S*] and the fluorescence quantum yield is intercept are the same as those shown for eqn 17G.5. y y-intercept
φF,0 = F = !#"# $ !#"#$ !### slope
"###$
I abs (kF + kISC + kIC )[S*] kF 1 1
φF = Example 17G.2 = + (kQ ×10−9 mol dm −3 s)
which, by cancelling the [S*], simplifies to kF + kISC + kIC + kQ[Q] Determining the quenching rate constant τ /(10−7 s) τ 0 /(10−7 s)
x
kF The ratio of the quantum yields without and with a quencher The molecule 2,2′-bipyridine (1, bpy) forms a complex with !###"###$
φF,0 = present is the Ru2+ ion. Tris-(2,2′-bipyridyl)ruthenium(II), [Ru(bpy)3]2+ × [Q]/(10−2 mol dm −3 )
kF + kISC + kIC
(2), has a strong metal-to-ligand charge transfer (MLCT)
Then, by using the result for the lifetime in eqn 17G.3b, φF,0 kF k + k + k + k [Q] transition (Topic 11F) at 450 nm.
= × F ISC IC Q Note that because slope = kQ ×10−9 mol dm −3 s, then kQ = slope ×
φF kF + kISC + kIC kF
(17G.4) 109 dm3 mol−1 s −1. Draw up the following table with Q =
φF,0 = kFτ 0 k + k + k + k [Q] 2+ [Fe(OH2)6]3+:
Quantum yield of fluorescence = F ISC IC Q N
kF + kISC + kIC N N
The observed fluorescence lifetime can be measured by N N
kQ Ru [[Fe(OH2)6]3+]/(10−2 mol dm–3) 0 1.6 4.7 7.0 9.4
using a pulsed laser technique. First, the sample is excited = 1+ [Q] N N
kF + kISC + kIC N N
1/(τ/10−7 s) 0.167 0.247 0.297 0.338 0.461
with a short light pulse from a laser using a wavelength at N
which S absorbs strongly. Then, the exponential decay of the By recognizing from eqn 17G.3b that 1/(kF + kISC + kIC ) = τ 0 , this Figure 17G.2 shows a plot of 1/(τ/10−7 s) against [[Fe(OH2)6]3+]/
fluorescence intensity after the pulse is monitored. expression becomes the Stern–Volmer equation: 1 2,2′-Bipyridine (bpy) 2 [Ru(bpy)3]2+
(10−2 mol dm−3) and the results of a fit to this expression. The
slope of the line is 0.029, so kQ = 2.9 × 107 dm3 mol−1 s −1.
Brief illustration 17G.2 φF,0 (17G.5)
= 1+ τ 0 kQ[Q]
φF Stern–Volmer equation The quenching of the *[Ru(bpy)3]2+ excited state by Fe3+
At a certain wavelength, the fluorescence quantum yield (present as the complex ion [Fe(OH2)6]3+) in acidic solution 0.5
and observed fluorescence lifetime of tryptophan in water The Stern–Volmer equation implies that a plot of ϕF,0/ϕF was monitored by measuring emission lifetimes at 600 nm.
are ϕF,0 = 0.20 and τ0 = 2.6 ns, respectively. It follows from eqn against [Q] should be a straight line with slope τ0 kQ. Such a Determine the quenching rate constant for this reaction from
17G.4 that the fluorescence rate constant kF is the following data: 0.4
plot is called a Stern–Volmer plot (Fig. 17G.1). The method

1/(τ/10–7 s)
φF,0 0.20 may also be applied to the quenching of phosphorescence.
kF = = = 7.7 ×107 s −1 [[Fe(OH2)6]3+]/(10−2 mol dm−3) 0 1.6 4.7 7.0 9.4
τ 0 2.6 ×10−9 s Equation 17G.4 in the form kF = φF,0 /τ 0 shows that the rate −7
0.3
constant for fluorescence, and hence the rate of fluorescence τ/(10 s) 6.00 4.05 3.37 2.96 2.17
(which determines the intensity of fluorescence), is pro- Collect your thoughts Rewrite the Stern–Volmer equation 0.2
portional to the quantum yield. The ratio ϕF,0/ϕF is therefore (eqn 17G.5) for use with lifetime data; then fit the data to a
17G.4 Quenching equal to the ratio IF,0/IF, where IF,0 is the intensity of fluores- straight line. 0.1
cence in the absence of quencher and IF the intensity when 0 2,5 5 7.5 10
The shortening of the lifetime of the excited state by the pres- quencher is present. Similarly, from the same equation in The solution Substitute τ0/τ for ϕF,0/ϕF in eqn 17G.5 and, after [[Fe(OH2)6 ]3+]/(10–2 mol dm–3)
ence of another species is called quenching. Quenching may the form τ 0 = φF,0 / kF, the fluorescence lifetime is also propor- rearrangement, obtain
Figure 17G.2 The Stern–Volmer plot of the data for Example
be either a desired process, such as in energy or electron trans- tional to the quantum yield, so the ratio τ0/τ (where τ is the 1 1
= + k [Q] 17G.2.
fer, or an undesired side reaction that can decrease the quan- lifetime in the presence of the quencher) is also equal to ϕF,0/ϕF. τ τ0 Q
tum yield of a desired photochemical process. Quenching
Because the axes of plots should be labelled with pure num-
effects are studied by monitoring the emission from the ex-
bers, it is necessary to introduce and handle units before using Comment. Measurements of emission lifetimes are preferred
cited state that is involved in the photochemical reaction.
this equation for the analysis of the data. To bring the expres- because they yield the value of kQ directly. To determine
The addition of a quencher, Q, opens an additional channel the value of kQ from intensity or quantum yield measure-
sion into a form suitable for plotting, it needs to be expressed
for deactivation of S*: in terms of τ/(10−7 s) and [Q]/(10−2 mol dm−3) to match the data, ments, it is necessary to make an independent measurement
Slope = τ0kQ
ϕF,0/ϕF

and therefore (with these dimensionless terms in blue) to of τ0.


Quenching: S* + Q → S + Q vQ = kQ[Q][S*] write it as
Self-test 17G.2 The quenching of tryptophan fluorescence
The fluorescence quantum yields ϕF,0 and ϕF measured in the 1 1 by dissolved O2 gas was monitored by measuring emission
= + {kQ[Q]/(10−2 mol dm −3 )} lifetimes at 348 nm in aqueous solutions. Determine the
absence and presence of Q, respectively, can be expressed in 1
(10−7 s)τ /(10−7 s) τ 0
terms of the molar concentration of the quencher, [Q]. −2 −3 quenching rate constant for this process from the following
× (10 mol dm )
data:
0
How is that done? 17G.2 Assessing the effect of a
Quencher concentration, [Q] Now multiply through by 10−7 s to obtain
[O2]/(10−2 mol dm−3) 0 2.3 5.5 8 10.8
quencher on the fluorescence quantum yield Figure 17G.1 The form of a Stern–Volmer plot and the 1 10−7 s
= + kQ × (10−2 mol dm −3 ) × (10−7 s) τ/ns 2.6 1.5 0.92 0.71 0.57
interpretation of the slope in terms of the rate constant for τ0
τ /(10−7 s)
In the presence of quenching, the steady-state approximation quenching and the observed fluorescence lifetime in the absence Answer: 1.3 × 1010 dm3 mol−1 s −1
for [S*] becomes of quenching. × [Q]/(10−2 mol dm −3 )

17G Photochemistry 767 768 17 Chemical kinetics

Three common mechanisms for bimolecular quenching of Table 17G.3 Values of R0 for some donor–acceptor pairs* Equation 17G.7 forms the basis of fluorescence resonance protein. From eqn 17G.6 it follows that ηT = 1 − 0.68/0.75 =
an excited singlet (or triplet) state are: energy transfer (FRET), in which the dependence of the en- 0.093 and from eqn 17G.7 and the known value of R0 = 5.4 nm
Donor‡ Acceptor R0/nm
ergy transfer efficiency, ηT, on the distance, R, between energy for the 1.5-I AEDANS/11-cis-retinal, R = 7.9 nm. Therefore,
Collisional deactivation: S* + Q → S + Q Naphthalene Dansyl 2.2 donor and acceptor is used to measure distances in biological take 7.9 nm to be the distance between the surface of the
Resonance energy transfer: S* + Q → S + Q* Dansyl ODR 4.3 systems. In a typical FRET experiment, a site on a biopolymer protein and 11-cis-retinal.
Electron transfer: S* + Q → S +/− + Q−/+ Pyrene Coumarin 3.9 or membrane is labelled covalently with an energy donor and
1.5-I AEDANS FITC 4.9 another site is labelled covalently with an energy acceptor.
NH I
The quenching rate constant itself does not give much insight Tryptophan 1.5-I AEDANS 2.2 In certain cases, the donor or acceptor may be natural con- HN
into the mechanism of quenching. Collisional quenching is stituents of the system, such as amino acid groups, cofactors, O
Tryptophan Haem (heme) 2.9
particularly efficient when Q is a species, such as iodide ion, or enzyme substrates. The distance between the labels is then
*Additional values may be found in J.R. Lacowicz, Principles of fluorescence spectros-
which receives energy from S* and then decays to the ground copy, Kluwer Academic/Plenum, New York (1999). calculated from the known value of R0 and eqn 17G.7. Several SO3–
CHO
state primarily by releasing energy as heat. For the system of ‡
Abbreviations: tests have shown that the FRET technique is useful for meas-
3 1.5-I AEDANS 4 11-cis-Retinal
Example 17G.2, it is known that the quenching of the excited Dansyl: 5-dimethylamino-1-naphthalenesulfonic acid uring distances ranging from 1 to 9 nm.
state of [Ru(bpy)3]2+ is a result of electron transfer to Fe3+, but FITC: fluorescein 5-isothiocyanate
the quenching data do not prove the mechanism. 1.5-I AEDANS: 5-(((2-iodoacetyl)amino)ethyl)amino)naphthalene-1-sulfonic acid (3)
Brief illustration 17G.3
ODR: octadecyl-rhodamine

As an illustration of the FRET technique, consider a study of If donor and acceptor molecules diffuse in solution or in
the protein rhodopsin. When an amino acid on the surface the gas phase, Förster theory predicts that the efficiency of
17G.5 Resonance energy transfer ηT =
R06 Efficiency of energy transfer in terms
(17G.7)
of rhodopsin was labelled covalently with the energy donor quenching by energy transfer increases as the average distance
R + R6
6
0
of the donor–acceptor distance 1.5-I AEDANS (3), the fluorescence quantum yield of the label travelled between collisions of donor and acceptor decreases.
The energy transfer process S* + Q → S + Q* can be regarded as decreased from 0.75 to 0.68 due to quenching by the visual That is, the quenching efficiency increases with concentration
taking place as follows. The oscillating electric field of the in- where R0 is a parameter (with dimensions of distance) that is pigment 11-cis-retinal (4), which is attached elsewhere in the of quencher, as predicted by the Stern–Volmer equation.
coming electromagnetic radiation induces an oscillating elec- characteristic of each donor–acceptor pair. It can be regarded
tric dipole moment (a transition dipole moment) in S. Energy as the distance at which energy transfer is 50 per cent efficient
is absorbed by S if the frequency of the incident radiation, ν, for a given donor–acceptor pair. (This assertion can be con-
is such that ν = ∆ES/h, where ∆ES is the energy separation of firmed by using R = R0 in eqn 17G.7.) Equation 17G.7 has been Checklist of concepts
the ground and excited electronic states of S and h is Planck’s verified experimentally and values of R0 are available for a
constant. This is the ‘resonance condition’ for absorption of number of donor–acceptor pairs (Table 17G.3). ☐ 1. The primary quantum yield of a photochemical reac- ☐ 4. Collisional deactivation, electron transfer, and res-
radiation (essentially the Bohr frequency condition, eqn 7A.9). The emission and absorption spectra of molecules span a tion is the number of reactant molecules producing onance energy transfer are common fluorescence
The oscillating dipole on S can now affect electrons bound to a range of wavelengths, so the second requirement of the Förster specified primary products for each photon absorbed. quenching processes.
nearby Q molecule by inducing an oscillating dipole moment theory is met when the emission spectrum of the donor mol-
☐ 2. The observed lifetime of an excited state is related to ☐ 5. The efficiency of resonance energy transfer decreases
(another transition dipole moment) in them. If the frequency ecule overlaps significantly with the absorption spectrum of
the quantum yield and rate constant of emission. with increasing separation between donor and acceptor
of oscillation of the electric dipole moment in S is such that the acceptor. In the overlap region, a photon emitted by the
☐ 3. A Stern–Volmer plot is used to analyse the kinetics of molecules.
ν = ∆EQ/h, where ∆EQ is the energy separation of the ground donor has the appropriate energy to be absorbed by the accep-
and excited electronic states of Q, then Q will absorb energy tor (Fig. 17G.3). fluorescence quenching in solution.
from S. The coupling of the two transition moments can be
regarded as an exchange of a photon, in which a photon gener-
ated by S is absorbed by Q. Emission
The efficiency, ηT, of resonance energy transfer is defined as spectrum of S* Checklist of equations
φF Efficiency of resonance energy transfer Absorption
ηT = 1− (17G.6) spectrum Property Equation Comment Equation number
φF,0 [definition]
Intensity

of Q
Primary quantum yield φ = v / I abs 17G.1b
According to the Förster theory of resonance energy transfer,
Excited state lifetime τ 0 = 1/(kF + kISC + kIC ) No quencher present 17G.3b
energy transfer is efficient when:
Quantum yield of fluorescence ϕF,0 = kF/(kF + kISC + kIC) = kFτ0 Without quencher present 17G.4
• The energy donor and acceptor are separated by a short Stern–Volmer equation φF,0 /φF = 1 + τ 0 kQ[Q] 17G.5
distance (of the order of nanometres). Efficiency of resonance energy transfer ηT = 1 −φF /φF,0 Definition 17G.6
• The photon is regarded as emitted by the excited state of Frequency, ν ηT = R06 /(R06 + R 6 ) Förster theory 17G.7
the donor and then absorbed directly by the acceptor. Figure 17G.3 According to the Förster theory, the rate of energy
transfer from a molecule S* in an excited state to a quencher
For donor–acceptor systems held rigidly either by covalent molecule Q is optimized at radiation frequencies for which the
bonds or by a protein ‘scaffold’, ηT increases with decreasing emission spectrum of S* overlaps the absorption spectrum of Q,
distance, R, according to as shown in the (dark green) shaded region.
Exercises and problems 769 770 17 Chemical kinetics

FOCUS 17 Chemical kinetics TOPIC 17B Integrated rate laws


Discussion questions
D17B.1 Describe the main features, including advantages and disadvantages, D17B.3 Write the rate law that corresponds to each of the following
TOPIC 17A The rates of chemical reactions of the following experimental methods for determining the rate law of a expressions: (a) [A] = [A]0 − krt, (b) ln([A]/[A]0) = −krt, and (c) [A] =
reaction: the isolation method, the method of initial rates, and fitting data to [A]0/(1 + krt[A]0).
Discussion questions integrated rate law expressions.
D17B.2 What is the origin of the classification of a reaction as having
D17A.1 Summarize the characteristics of zeroth-order, first-order, second- D17A.3 What are the advantages of ascribing an order to a reaction?
pseudofirst and pseudosecond order? Under what conditions can the apparent
order, and pseudofirst-order reactions.
D17A.4 Summarize the experimental procedures that can be used to monitor order of a reaction change?
D17A.2 When can a reaction order not be ascribed? the composition of a reaction system.

Exercises
Exercises E17B.1(a) A number of reactions that take place on the surfaces of catalysts are E17B.4(a) The second-order rate constant for the reaction CH3COOC2H5(aq) +
E17A.1(a) Predict how the total pressure varies during the reaction 2 ICl(g) + E17A.5(b) The rate law for the reaction in Exercise E17A.3(b) was found to be zeroth order in the reactant. One example is the decomposition of ammonia OH−(aq) → CH3CO2−(aq) + CH3CH2OH(aq) is 0.11 dm3 mol−1 s−1. What is
2
H2(g) → I2(g) + 2 HCl(g) in a constant-volume container. Assume that at the v = kr[A][B] . What are the units of kr when the concentrations are in moles on hot tungsten. In an experiment, the partial pressure of ammonia decreased the concentration of ester (CH3COOC2H5) after (i) 20 s, (ii) 15 min when
start of the reaction the partial pressures of the reactants are equal and that no per cubic decimetre? Express the rate law in terms of (i) the rate of formation from 21 kPa to 10 kPa in 770 s. (i) What is the rate constant for the zeroth- ethyl ethanoate is added to aqueous sodium hydroxide so that the initial
products are present. of C and (ii) the rate of consumption of A. order reaction? (ii) How long will it take for all the ammonia to be concentrations are [NaOH] = 0.060 mol dm−3 and [CH3COOC2H5] =
E17A.1(b) Predict how the total pressure varies during the reaction N2(g) + consumed? 0.110 mol dm−3?
E17A.6(a) The rate law for the reaction in Exercise E17A.4(a) was reported as
3 H2(g) → 2 NH3(g) in a constant-volume container. Assume that at the start E17B.1(b) In a study of the enzyme-catalysed oxidation of ethanol, the E17B.4(b) The second-order rate constant for the reaction A + 2 B → C + D
d[C]/dt = kr[A][B][C]. Express the rate law in terms of the reaction rate v. What
of the reaction the partial pressures of H2 and N2 are in the ratio 3 to 1 and molar concentration of ethanol decreased in a first-order reaction from is 0.34 dm3 mol−1 s−1. What is the concentration of C after (i) 20 s, (ii) 15 min
are the units of kr when the concentrations are in moles per cubic decimetre?
that no products are present. 220 mmol dm−3 to 56.0 mmol dm−3 in 1.22 × 104 s. What is the rate constant of when the reactants are mixed with initial concentrations of [A] = 0.027 mol dm−1
E17A.6(b) The rate law for the reaction in Exercise E17A.4(b) was reported as
the reaction? and [B] = 0.130 mol dm−3?
E17A.2(a) The rate of formation of NO in the reaction 2 NOBr(g) → 2 NO(g) + d[C]/dt = kr[A][B][C]−1. Express the rate law in terms of the reaction rate v. What
Br2(g) was reported as d[NO]/dt = 0.24 mmol dm−3 s−1 under particular are the units of kr when the concentrations are in moles per cubic decimetre? E17B.2(a) At 518 °C, the half-life for the decomposition of a sample of gaseous E17B.5(a) A reaction 2 A → P has a second-order rate law with kr = 4.30 ×
conditions. What is the rate of formation of Br2? ethanal (acetaldehyde) initially at a partial pressure of 363 Torr was 410 s. 10−4 dm3 mol−1 s−1. Calculate the time required for the concentration of A to
E17A.7(a) If the rate laws are expressed with (i) concentrations in moles per
E17A.2(b) The rate of change of molar concentration of CH3 radicals in the When the partial pressure was 169 Torr, the half-life was 880 s. Identify the change from 0.210 mol dm−3 to 0.010 mol dm−3.
cubic decimetre, (ii) pressures in kilopascals, what are the units of a second-
reaction 2 CH3(g) → CH3CH3(g) was reported as d[CH3]/dt = −1.2 mol dm−3 s−1 order of the reaction. E17B.5(b) A reaction 2 A → P has a third-order rate law with kr = 6.50 ×
order and of a third-order rate constant?
under particular conditions. What is the rate of formation of CH3CH3? E17B.2(b) At 400 K, the half-life for the decomposition of a sample of a gaseous 10−4 dm6 mol−2 s−1. Calculate the time required for the concentration of A to
E17A.7(b) If the rate laws are expressed with (i) concentrations in molecules
compound initially at a partial pressure of 55.5 kPa was 340 s. When the change from 0.067 mol dm−3 to 0.015 mol dm−3.
E17A.3(a) The rate of the reaction A + 2 B → 3 C + D was reported as 2.7 mol per cubic metre, (ii) pressures in pascals, what are the units of a second-order
partial pressure was 28.9 kPa, the half-life was 178 s. Identify the order of the
dm−3 s−1. State the rates of formation and consumption of the participants. and of a third-order rate constant? E17B.6(a) The reaction A + B → P is found to be first order in both A and B.
reaction.
E17A.3(b) The rate of the reaction A + 3 B → C + 2 D was reported as 2.7 mol 1/2 The reaction was carried out in a solution that was initially 0.080 mol dm−3
E17A.8(a) The rate law v = (kr1[A][B])/(kr 2 + kr3[B] ) was established in a series of
dm−3 s−1. State the rates of formation and consumption of the participants. E17B.3(a) The rate constant for the first-order decomposition of N2O5 in the in A and 0.060 mol dm−3 in B. After 1.0 h the concentration of B had fallen to
experiments. Identify the conditions under which (i) an order with respect to A,
reaction 2 N2O5(g) → 4 NO2(g) + O2(g) is kr = 3.38 × 10−5 s−1 at 25 °C. What is 0.030 mol dm−3. (i) Calculate the rate constant. (ii) What are the half-lives of
E17A.4(a) The rate of formation of C in the reaction 2 A + B → 2 C + 3 D (ii) an order with respect to B, and (iii) an overall order, can be assigned.
the half-life of N2O5? If the initial partial pressure of N2O5 is 500 Torr, what the reactants?
is 2.7 mol dm−3 s−1. State the reaction rate, and the rates of formation or E17A.8(b) Certain gas-phase reactions of the type A → P have rate laws of the
will its partial pressure be (i) 50 s, (ii) 20 min after initiation of the reaction? E17B.6(b) A second-order reaction of the type A + 2 B → P was carried out in
consumption of A, B, and D. form v = ka kb[A]2/(kb + ka′[A]). What is the order with respect to A under a
E17B.3(b) The rate constant for the first-order decomposition of a compound a solution that was initially 0.050 mol dm−3 in A and 0.030 mol dm−3 in B. After
E17A.4(b) The rate of consumption of B in the reaction A + 3 B → C + 2 D variety of conditions that you should specify?
A in the reaction 2 A→ P is kr = 3.56 × 10−7 s−1 at 25 °C. What is the half-life 1.0 h the concentration of A had fallen to 0.040 mol dm−3. (a) Calculate the
is 2.7 mol dm−3 s−1. State the reaction rate, and the rates of formation or
E17A.9(a) At 400 K, the rate of decomposition of a gaseous compound was of A? If the initial partial pressure of A is 33.0 kPa, what will be its partial rate constant. (b) What is the half-life of each reactant?
consumption of A, C, and D.
9.71 Pa s−1 when 10.0 per cent had reacted and 7.67 Pa s−1 when 20.0 per cent pressure (i) 50 s, (ii) 20 min after initiation of the reaction?
E17A.5(a) The rate law for the reaction in Exercise E17A.3(a) was found to be had reacted. Identify the order of the reaction.
v = kr[A][B]. What are the units of kr when the concentrations are in moles E17A.9(b) At 350 K, the rate of decomposition of a gaseous compound was
per cubic decimetre? Express the rate law in terms of (i) the rate of formation 10.01 Pa s−1 when 10.0 per cent had reacted and 8.90 Pa s−1 when 20.0 per cent Problems
of C and (ii) the rate of consumption of A. had reacted. Identify the order of the reaction.
P17B.1 For a first-order reaction of the form A → n B (with n possibly P17B.4 The data below apply to the reaction, (CH3)3CBr(aq) + H2O(l) →
fractional) and [B]0 = 0, the concentration of the product varies with time as (CH3)3COH(aq) + HBr(aq). Identify the order of the reaction and calculate the
Problems [B]= n[A]0 (1 − e − krt ). Plot the time dependence of [A] and [B] for n = 12 , 1, and rate constant and the molar concentration of (CH3)3CBr remaining after 43.8 h.
2. Hint: To make your plots general, let the horizontal axis be krt and plot
P17A.1 The following initial-rate data were obtained on the rate of binding of P17A.3 The following kinetic data (v0 is the initial rate) were obtained for the [A]/[A]0 or [B]/[A]0 on the vertical axis. t/h 0 3.15 6.20 10.00 18.30 30.80
glucose with the enzyme hexokinase present at 1.34 mmol dm−3. What is (a) reaction 2 ICl(g) + H2(g) → I2(g) + 2 HCl(g): [(CH3)3CBr]/(10−2 mol dm−3) 10.39 8.96 7.76 6.39 3.53 2.07
the order of reaction with respect to glucose, (b) the rate constant? P17B.2 For a second-order reaction of the form A → n B (with n possibly
fractional) and [B]0 = 0, the concentration of the product varies with time
Experiment [ICl]0/(mmol dm−3) [H2]0/(mmol dm−3) v0/(mol dm−3 s−1) P17B.5 The thermal decomposition of an organic nitrile produced the
[C6H12O6]/(mmol dm−3) 1.00 1.54 3.12 4.02 as [B] = nkrt[A]02/(1 + krt[A]0). Plot the time dependence of [A] and [B] for
following data:
1 1.5 1.5 3.7 × 10−7 n = 12 , 1, and 2. Hint: See the hint to Problem P17B.1.
v0/(mol dm−3 s−1) 5.0 7.6 15.5 20.0
2 3.0 1.5 7.4 × 10−7 P17B.3 The data below apply to the formation of urea from ammonium t/(103 s) 0 2.00 4.00 6.00 8.00 10.00 12.00
P17A.2 The following data were obtained on the initial rates of a reaction of a
3 3.0 4.5 22 × 10−7 cyanate, NH4CNO → NH2CONH2. Initially 22.9 g of ammonium cyanate was [nitrile]/(mol dm−3) 1.50 1.26 1.07 0.92 0.81 0.72 0.65
d-metal complex with a reactant Y in aqueous solution. What is (a) the order
dissolved in enough water to prepare 1.00 dm3 of solution. Identify the order
of reaction with respect to the complex and Y, (b) the rate constant? For the 4 4.7 2.7 ?
of the reaction and calculate the rate constant and the mass of ammonium Identify the order of the reaction and calculate the rate constant.
experiments (i), [Y] = 2.7 mmol dm−3 and for experiments (ii) [Y] = 6.1 mmol dm−3.
cyanate left after 300 min. −
[complex]/(mmol dm−3) 8.01 9.22 12.11 P17B.6 The oxidation of HSO by O2 in aqueous solution is a reaction

(a) Write the rate law for the reaction. (b) From the data, determine the 3
of importance to the processes of acid rain formation and flue gas
v0/(mol dm−3 s−1) (i) 125 144 190 value of the rate constant. (c) Use the data to predict the reaction rate for t/min 0 20.0 50.0 65.0 150
(ii) 640 730 960 experiment 4. m(urea)/g 0 7.0 12.1 13.8 17.7 ‡
These problems were supplied by Charles Trapp and Carmen Giunta.

Exercises and problems 771 772 17 Chemical kinetics

desulfurization. R.E. Connick et al. (Inorg. Chem. 34, 4543 (1995)) report P17B.11 The composition of a liquid phase reaction 2 A → B was monitored by
that the reaction 2 HSO3−(aq) + O2(g) → 2 SO42−(aq) + 2 H+(aq) follows the
rate law v = kr[HSO3−]2[H+]2. Given pH = 5.6 and an O2 molar concentration
a spectrophotometric method with the following results:
TOPIC 17C Reactions approaching equilibrium
of 0.24 mmol dm−3 (both presumed constant), an initial HSO3− molar t/min 0 10 20 30 40 ∞
concentration of 50 μmol dm−3, and a rate constant of 3.6 × 106 dm9 mol−3 s−1, [B]/(mol dm−3) 0 0.089 0.153 0.200 0.230 0.312 Discussion questions
what is the initial rate of reaction? How long would it take for HSO3− to reach
D17C.1 Describe the strategy of a temperature-jump experiment. What param- D17C.2 What feature of a reaction would ensure that its rate can respond to a
half its initial concentration? Identify the order of the reaction with respect to A and calculate its rate eters of a reaction can be determined by this technique? pressure jump?
P17B.7 Pharmacokinetics is the study of the rates of absorption and constant.
elimination of drugs by organisms. In most cases, elimination is slower P17B.12 In the gas phase, the ClO radical decays rapidly by way of the reaction
than absorption and is a more important determinant of availability 2 ClO(g) → Cl2(g) + O2(g). The following data have been obtained: Exercises
of a drug for binding to its target. A drug can be eliminated by many
mechanisms, such as metabolism in the liver, intestine, or kidney followed E17C.1(a) The rates of the forward and reverse reactions for a reaction E17C.2(a) In a temperature-jump experiment to investigate the kinetics of an
t/ms 0.12 0.62 0.96 1.60 3.20 4.00 5.75
by excretion of breakdown products through urine or faeces. As an example A + B ! C were found to be 5.0 × 106 dm3 mol−1 s−1 (second order) and isomerization reaction that is first order in both directions, the relaxation time
of pharmacokinetic analysis, consider the elimination of beta adrenergic [ClO]/(μmol dm−3) 8.49 8.09 7.10 5.79 5.20 4.77 3.95 2.0 × 104 s−1 (first order). What is the equilibrium constant of the reaction? was measured as 27.6 μs. The rate constant for the forward reaction is known
blocking agents (beta blockers), which are used in the treatment of E17C.1(b) The equilibrium constant for the binding of a drug molecule to to be 12.4 ms–1. Calculate the rate constant for the reverse reaction.
hypertension. After intravenous administration of a beta blocker, the blood Calculate the rate constant of the reaction and the half-life of ClO. a protein was measured as 200. In a separate experiment, the rate constant E17C.2(b) The half-lives for the forward and reverse reactions that are first
plasma of a patient was analysed for remaining drug and the data are shown for the binding process, which is second order overall, was found to be order in both directions are 24 ms and 39 ms, respectively. Calculate the
P17B.13 Cyclopropane isomerizes into propene when heated to 500 °C in
below, where c is the drug concentration measured at a time t after the 1.5 × 108 dm3 mol–1 s–1. What is the rate constant for the first-order corresponding relaxation time for return to equilibrium after a temperature
the gas phase. The extent of conversion for various initial pressures has been
injection. dissociation of the drug molecule from the protein–drug complex? jump.
followed by gas chromatography by allowing the reaction to proceed for a
time with various initial pressures:
t/min 30 60 120 150 240 360 480
c/(ng cm−3) 699 622 413 292 152 60 24 p0/Torr 200 200 400 400 600 600
Problems
t/s 100 200 100 200 100 200 P17C.1 Show by differentiation that eqn 17C.4 is a solution of eqn 17C.3. for the relaxation time in terms of the total concentration of A, [A]tot = [A] +
(a) Is the decay of the concentration of the drug first- or second-order in the 2[A2]. (b) Describe a straight-line plot you could use to determine values of
p/Torr 186 173 373 347 559 520 P17C.2 Set up the rate equations and plot the corresponding graphs for the
drug? (b) Calculate the rate constant and half-life of the process. Comment: the rate constants ka and k′a from measurements of τ for different values of
approach to an equilibrium of a reaction of the form A ! 2B (first-order
An essential aspect of drug development is the optimization of the half-life [A]tot. (c) The following data refer to the dimerization of 2-pyridone, P.
where p0 is the initial partial pressure and p is the final partial pressure of forward, second-order reverse.)
of elimination, which needs to be long enough to allow the drug to find Analyse the data to obtain values of the rate constants ka and k′,a and the
and act on its target organ but not so long that harmful side effects become cyclopropane. What is the order and rate constant for the reaction under these P17C.3 The reaction A ! B is first-order in both directions. (a) Derive an equilibrium constant K for the dimerization reaction:
important. conditions? expression for the concentration of A as a function of time when the initial
molar concentrations of A and B are [A]0 and [B]0. (b) What is the final [P]/(mol dm−3) 0.500 0.352 0.251 0.151 0.101
P17B.8 The following data have been obtained for the decomposition of P17B.14 The addition of hydrogen halides to alkenes has played a fundamental
composition of the system?
N2O5(g) at 67 °C according to the reaction 2 N2O5(g) → 4 NO2(g) + O2(g). role in the investigation of organic reaction mechanisms. In one study (M.J. τ/ns 2.3 2.7 3.3 4.0 5.3
Identify the order of the reaction with respect to N2O5 and calculate the rate Haugh and D.R. Dalton, J. Amer. Chem. Soc. 97, 5674 (1975)), high pressures P17C.4 Show that eqn 17C.8 is an expression for the overall equilibrium
constant and the half-life of N2O5. Hint: It is not necessary to obtain the result of hydrogen chloride (up to 25 atm) and propene (up to 5 atm) were examined constant in terms of the rate constants for the intermediate steps of a reaction P17C.6 The equilibrium A ! B+ C at 25 °C is subjected to a temperature
graphically; you may do a calculation by making estimates of the rates of over a range of temperatures and the amount of 2-chloropropane formed was mechanism. Hint: Begin with a mechanism containing three steps, and then jump which slightly increases the concentrations of B and C. The measured
change of concentration. determined by NMR. (a) Show that if the reaction A + B → P proceeds for a argue that your expression may be generalized for any number of steps. relaxation time is 3.0 μs. The equilibrium constant for the system is 2.0 × 10−16
short time δt, the concentration of product follows [P]/[A] = kr[A]m−1[B]nδt if at the new temperature, and the equilibrium concentrations of B and C then
P17C.5 Consider the dimerization 2 A ! A 2 , with forward rate constant ka and
t/min 0 1 2 3 4 5 the reaction is mth-order in A and nth-order in B. (b) In a series of runs the are both 0.20 mmol dm−3. Calculate the rate constants for the forward and
reverse rate constant k′;a the forward step is second-order in A, and the reverse
[N2O5]/(mol dm−3) 1.000 0.705 0.497 0.349 0.246 0.173 ratio of [chloropropane] to [propene] was independent of [propene] but the reverse steps given that the forward step is first-order in A, and the reverse
step is first-order in A2. (a) Derive the expression
ratio of [chloropropane] to [HCl] for constant amounts of propene depended step is first-order in both B and C.
on [HCl]. For δt ≈ 100 h (which is short on the timescale of the reaction) the 1
P17B.9 The gas phase decomposition of ethanoic acid at 1189 K proceeds by = ka′ 2 + 8ka ka′[A]tot
latter ratio rose from zero to 0.05, 0.03, 0.01 for p(HCl) = 10 atm, 7.5 atm, τ2
way of two parallel reactions:
5.0 atm, respectively. What are the orders of the reaction with respect to each
reactant?
(1) CH3COOH → CH4 + CO2 k1 = 3.74 s−1
P17B.15 (a) Show that t1/2 is given by eqn 17B.6 for a reaction that is nth order
(2) CH3COOH → CH2CO + H2O k2 = 4.65 s−1
in A. (b) Derive an expression for the time it takes for the concentration of a
substance to fall to one-third the initial value in an nth-order reaction.
TOPIC 17D The Arrhenius equation
(a) What is the maximum theoretical yield of the ketene CH2CO at this
temperature? (b) Does the ratio of ketene to methane vary over time? P17B.16 Derive an integrated expression for a second-order rate law v = Discussion questions
P17B.10 Sucrose is readily hydrolysed to glucose and fructose in acidic
kr[A][B] for a reaction of stoichiometry 2 A + 3 B → P, with [P]0 = 0. Express
your rate law in terms of [A]0, [B]0, and x, where [A] = [A]0 − 2x. D17D.1 Define the terms in ln kr = ln A − Ea/RT and discuss the conditions D17D.2 What might account for the failure of the Arrhenius equation to fit
solution. The hydrolysis can be monitored by measuring the angle of
under which the expression is valid. experimental data at low temperatures?
rotation of plane-polarized light passing through the solution because the 2
P17B.17 Derive the integrated form of a third-order rate law v = kr[A] [B] in
concentration of sucrose can be inferred from this angle. An experiment on which the stoichiometry is 2 A + B → P and the reactants are initially present
the hydrolysis of sucrose in 0.50 m HCl(aq) produced the following data: in (a) their stoichiometric proportions ([B]0 = 12 [A]0); (b) with B present
initially in twice that amount ([B]0 = [A]0). Express your rate law in terms of
Exercises
t/min 0 14 39 60 80 110 140 170 210
[A]0, [B]0, and x, where [A] = [A]0 − 2x. E17D.1(a) Calculate the rate constant at 500 K for the second-order gas-phase E17D.2(b) The rate constant for the decomposition of a certain substance is
[sucrose]/ 0.316 0.300 0.274 0.256 0.238 0.211 0.190 0.170 0.146 reaction between Cl and H2 given the frequency factor, A = 8.1 × 10−10 2.25 × 10−2 dm3 mol−1 s−1 at 29 °C and 4.01 × 10−2 dm3 mol−1 s−1 at 37 °C. Evaluate
(mol dm−3) P17B.18 Show that the ratio t1/2/t3/4, where t1/2 is the half-life and t3/4 is the time
dm3 mol−1 s−1 and activation energy Ea = 23 kJ mol–1. the Arrhenius parameters of the reaction.
for the concentration of A to decrease to 43 of its initial value (implying that
E17D.1(b) The Arrhenius parameters for the gas-phase decomposition of
Assume that the reaction is first-order in sucrose, and determine the rate t3/4 < t1/2), can be written as a function of n alone, and can therefore be used as E17D.3(a) The rate constant of a chemical reaction is found to triple
cyclobutane, C4H8(g) → 2 C2H4(g), are A = 4.00 × 1015 s−1 and Ea = 261 kJ mol−1.
constant of the reaction and the half-life of sucrose. a rapid assessment of the order of a reaction. when the temperature is raised from 24 °C to 49 °C. Evaluate the activation
What is the half-life of cyclobutane at (i) 20 °C, (ii) 500 °C?
energy.
E17D.2(a) The rate constant for the decomposition of a certain substance is E17D.3(b) The rate constant of a chemical reaction is found to double
−3 3 −1 −1 −2 3 −1 −1
3.80 × 10 dm mol s at 35 °C and 2.67 × 10 dm mol s at 50 °C. Evaluate when the temperature is raised from 25 °C to 35 °C. Evaluate the activation
the Arrhenius parameters of the reaction. energy.
Exercises and problems 773 774 17 Chemical kinetics

E17D.4(a) The activation energy of one of the reactions in a biochemical E17D.5(a) At what temperature does the fraction of molecular collisions E17E.2(a) The following mechanism has been proposed for the decomposition Show that if it can be assumed that there is a pre-equilibrium involving step
process is 87 kJ mol−1. What is the change in rate constant when the with enough energy to result in a bimolecular reaction reach 0.10 if Ea = of ozone in the atmosphere: (1), the rate of formation of ethylene oxide is v = k2K[CH2ClCH2OH][OH−],
temperature falls from 37 °C to 15 °C? 50 kJ mol−1? where K is the equilibrium constant for the first step and k2 is the rate constant
O3 → O2 + O ka
E17D.4(b) The activation energy for the decomposition of benzene diazonium E17D.5(b) At 500 K, what is the fraction of molecular collisions with enough for the second step.
chloride is 99.1 kJ mol–1. At what temperature is the rate constant 10 per cent energy to result in a bimolecular reaction with Ea = 80 kJ mol−1? O2 + O → O3 k′a
E17E.3(a) The mechanism of a reaction consists of a pre-equilibrium step
greater than at 25 °C? O + O3 → O2 + O2 kb with forward and reverse activation energies of 25 kJ mol−1 and 38 kJ mol−1,
Show that if the third step is rate limiting, then the rate law for the respectively, followed by a rate-limiting elementary step of activation energy
decomposition of O3 is second-order in O3 and of order −1 in O2. 10 kJ mol−1. What is the activation energy of the overall reaction?
Problems E17E.2(b) The mechanism for the reaction between 2-chloroethanol, E17E.3(b) The mechanism of a reaction consists of a pre-equilibrium step
P17D.1 Show that the definition of Ea given in eqn 17D.3 reduces to eqn 17D.1 P17D.5‡ Methane is a by-product of a number of natural processes (such as CH2ClCH2OH, and hydroxide ions in aqueous solution to form ethylene with forward and reverse activation energies of 27 kJ mol−1 and 35 kJ mol−1,
for a temperature-independent activation energy. digestion of cellulose in ruminant animals, and anaerobic decomposition of oxide, (CH2CH2)O, is thought to consist of the steps respectively, followed by a rate-limiting elementary step of activation energy
organic waste matter), and industrial processes (such as food production and 15 kJ mol−1. What is the activation energy of the overall reaction?
P17D.2 A first-order decomposition reaction is observed to have the (1) CH2ClCH2OH + OH− ! CH2ClCH2O− + H2O
fossil fuel use). Reaction with the hydroxyl radical OH is the main path by
following rate constants at the indicated temperatures. Estimate the (2) CH2ClCH2O− → (CH2CH2)O + Cl−
which CH4 is removed from the lower atmosphere. T. Gierczak et al. (J. Phys.
activation energy.
Chem. A 101, 3125 (1997)) measured the rate constants for the elementary
−3 −1
kr/(10 s ) 2.46 45.1 576 bimolecular gas-phase reaction of methane with the hydroxyl radical over a
range of temperatures of importance to atmospheric chemistry. Deduce the Problems
θ/°C 0 20.0 40.0 Arrhenius parameters A and Ea from the following measurements:
P17E.1 Use mathematical software or a spreadsheet to examine the time P17E.7 Show that the following mechanism can account for the rate law of the
P17D.3 The rate constant for the gas-phase reaction of ethene and hydrogen, T/K 295 223 218 213 206 200 195 dependence of [I] in the reaction mechanism A k a
→ I k b
→ P. In all the reaction in Problem P17B.14 (the final step is rate determining):
C2H4(g) + H2(g) → C2H6(g), was measured at different temperatures. Use the following calculations, use [A]0 = 1 mol dm−3 and a time range of 0–5 s. (a) Plot
kr/(106 dm3 mol−1 s−1) 3.55 0.494 0.452 0.379 0.295 0.241 0.217 HCl + HCl ! (HCl)2 K1
following values to calculate the Arrhenius parameters. [I] against t for ka = 10 s−1 and kb = 1 s−1. (b) Increase the ratio kb/ka steadily by
decreasing the value of ka and examine the plot of [I] against t at each turn. HCl + CH3CH=CH2 ! complex K2
P17D.6‡ As described in Problem P17D.5, reaction with the hydroxyl radical What approximation about d[I]/dt becomes increasingly valid? (HCl)2 + complex → CH3CHClCH3 + HCl + HCl kr
T/K 1000 1200 1400 1600
OH is the main path by which CH4 is removed from the lower atmosphere.
kr/(dm3 mol−1 s−1) 8.35 × 10−10 3.08 × 10−8 4.06 × 10−7 2.80 × 10−6 T. Gierczak et al. (J. Phys. Chem. A 101, 3125 (1997)) measured the rate P17E.2 Use mathematical software or a spreadsheet to investigate the effects What further tests could you apply to verify this mechanism?
constants for the bimolecular gas-phase reaction CH4 + OH → CH3 + H2O on [A], [I], [P], and tmax of decreasing the ratio ka/kb from 10 (as in Fig. 17E.1)
P17E.8 Polypeptides are polymers of amino acids. Suppose that a long
P17D.4 The second-order rate constants for the reaction of oxygen atoms with and found A = 1.13 × 109 dm3 mol−1 s−1 and Ea = 14.1 kJ mol−1 for the Arrhenius to 0.01.
polypeptide chain can undergo a transition from a helical conformation to a
aromatic hydrocarbons have been measured (R. Atkinson and J.N. Pitts, J. parameters. (a) Estimate the rate of consumption of CH4 under the following P17E.3 Two radioactive nuclides decay by successive first-order processes: random coil. Consider a mechanism for a helix–coil transition that begins in
Phys. Chem. 79, 295 (1975)). In the reaction with benzene the rate constants conditions: take the average OH concentration to be 3.5 × 10−15 mol dm−3, that X 22.5
 d
→ Y 33.0
 d
→ Z (the quantities over the arrows are the half-lives in the middle of the chain:
are 1.44 × 107 dm3 mol−1 s−1 at 300.3 K, 3.03 × 107 dm3 mol−1 s−1 at 341.2 K, and of CH4 to be 40 nmol dm−3, and the temperature to be −10 °C. (b) Estimate the days). Suppose that Y is an isotope that is required for medical applications.
6.9 × 107 dm3 mol−1 s−1 at 392.2 K. Find the frequency factor and activation global annual mass of CH4 consumed by this reaction (which is slightly less hhhh… ! hchh…
At what time after X is first formed will Y be most abundant?
energy of the reaction. than the mass introduced to the atmosphere) given an effective volume for the hchh… ! cccc…
Earth’s lower atmosphere of 4 × 1021 dm3. P17E.4 Set up the rate equations for the reaction mechanism:
ka kb
in which h and c label, respectively, an amino acid in a helical or coil part of the

A← 
→ B←

→C chain. The first conversion from h to c, also called a nucleation step, is relatively
k′a k′ b
slow, so neither step may be rate determining. (a) Set up the rate equations for
Show that, under specific circumstances which you should identify, the
this mechanism. (b) Apply the steady-state approximation and show that, under
mechanism is equivalent to
these circumstances, the mechanism is equivalent to hhhh… ! cccc….
kr

A← 
→C P17F.9‡ J. Czarnowski and H.J. Schuhmacher (Chem. Phys. Lett. 17, 235
TOPIC 17E Reaction mechanisms k′r

P17E.5 Derive an equation for the steady-state rate of the sequence of reactions
(1972)) suggested the following mechanism for the thermal decomposition of
F2O in the reaction 2 F2O(g) → 2 F2(g) + O2(g):
A ! B ! C ! D, with [A] maintained at a fixed value and the product D
Discussion questions removed as soon as it is formed. (1) F2O + F2O → F + OF + F2O ka
(2) F + F2O → F2 + OF kb
D17E.1 Distinguish between reaction order and molecularity. D17E.4 Explain and illustrate how reaction orders may change under different P17E.6 The oxidation of NO to NO2, 2 NO(g) + O2(g) → 2 NO2(g), proceeds by
circumstances. the following mechanism: (3) OF + OF → O2 + F + F kc
D17E.2 Comment on the validity of the statement that the rate-determining
step is the slowest step in a reaction mechanism. D17E.5 Distinguish between kinetic and thermodynamic control of a reaction. (4) F + F + F2O → F2 + F2O kd
NO + NO → N2O2 ka
Suggest criteria for expecting one rather than the other. N2O2 → NO + NO k′a Use the steady-state approximation to show that this mechanism is consistent
D17E.3 Distinguish between the pre-equilibrium approximation and the
steady-state approximation. Why might they lead to different conclusions? D17E.6 Explain how it is possible for the activation energy of a reaction to be with the experimental rate law −d[F2O]/dt = kr[F2O]2 + k′[F 3/2
2O] .
N2O2 + O2 → NO2 + NO2 kb r

negative. P17E.10 Consider two products formed from reactant R in reactions for
Verify that application of the steady-state approximation to the intermediate which: (a) product P1 is thermodynamically more stable than product P2; and
N2O2 results in the rate law (b) the activation energy Ea for the reaction leading to P2 is greater than that
Exercises d[NO2 ] 2ka kb[NO]2[O2 ] leading to P1. Derive an expression for the ratio [P2]/[P1] when the reaction is
= under thermodynamic control. State your assumptions.
E17E.1(a) The reaction mechanism for the decomposition of A2 is thought to E17E.1(b) The reaction mechanism for renaturation of a double helix from its dt ka′ + kb[O2 ]
be strands A and B is thought to be

k ka

A2 ← 
a
→ A+A A + B 
bk
→P 
A+B ← 
→U
 U 
b
→H
k
ka′ ka′

where the dissociation of A2 is first order in A2, and the recombination of A where U is an unstable helix, and H is the stable form of the helix. The reaction
TOPIC 17F Examples of reaction mechanisms
is second order in A; the reaction of A with B is first order in both A and B. between A and B is first order in each species and the return of U to A + B is
Deduce the rate law for the rate of formation of P in two ways: (i) by assuming first order in U; the reaction of U to H is first order in U. Deduce the rate law Discussion questions
a pre-equilibrium between A2 and A, and (ii) by assuming that the steady- for the rate of formation of H in two ways: (i) by assuming a pre-equilibrium
D17F.1 Discuss the conditions under which the expression kr = kakb[A]/(kb + the Lindemann–Hinshelwood mechanism results in a (a) first-order, or (b)
state approximation can be applied to A. and (ii) by assuming that the steady-state approximation can be applied to U.
k′[A])
a for the effective rate constant of a unimolecular reaction according to second-order rate law.

Exercises and problems 775 776 17 Chemical kinetics

D17F.2 Bearing in mind distinctions between the mechanisms of stepwise and D17F.4 A plot of the rate of an enzyme-catalysed reaction against temperature
chain polymerization, describe how it is possible to control the molar mass of
a polymer by manipulating the kinetic parameters of polymerization.
has a maximum, in an apparent deviation from the behaviour predicted by the
Arrhenius equation (Topic 17D). Suggest an interpretation.
TOPIC 17G Photochemistry
D17F.3 Discuss the features, advantages, and limitations of the Michaelis–
Menten mechanism of enzyme action.
Discussion question
D17G.1 Consult literature sources and list the observed ranges of timescales proton transfer reactions, energy transfer between fluorescent molecules used
during which the following processes occur: radiative decay of excited in FRET analysis, electron transfer events between complex ions in solution,
Exercises electronic states, molecular rotational motion, molecular vibrational motion, and collisions in liquids.

E17F.1(a) The effective rate constant for a gaseous reaction which proceeds E17F.3(b) Consider a polymer formed by a chain process. By how much does the
by a Lindemann–Hinshelwood mechanism is 2.50 × 10−4 s−1 at 1.30 kPa and kinetic chain length change if the concentration of initiator is decreased by a
2.10 × 10−5 s−1 at 12 Pa. Calculate the rate constant for the activation step in the factor of 10.0 and the concentration of monomer is increased by a factor of 5.0?
Exercises
mechanism. E17G.1(a) In a photochemical reaction A → 2 B + C, the quantum yield concentration of quencher required to decrease the fluorescence intensity of
E17F.4(a) The enzyme-catalysed conversion of a substrate at 25 °C has a
E17F.1(b) The effective rate constant for a gaseous reaction which proceeds −3 with 500 nm light is 210 mmol einstein−1 (1 einstein = 1 mol photons). After the organic species to 50 per cent of the unquenched value.
Michaelis constant of 0.046 mol dm . The rate of the reaction is 1.04 mmol
by a Lindemann–Hinshelwood mechanism is 1.7 × 10−3 s−1 at 1.09 kPa and exposure of 300 mmol of A to the light, 2.28 mmol of B is formed. How many E17G.3(b) Consider the quenching of an organic fluorescent species with
dm−3 s−1 when the substrate concentration is 0.105 mol dm−3. What is the
2.2 × 10−4 s−1 at 25 Pa. Calculate the rate constant for the activation step in the photons were absorbed by A? τ0 = 3.5 ns by a d-metal ion with kQ = 2.5 × 109 dm3 mol−1 s−1. Predict the
maximum velocity of this reaction?
mechanism. E17G.1(b) In a photochemical reaction A → B + C, the quantum yield with concentration of quencher required to decrease the fluorescence intensity of
E17F.4(b) The enzyme-catalysed conversion of a substrate at 25 °C has a
E17F.2(a) Calculate the fraction condensed and the degree of polymerization at Michaelis constant of 0.032 mol dm−3. The rate of the reaction is 0.205 mmol 500 nm light is 120 mmol einstein−1 (1 einstein = 1 mol photons). After the organic species to 75 per cent of the unquenched value.
t = 5.00 h of a polymer formed by a stepwise process with kr = 1.39 dm3 mol−1 s−1 dm−3 s−1 when the substrate concentration is 0.875 mol dm−3. What is the exposure of 200 mmol A to the light, 1.77 mmol B is formed. How many
E17G.4(a) An amino acid on the surface of a protein was labelled covalently
and an initial monomer concentration of 10.0 mmol dm−3. maximum velocity of this reaction? photons were absorbed by A?
with 1.5-I AEDANS and another was labelled covalently with FITC. The
E17F.2(b) Calculate the fraction condensed and the degree of polymerization at E17G.2(a) A substance has a fluorescence quantum yield of ϕF,0 = 0.35. In an fluorescence quantum yield of 1.5-I AEDANS decreased by 10 per cent due to
E17F.5(a) The ratio kb/KM is called the catalytic efficiency of an enzyme.
t = 10.00 h of a polymer formed by a stepwise process with kr = 2.80 × 10−2 dm3 experiment to measure the fluorescence lifetime of this substance, it was quenching by FITC. What is the distance between the amino acids? (Refer to
Calculate the catalytic efficiency of carbonic anhydrase by using the data in
mol−1 s−1 and an initial monomer concentration of 50.0 mmol dm−3. observed that the fluorescence emission decayed with a half-life of 5.6 ns. Table 17G.3 for the appropriate value of R0.)
Example 17F.2.
E17F.3(a) Consider a polymer formed by a chain process. By how much does E17F.5(b) The enzyme-catalysed conversion of a substrate at 298 K has KM =
What is the fluorescence rate constant of this substance? E17G.4(b) An amino acid on the surface of an enzyme was labelled covalently
E17G.2(b) A substance has a fluorescence quantum yield of ϕF,0 = 0.16. In an with 1.5-I AEDANS and it is known that the active site contains a tryptophan
the kinetic chain length change if the concentration of initiator is increased 0.032 mol dm−3 and vmax = 4.25 × 10−4 mol dm−3 s−1 when the enzyme
by a factor of 3.6 and the concentration of monomer is decreased by a factor concentration is 3.60 × 10−9 mol dm−3. Calculate the catalytic efficiency of the experiment to measure the fluorescence lifetime of this substance, it was residue. The fluorescence quantum yield of tryptophan decreased by 15 per
of 4.2? enzyme, as defined in Exercise E17F.5(a). observed that the fluorescence emission decayed with a half-life of 1.5 ns. cent due to quenching by 1.5-I AEDANS. What is the distance between the
What is the fluorescence rate constant of this substance? active site and the surface of the enzyme?
E17G.3(a) Consider the quenching of an organic fluorescent species with
Problems τ0 = 6.0 ns by a d-metal ion with kQ = 3.0 × 108 dm3 mol−1 s−1. Predict the

P17F.1 The isomerization of cyclopropane over a limited pressure range was ES → ES′ v = kb[ES]
examined in Problem 17B.13. If the Lindemann–Hinshelwood mechanism
of unimolecular reactions is to be tested data is also needed at low pressures.
ES′ → E + P v = kc[ES′] Problems
This information has been obtained (H.O. Pritchard et al., Proc. R. Soc. A 217, P17G.1 In an experiment to measure the quantum yield of a photochemical with amine concentration as shown below. A time-resolved laser spectroscopy
Show that the rate of formation of product has the same form as that shown in
563 (1953)): reaction, the absorbing substance was exposed to 320 nm radiation from an experiment had also shown that the half-life of the phosphorescence in the
eqn 17F.16, but with vmax and KM given by
87.5 W source for 28.0 min. The intensity of the transmitted radiation was absence of quencher is 29 μs. What is the value of kQ?
p/Torr 84.1 11.0 2.89 0.569 0.120 0.067 0.257 that of the incident radiation. As a result of irradiation, 0.324 mol of the
kbkc[E]0 kc (ka′ + kb )
104 kr/s−1 2.98 2.23 1.54 0.857 0.392 0.303 vmax = KM = absorbing substance decomposed. Evaluate the quantum yield. [Q]/(mmol dm−3) 1.0 5.0 10.0
kb + kc ka (kb + kc )
P17G.2‡ Ultraviolet radiation photolyses O3 to O2 and O. Determine the rate at Ip/(arbitrary units) 0.41 0.25 0.16
Test the Lindemann–Hinshelwood mechanism with these data. P17F.7 The following results were obtained for the action of an ATPase on ATP which ozone is consumed by 305 nm radiation in a layer of the stratosphere of
P17G.5 An electronically excited state of Hg can be quenched by N2 according
P17F.2 Calculate the average polymer length in a polymer produced by a chain at 20 °C, when the concentration of the ATPase was 20 nmol dm−3: thickness 1.0 km. The quantum yield is 0.94 at 220 K, the concentration about
to Hg*(g) + N2(g, v = 0) → Hg(g) + N2(g, v = 1) in which energy transfer
mechanism in which termination occurs by a disproportionation reaction of 8 nmol dm−3, the molar absorption coefficient 260 dm3 mol−1 cm−1, and the flux
[ATP]/(μmol dm−3) 0.60 0.80 1.4 2.0 3.0 from Hg* excites N2 vibrationally. The data below give the measured time
the form ⋅HMn + ⋅Mm → Mn + HMm. of 305 nm radiation about 1 × 1014 photons cm−2 s−1. Data from W.B. DeMore
dependence of the intensity of fluorescence for samples of Hg with and
v/(μmol dm−3 s−1) 0.81 0.97 1.30 1.47 1.69 et al. (Chemical kinetics and photochemical data for use in stratospheric
P17F.3 Derive an expression for the time dependence of the degree of without N2 present (for T = 300 K):
modeling: Evaluation Number 11, JPL Publication 94–26 (1994)).
polymerization for the stepwise polymerization of a hydroxyacid HO−R−
Evaluate the Michaelis constant and the maximum velocity of the reaction. P17G.3 Dansyl chloride, which absorbs maximally at 330 nm and fluoresces pN2 = 0
COOH for which the rate law is d[A]/dt = −kr[A]2[OH], where A denotes the
carboxylic acid group. P17F.8 There are different ways to represent and analyse data for enzyme- maximally at 510 nm, can be used to label amino acids in fluorescence Relative fluorescence intensity 1.000 0.606 0.360 0.22 0.135
catalysed reactions. The text shows how to construct a linear Lineweaver– microscopy and FRET studies. Tabulated below is the variation of the
P17F.4 Michaelis and Menten derived their rate law by assuming a rapid pre- fluorescence intensity of an aqueous solution of dansyl chloride with time t/μs 0.0 5.0 10.0 15.0 20.0
equilibrium of E, S, and ES. Derive the rate law in this manner, and identify Burk plot of 1/v against 1/[S]0. (a) Show, by rearranging eqn 17F.16, that an
Eadie–Hofstee plot of v/[S]0 against v is also expected to be a straight line. after excitation by a short laser pulse (with I0 the initial fluorescence intensity).
the conditions under which it becomes the same as that based on the steady- pN2 = 9.74 ×10−4 atm
Identify how the Michaelis constant and the maximum velocity of the reaction The ratio of intensities is equal to the ratio of the rates of photon emission.
state approximation (eqn 17F.16). Relative fluorescence intensity 1.000 0.585 0.342 0.200 0.117
may be obtained from such a plot. (b) In the same way, show that a Hanes
P17F.5 Use the Michaelis–Menten equation (eqn 17F.16) to generate two t/ns 5.0 10.0 15.0 20.0
plot of v/[S]0 against [S]0 is also a straight line. Identify how the parameters t/μs 0.0 3.0 6.0 9.0 12.0
families of curves showing the dependence of v on [S]: one in which KM varies may be obtained from such a plot. (c) The enzyme catalase, catalyses the If/I0 0.45 0.21 0.11 0.05
but vmax is constant, and another in which vmax varies but KM is constant. Hint: decomposition of hydrogen peroxide, H2O2. By constructing Lineweaver– Evaluate the rate constant for the energy transfer process. You may assume
Use mathematical software or a spreadsheet. Burk, Eadie–Hofstee, and Hanes plots, use the following values for the rate of (a) Calculate the observed fluorescence lifetime of dansyl chloride in water. that all gases are perfect.
P17F.6 For many enzymes, the mechanism of action involves the formation of
reaction for various initial concentrations of hydrogen peroxide to calculate (b) The fluorescence quantum yield of dansyl chloride in water is 0.70. What
P17G.6 The Förster theory of resonance energy transfer and the basis for the
two intermediates: the Michaelis constant and the maximum velocity of the reaction. is the fluorescence rate constant?
FRET technique can be tested by performing fluorescence measurements on
P17G.4 When benzophenone is exposed to ultraviolet radiation it is a series of compounds in which an energy donor and an energy acceptor are
E + S → ES v = ka[E][S] [H2O2]/(mol dm−3) 0.300 0.400 0.500 0.600 0.700 excited into a singlet state. This singlet changes rapidly into a triplet, which covalently linked by a rigid molecular linker of variable and known length. L.
ES → E + S v = ka′ [ES]
−3 −1
v/(mol dm s ) 4.431 4.518 4.571 4.608 4.634 phosphoresces. Triethylamine acts as a quencher for the triplet. In an Stryer and R.P. Haugland (Proc. Natl. Acad. Sci. USA 58, 719 (1967)) collected
experiment in the solvent methanol, the phosphorescence intensity varied the following data on energy transfer efficiencies, ηT, for a family of compounds
Exercises and problems 777

with the general composition dansyl-(l-prolyl)n-naphthyl, in which the distance P17G.7 The first step in plant photosynthesis is absorption of light by
R between the naphthyl donor and the dansyl acceptor was varied from 1.2 nm chlorophyll molecules bound to proteins known as ‘light-harvesting
to 4.6 nm by increasing the number of prolyl units in the linker: complexes’, where the fluorescence of a chlorophyll molecule is quenched by
other nearby chlorophyll molecules. Given that for a pair of chlorophyll a
R/nm 1.2 1.5 1.8 2.8 3.1 3.4 3.7 4.0 4.3 4.6
molecules R0 = 5.6 nm, by what distance should two chlorophyll a molecules
ηT 0.99 0.94 0.97 0.82 0.74 0.65 0.40 0.28 0.24 0.16 be separated to shorten the fluorescence lifetime from 1 ns (a typical value for
monomeric chlorophyll a in organic solvents) to 10 ps?
Are the data described adequately by eqn 17G.7? If so, what is the value of R0
for the naphthyl–dansyl pair?

FOCUS 17 Chemical kinetics


Integrated activities
I17.1 Autocatalysis is the catalysis of a reaction by the products. For example, Which are the autocatalytic steps of this mechanism? Find the conditions
for a reaction A → P it may be found that the rate law is v = kr[A][P] and on the ratio a/r that decide whether the disease will spread (an epidemic)
the reaction rate is proportional to the concentration of P. The reaction gets or die out. Show that a constant population is built into this system, namely
started because there are usually other reaction routes for the formation of that S + I + R = N, meaning that the timescales of births, deaths by other
some P initially, which then takes part in the autocatalytic reaction proper. causes, and migration are assumed large compared to that of the spread of
(a) Integrate the rate equation for an autocatalytic reaction of the form A → P, the disease.
with rate law v = kr[A][P], and show that
I17.3 Acid- and base-catalysed reactions are common in organic
[P] (1 + b )eat transformations. (a) Deduce the rate law of the base-catalysed reaction in
= which AH goes to products according to the following scheme
[P]0 1 + beat
ka

AH + B ← 
→ BH+ + A−
where [P]0 is the initial concentration of P, a = ([A]0 + [P]0)kr and b = k′ a

[P]0/[A]0. Hint: Start from the expression v = −d[A]/dt = kr[A][P], write A− + AH k
b
→ product (rate-determining)
[A] = [A]0 − x, [P] = [P]0 + x, and then write the expression for the rate of
change of either species in terms of x. To integrate the resulting expression, (b) Deduce the rate law of the acid-catalysed reaction in which HA goes to
use integration by the method of partial fractions (see The chemist’s toolkit 30 products according to the following scheme
in Topic 17B). (b) Plot [P]/[P]0 against at for several values of b. Discuss the ka
effect of autocatalysis on the shape of a plot of [P]/[P]0 against t by comparing 
HA + H+ ← 
→ HAH+
k′ a
your results with those for a first-order process, in which [P]/[P]0 = 1 − e − krt. HAH+ + B 
b k
→ BH+ + AH (rate-determining)
(c) Show that for the autocatalytic process discussed in parts (a) and (b), the
reaction rate reaches a maximum at tmax = −(1/a) ln b. (d) An autocatalytic 2 2 1/2
I17.4 Express the root-mean-square deviation {〈M 〉 − 〈M〉 } of the molar
reaction A → P is observed to have the rate law d[P]/dt = kr[A]2[P]. Solve the mass of a condensation polymer in terms of the fraction p, and deduce its
rate law for initial concentrations [A]0 and [P]0. Calculate the time at which time dependence.
the rate reaches a maximum. (e) Another reaction with the stoichiometry
A → P has the rate law d[P]/dt = kr[A][P]2; integrate the rate law for initial I17.5 Calculate the ratio of the mean cube molar mass to the mean square
concentrations [A]0 and [P]0. Calculate the time at which the rate reaches a molar mass in terms of (a) the fraction p, (b) the chain length.
maximum. I17.6 Conventional equilibrium considerations do not apply when a reaction
I17.2 Many biological and biochemical processes involve autocatalytic is driven by light absorption and the steady-state concentration of products
steps (see Integrated activity I17.1). In the SIR model of the spread and and reactants might differ significantly from equilibrium values. For instance,
decline of infectious diseases the population is divided into three classes; suppose the reaction A → B is driven by light absorption, and that its rate is
the ‘susceptibles’, S, who can catch the disease, the ‘infectives’, I, who have Ia, but that the reverse reaction B → A is bimolecular and second order with
the disease and can transmit it, and the ‘removed class’, R, who have either a rate kr[B]2. What is the stationary state concentration of B? Why does this
had the disease and recovered, are dead, are immune or isolated. The model ‘photostationary state’ differ from the equilibrium state?
mechanism for this process, written as S → I → R, implies the following rate I17.7 The photochemical chlorination of trichloromethane (chloroform,
laws: CHCl3) in the gas to give CCl4 has been found to follow the rate law
dS dI dR d[CCl4]/dt = kr[Cl2]1/2Ia1/2. Devise a mechanism that leads to this rate law
= − rSI = rSI − aI = aI when the chlorine pressure is high.
dt dt dt

FOCUS 18 TOPIC 18A Collision theory

Reaction dynamics quency of collisions, and therefore to the mean speed of the
➤ Why do you need to know this material? molecules, vmean ∝ (T/M)1/2 where M is some combination of
the molar masses of A and B. The rate can also be expected to
A major component of chemistry is the study of the
be proportional to the target area the molecules present, which
detailed molecular mechanisms of chemical reactions.
is their collision cross-section, σ (Topic 1B), and to the num-
One of the earliest approaches, which continues to give
ber densities NA and NB of A and B:
insight into the details of mechanisms of gas-phase reac-
This Focus examines the details of what happens to molecules at 18C Transition-state theory tions, is collision theory.
the climax of reactions. Extensive changes of structure are tak- NJ ∝ [J]
ing place and energies the size of dissociation energies are being This Topic discusses ‘transition-state theory’, in which it is as- ➤ What is the key idea? v ∝ σ(T/M)1/2NANB ∝ σ(T/M)1/2[A][B]
redistributed among bonds: old bonds are being ripped apart sumed that the reactant molecules form a complex that can be
According to collision theory a bimolecular gas-phase
and new bonds are being formed. This is the heart of chemistry. discussed in terms of the population of its energy levels. The However, a collision is likely to be successful only if the kinetic
reaction takes place when reactants collide, provided their
The calculation of the rates of such processes from first prin- theory inspires a thermodynamic approach to reaction rates, energy of the molecules exceeds a minimum value, denoted E′.
relative kinetic energy exceeds a threshold value and cer-
ciples is very difficult. Nevertheless, like so many intricate prob- in which the rate constant is expressed in terms of thermody- This requirement suggests that the rate should also be propor-
tain steric requirements are fulfilled.
lems, the broad features can be established quite simply. Only namic parameters. This approach is useful for parametrizing tional to a Boltzmann factor of the form e–E′/RT representing the
upon deeper inquiry do the complications emerge. Several the rates of reactions in solution. ➤ What do you need to know already? fraction of collisions with at least the minimum required en-
approaches to the calculation of a rate constant for elementary 18C.1 The Eyring equation; 18C.2 Thermodynamic aspects; ergy (Topic 17D). Therefore,
This Topic draws on the kinetic theory of gases, especially
bimolecular processes are explored here, ranging from electron 18C.3 The kinetic isotope effect
the expression for the mean speed of molecules (Topic 1B),
transfer to chemical reactions involving bond breakage and for- v ∝ σ(T/M)1/2e–E′/RT[A][B]
and extends the account of the Lindemann–Hinshelwood
mation. Although a great deal of information can be obtained
mechanism of gas-phase reactions (Topic 17F). One argu-
from gas-phase reactions, many reactions of interest take place in and, by writing the reaction rate in the form given in eqn
condensed phases, and it is useful to attempt to predict their rates.
18D The dynamics of molecular ment draws on the Maxwell–Boltzmann distribution of
18A.1a, it follows that
molecular speeds (Topic 1B).
collisions
kr ∝ σ(T/M)1/2 e–E′/RT
The highest level of sophistication in the theoretical study of
18A Collision theory chemical reactions is in terms of potential energy surfaces and At this point, the form of the Arrhenius equation, eqn 18A.1b,
the motion of molecules on these surfaces. As explained in The rate constant of the bimolecular elementary reaction begins to emerge, with the minimum kinetic energy E′ iden-
This Topic explores ‘collision theory’, the simplest quantitative this Topic, such an approach gives an intimate picture of the tified as the activation energy Ea of the reaction. This identi-
A+B→P v = kr[A][B] (18A.1a)
account of reaction rates. The treatment can be used only for events that occur when molecules collide, and provides a basis fication, however, should not be regarded as precise, because
the discussion of reactions between simple species in the gas for studying them by using molecular beams. depends on the temperature according to the Arrhenius ex- collision theory is only a rudimentary model of chemical re-
phase. Basic collision theory considers only the impact of one 18D.1 Molecular beams; 18D.2 Reactive collisions; 18D.3 Potential pression (Topic 17D): activity.
molecule on another. An elaboration considered in this Topic energy surfaces; 18D.4 Some results from experiments and Not every collision will lead to reaction even if the energy
calculations kr = Ae − E /RT
a
Arrhenius expression (18A.1b)
takes into account how the resulting excitation energy accu- requirement is satisfied, because the reactants might need to
mulates in the bond where it is needed. where A is the ‘frequency factor’ and Ea is the ‘activation en- collide in a certain relative orientation. This ‘steric require-
18A.1 Reactive encounters; 18A.2 The RRK model ergy’. This form of the Arrhenius expression can be explained ment’ suggests that a further factor, P, should be introduced,
by a model in which molecules in the gas collide and in the and that
18E Electron transfer in homogeneous process may acquire sufficient energy to undergo reaction.
kr ∝ Pσ(T/M)1/2 e–E′/RT (18A.2)
systems Like all models, this one can be improved, but it is a good
18B Diffusion-controlled reactions starting point for the discussion of gas-phase reactions. As seen in detail below, this expression has the form predicted
In this Topic transition-state theory is used to examine the by collision theory. It reflects three aspects of a successful col-
Reactions in solution are classified into two types: ‘diffusion- transfer of electrons in homogeneous systems, which include lision:
controlled’ where the rate is controlled by the frequency with oxidation–reduction reactions in solution. One widely used
which reactants meet, and ‘activation-controlled’, where the theory, Marcus theory, establishes a relation between the ac- 18A.1 Reactive encounters Steric Encounter
Minimum
energy
accumulation of sufficient energy in a pair that have met is rate- tivation parameters and the rate constant of electron transfer, requirement rate
requirement
determining. The rate constants for the former can be expressed and can be expressed in terms of structural parameters of the The general form of the expression for kr in eqn 18A.1a can be
quantitatively in terms of the diffusional characteristics of species species involved. anticipated by considering the physical requirements for reac- % !#"#$ %
in liquids. A more detailed account of the space- and time-devel- 18E.1 The rate law; 18E.2 The role of electron tunnelling; 18E.3 The tion. The rate v can be expected to be proportional to the fre- kr ∝ P σ(T / M )1/2 e − E ′/RT
opment of products is obtained by using the diffusion equation. rate constant; 18E.4 Experimental tests of the theory
18B.1 Reactions in solution; 18B.2 The material-balance equation
18A Collision theory 781 782 18 Reaction dynamics

18A.4a are replaced by [J] = nJ/V = pJ/RT. For collisions be- ε in a sample, eqn 18A.5b must be averaged over a Boltzmann
(a) Collision rates in gases
tween like molecules μ = 12 mA and eqn 18A.4a becomes distribution of energies f(ε) to give d
As remarked, the reaction rate, and hence kr, is expected to de- B

{∫ σ (ε )v }
1/2
 16kT  d[A] ∞
a
pend on the frequency with which molecules collide. The col- Z AA = 12 σ  N A2[A]2 =− f (ε )dε N A[A][B] (18A.6) vrel, A–B
 πmA  dt rel
0
lision density, ZAB, is the number of (A,B) collisions in a region Collision density
1/2 (18A.4b) θ
of the sample in an interval of time divided by the volume of  4 kT  [identical molecules] where f(ε)dε is the probability that the approach energy is be- A
vrel
=σ  N A2[A]2
the region and the duration of the interval. The frequency of  πmA  tween ε and ε + dε. By comparison with eqn 18A.1a it follows
(d2 – a2)1/2
collisions of a single molecule in a gas is calculated in Topic 1B that
(eqn 1B.12a, z = σ vrel NA). That result can be adapted to derive where the (blue) factor of 12 has been introduced to avoid dou-

an expression for ZAB. ble counting of collisions (Smith with Jones and Jones with kr = N A ∫ σ (ε )vrel f (ε )dε Rate constant (18A.7) Figure 18A.2 The parameters used in the calculation of the
0
Smith, for instance). relative kinetic energy associated with the head-on component
To evaluate this integral it is necessary to establish the energy of the collision of two molecules A and B.
How is that done? 18A.1 Deriving an expression for the dependence of the collision cross-section, σ(ε).
Brief illustration 18A.1
collision density
The parameter vrel in the expression z = σ vrelN is the mean In nitrogen at 25 °C and 1.0 bar, when [N2] ≈ 40 mol m−3, with is ε = 12 µvrel
2
. You can relate these two quantities by using the
σ = 0.43 nm2 and mN2 = 28.02 mu the collision density is How is that done? 18A.2 Deriving an expression for the
relative speed of the colliding molecules and σ is the collision result from Step 2:
cross-section: σ = πd2, with d = 12 (dA + dB), as shown in Fig. 1/2 energy dependence of the collision cross-section
 4 × (1.381×10 JK ) × (298K)  −23 −1

18A.1. For collisions between A molecules of mass mA and B Z N2N2 = (4.3 ×10−19 m 2 ) × 
 π × 28.02 × (1.661×10 kg)  The key aspect of this model is that in a collision only the vrel,A−B = vrel cos θ cos2 θ = (d2 − a2)/d2
−27

molecules of mass mB, the mean relative speed is eqn 1B.11b kinetic energy associated with a head-on collision is effective ε
× (6.022 ×1023 mol −1 )2 × (40molm −3 )2 !  d 2 − a2 
(vrel = (8kT / πµ )1/2 , where µ = mAmB /(mA + mB )). It follows that at bringing about reaction. ε A−B = µv
1 2
= µ(vrel cos θ ) = 12 µvrel
1 2
2
cos 2 θ = ε 
 d 
2 rel,A–B 2 2
the collision rate of one A molecule with B molecules present at = 8.4 ×1034 m −3 s −1
number density NB is σ vrelNB. The collision density is therefore Step 1 Consider how the geometry of the collision affects the
this rate multiplied by the number density of A molecules, NA: This result shows that collision densities may be very large: energy available for reaction Step 4 Introduce an energy threshold
even in 1 cm3, there are over 8 × 1016 collisions in each pico- Consider two molecules A and B colliding with relative speed As a increases, the kinetic energy associated with the head-on
ZAB = σvrel NANB (18A.3)
second. vrel and therefore relative kinetic energy ε = 12 μ vrel
2
. Although collision decreases. The existence of an energy threshold, εa,
The number density of a species J is NJ = NA[J], where [J] is its a collision is counted when the centres of the molecules for the formation of products implies that there is a maxi-
molar concentration and NA is Avogadro’s constant. It follows come within a distance d of each other, that might be more a mum value of a, amax, above which reaction does not occur.
that glancing blow than a head-on collision. Intuitively a head-on Therefore, set a = amax and εA−B = εa and obtain
1/2 (18A.4a)
(b) The energy requirement collision between A and B will be most effective in bringing
 8kT  about a chemical reaction, so from now on suppose that only  d 2 − amax
2
  ε 
Z AB = σ  N A2[A][B]
 πµ  According to collision theory, the rate of change of NA due to εa = ε  2
the kinetic energy associated with the head-on component of which rearranges to amax =  1− a  d 2

Collision density 2
 ε
[KMT] reaction is the product of the collision density and the prob-  d
the collision leads to reaction. This contribution to the kinetic
ability that a collision occurs with sufficient energy. The latter energy depends on vrel,A−B, the magnitude of the relative veloc-
condition can be incorporated by writing the collision cross- ity component parallel to an axis connecting the centres of A Step 5 Rewrite the expression in terms of the collision cross-
section σ as a function of the kinetic energy ε of approach of and B. section
dA the two colliding species, and setting the cross-section, σ(ε), The energy-dependent collision cross-section is given in
Step 2 Find an expression for the head-on component of the
equal to zero if the kinetic energy of approach is below a cer- terms of a max as σ (ε) = πa 2max, and πd 2 is identified as the
A velocity
tain threshold value, εa. Later, NAεa will be identified as Ea, the (simple) collision cross-section σ introduced in Fig. 18A.1.
d
(molar) activation energy of the reaction. For a collision be- As shown in the arrangement in Fig. 18A.2, the distance a is It follows that
tween A and B with a specific relative speed of approach vrel the closest approach of the centres of the two molecules and
dB B d is the distance between the centres. From trigonometry and ε (18A.8)
(not, at this stage, a mean value) it follows from eqn 18A.3 that σ (ε ) =  1− a  σ
the definition of the angle θ given in the diagram, it follows  ε Energy dependence of σ(ε)
the rate of change of NA is
that [ε > εa]
Area σ
dN A
= − σ (ε )vrel N A N B (18A.5a)
Figure 18A.1 The collision cross-section for two molecules can dt cos θ = (d2 − a2)1/2/d
This form of the energy-dependence of σ(ε) is broadly
be regarded to be the area within which the projectile molecule 1/2
or, in terms of molar concentrations,  d 2 − a2  consistent with experimental determinations of the re-
(A) must enter around the target molecule (B) in order for a vrel,A−B = vrel cosθ = vrel 
 d  action between H and D2 as determined by molecular
2
collision to occur. If the diameters of the two molecules are d[A]
dA and dB, the radius of the target area is d = 21 (dA + dB) and the = − σ (ε )vrel NA[A][B] (18A.5b) beam measurements of the kind described in Topic 18D
dt Step 3 Relate the velocities to energies
cross-section is πd2. (Fig. 18A.3).
The kinetic energy associated with the relative motion of the The kinetic energy associated with the head-on collision is With the energy dependence of the collision cross-section
two particles is ε = 12 μvrel . Therefore the relative speed can also ε A−B = 12 µvrel,A–B
2
, and the total kinetic energy of the collision established, the rate constant can now be calculated.
If the collision density is required in terms of the partial be expressed in terms of the relative kinetic energy as vrel =
pressure of each gas J, then the molar concentrations in eqn (2ε/μ )1/2. Because there is a wide range of approach energies

18A Collision theory 783 784 18 Reaction dynamics

1 With σ(ε) from eqn 18A.8, write From Table 1B.1, σ(H2) = 0.27 nm2 and σ(C2H4) = 0.64 nm2,
Deflected reactant
giving σ(H2,C2H4) ≈ 0.44 nm2. The activation energy for this
molecule
0.8 1.7 × 10 pm , 1.95 eV
4 2 σ(ε) = (1−εa/ε)σ ; σ = 0 for ε < εa reaction is 180 kJ mol−1; therefore,

ε
εσ ( ε )e − ε /kT d ε = σ ∫ ε  1− a  e − ε /kT dε
∞ ∞
0.6 ∫ εa  ε kr = (4.4 ×10−19 m 2 ) × (6.022 ×1023 mol −1 ) × (2.65…×103 ms −1 )
σ(ε)/σ

0
5 −1
)/(8.3145JK −1 mol −1 )×(628K ) Products
1.0 × 104 pm2, 0.90 eV
×e − (1.80×10 Jmol
0.4 Integral E.2 Integral E.1 Area σ*
!# #"## $ !#"#$ !### #"####
A
$ Area σ
{ }
∞ ∞
0.2 = σ ∫ ε e − ε /kT d ε − ε a ∫ e − ε /kT d ε = (kT )2 σ e − ε a /kT = 7.05…×108 m3 mol −1 s −1 × e −34.5… = 7.5 ×10−7 m3 mol −1 s −1
εa εa
Figure 18A.4 The collision cross-section is the target area
0 or 7.5 × 10−4 dm3 mol−1 s −1. that results in simple deflection of the projectile molecule; the
0 5 10 15 It follows that
ε/εa reactive cross-section is the corresponding area for chemical
1/2 change to occur on collision.
∞  8kT  − ε a /kT The steric requirement
Figure 18A.3 The variation of the reactive cross-section with
energy as expressed by eqn 18A.8. The data points are from
∫0
σ (ε )vrel f (ε )d ε = σ 
 πµ 
e (c)

experiments on the reaction H + D2 → HD + D (K. Tsukiyama et al., Table 18A.1 compares some values of the frequency factor
J. Chem. Phys. 84, 1934 (1986)). Step 3 Finalize the expression for the rate constant calculated from collision cross-sections determined in other Brief illustration 18A.3
Because Ea = NAεa it follows that εa/kT = Ea/RT. With these measurements with values obtained from Arrhenius plots.
substitutions it follows from the integral just evaluated and One of the reactions shows fair agreement between theory and It is found experimentally that the frequency factor for the
eqn 18A.7 that experiment, but for others there are major discrepancies. In reaction H2 + C2H4 → C2H6 at 628 K is 1.24 × 106 dm3 mol−1 s −1.
How is that done? 18A.3 Deriving an expression for the some cases the experimental values are orders of magnitude The result in Brief illustration 18A.2 can be expressed as A =
1/2 (18A.9) smaller than those calculated, which suggests that the colli- 7.05… × 1011 dm3 mol−1 s −1. It follows that the steric factor for
rate constant  8kT  − Ea /RT
kr = σ N A  e
 πµ  Rate constant sion energy is not the only criterion for reaction and that some this reaction is
The calculation involves evaluating the integral in eqn 18A.7 [collision theory]
other feature, such as the relative orientation of the colliding
with the energy-dependent collision cross section given in Aexperimental 1.24 ×106 dm3 mol −1 s −1
species, is important. Moreover, one reaction in the table has a P= = ≈ 1.8 ×10−6
eqn 18A.8. Acalculated 7.05…×1011 dm3 mol −1 s −1
This equation has the Arrhenius form k r = Ae − E /RT provided a
pre-exponential factor larger than theory, which seems to in-
Step 1 Use the Maxwell–Boltzmann distribution to write an the exponential temperature dependence dominates the dicate that the reaction occurs more quickly than the particles The very small value of P is one reason, the other being the
expression for f (ε )dε weak square-root temperature dependence of the frequency collide! high activation energy, why catalysts are needed to bring
Adapt eqn 1B.4 in Topic 1B by replacing M/R with μ/k and so factor. It follows that, within the constraints of collision The disagreement between experiment and theory can be this reaction about at a reasonable rate. As a general guide,
writing the distribution of relative molecular speeds as theory, the activation energy, E a , can be identified with the eliminated by introducing a steric factor, P, and expressing the the more complex the reactant molecules, the smaller is the
minimum kinetic energy along the line of approach that reactive cross-section, σ*, the actual cross-section for reactive value of P.
3/2
 µ  2
2 − µvrel /2 kT is needed for reaction, and that the frequency factor (after collisions, as a multiple of the collision cross-section, σ* = Pσ
f (vrel )dvrel = 4 π  vrel e dvrel
 2 πkT  multiplication by [A][B]) determines the rate at which col- (Fig. 18A.4). Then the rate constant becomes
lisions occur.
You can write this expression in terms of the relative kinetic The simplest procedure for calculating kr is to use for σ the 1/2
An example of a reaction for which it is possible to esti-
 8kT  − E /RT
energy, ε , by noting that ε = 12 μ vrel2
. It follows that vrel = (2ε / µ )1/2 kr = PσN A  e
 πµ 
a
(18A.10)
values obtained for non-reactive collisions (e.g. typically those mate the steric factor is K + Br2 → KBr + Br, for which P =
and so dvrel = 12 (2/ µ )1/2 ε −1/2dε = dε /(2 µε )1/2 . With this substi- obtained from viscosity measurements) or from tables of mo- 4.8. In this reaction, the distance of approach at which reac-
tution the distribution becomes
lecular radii. If the collision cross-sections of A and B are σA This expression has the form anticipated in eqn 18A.2. The tion occurs appears to be considerably larger than the dis-
3/2 and σB, then an approximate value of the AB cross-section is steric factor is normally found to be several orders of magni- tance needed for deflection of the path of the approaching
 µ   2ε  − ε/kT dε
f (vrel )dvrel = 4 π   µ  e estimated from σ = πd2, with d = 12 (dA + dB). That is, tude smaller than 1. molecules in a non-reactive collision. It has been proposed
 2 πkT  (2 µε )1/2
that the reaction proceeds by a harpoon mechanism. This
 1 
3/2
σ ≈ 14 (σ A1/2 + σ B1/2 )2 brilliant name is based on a model of the reaction in which
= 2π  ε 1/2e − ε /kT dε
 πkT  the K atom is pictured as approaching a Br2 molecule, and
!####"#### $ Table 18A.1 Arrhenius parameters for gas-phase reactions*
f(ε)dε
when the two are close enough an electron (the harpoon)
A/(dm3 mol−1 s−1) Ea/(kJ mol−1) P
flips across from K to Br2. In place of two neutral particles
Brief illustration 18A.2
Step 2 Evaluate the integral there are now two ions, so there is a Coulombic attraction be-
Experiment Theory
Now evaluate the integral To estimate the rate constant for the reaction H2 + C2H4 → tween them: this attraction is the line on the harpoon. Under
2 NOCl → 9.4 × 109 5.9 × 1010 102 0.16 its influence the ions move together (the line is wound in),
C2H6 at 628 K, first calculate μ by setting m(H2) = 2.016mu and 2 NO + 2 Cl
(2ε/μ)1/2 the reaction takes place, and KBr + Br emerge. The harpoon
! 1 
3/2

1/2 m(C2H4) = 28.05mu. A straightforward calculation gives μ =
σ (ε ) vrel f (ε )dε = 2 π  σ (ε )  ε 1/2e − ε/kT dε
∞ ∞
2 ClO → Cl2 + O2 6.3 × 107 2.5 × 1010 0 2.5 × 10−3
∫0  πkT  ∫0  µ 3.123 × 10−27 kg. It then follows that
H2 + C2H4 → C2H6 1.24 × 106 7.4 × 1011 180 1.7 × 10−6
extends the cross-section for the reactive encounter, and the
1/2
reaction rate is significantly underestimated by taking for
1/2
 8   1 
1/2
∞  8kT   8 × (1.381×10−23 JK −1 ) × (628K)  −1
K + Br2 → KBr + Br 1.0 × 1012 2.1 × 1011 0 4.8 the collision cross-section the value for simple mechanical
= = 2.65…kms
=
 πµkT   kT  ∫0
εσ (ε )e − ε /kT d ε  πµ 
 π× (3.123 ×10−27 kg)  * More values are given in the Resource section. contact between K and Br2.
18A Collision theory 785 786 18 Reaction dynamics

Example 18A.1 18A.2 The RRK model


Estimating a steric factor Checklist of concepts
Estimate the value of P for the harpoon mechanism by calcu- The rate constants of ‘unimolecular’ gas-phase reactions like
lating the distance at which it becomes energetically favour- those treated by the Lindemann–Hinshelwood mechanism ☐ 1. In collision theory, it is supposed that the rate is pro- ☐ 3. The activation energy is the minimum kinetic energy
able for the electron to leap from K to Br2. Take the sum of (Topic 17F) can be estimated with a calculation based on the portional to the collision frequency, a steric factor, and along the line of approach of reactant molecules that is
the radii of the reactants (treating them as spherical) to be Rice–Ramsperger–Kassel model (RRK model). That model was the fraction of collisions that occur with at least the required for reaction.
400 pm. proposed in 1926 by O.K. Rice and H.C. Ramsperger and almost kinetic energy Ea along their lines of centres. ☐ 4. The steric factor is an adjustment that takes into
Collect your thoughts Begin by identifying all the energy simultaneously by L.S. Kassel. It has been elaborated, largely by ☐ 2. The collision density is the number of collisions in a account the orientational requirements for a successful
terms involved in the electron transfer process K + Br2 → K+ + R.A. Marcus, into the ‘RRKM model’. The essential feature of the region of the sample in an interval of time divided collision.
Br2−. There are three terms: the first is the ionization energy, model is that although a molecule might have enough energy to by the volume of the region and the duration of the ☐ 5. The rate constant for decomposition of an energized
I, of K; the second is the electron affinity, E ea, of Br2; and react, that energy is distributed over all the modes of motion of interval. molecule can be estimated by using the RRK model.
the third is the Coulombic interaction energy between the the molecule, and reaction will occur only when enough of that
ions when they have been formed. When the separation energy has migrated into a particular location (such as a particu-
of the ions is R, the Coulombic attraction energy is lar bond) in the molecule. The details are given in A deeper look 12
−e2/4πε0R. The electron flips across when the sum of these on the website of this book. Assuming that a molecule consists of
three contributions changes from positive to negative (that s identical harmonic oscillators, the principal conclusion is that
Checklist of equations
is, when the sum becomes zero) so making the process the Kassel form of the unimolecular rate constant for the decay
energetically favourable. Property Equation Comment Equation number
of the energized molecule A* to products is

( EE* )
s −1 Collision density Z AB = σ (8kT / πµ )1/2 N A2[A][B] Unlike molecules, KMT (kinetic molecular theory) 18A.4a
The solution The net change in energy when the transfer Unimolecular
occurs at a separation R is kb ( E ) = 1− kb for E ≥ E * rate constant (18A.11) Energy dependence of σ σ (ε ) = (1 − ε a / ε )σ ε ≥ εa, σ = 0 otherwise 18A.8
[Kassel form]
Rate constant kr = Pσ N A (8kT / πµ )1/2 e − Ea /RT KMT, collision theory 18A.10
2
e where k b is the rate constant used in the original Lindemann– Unimolecular rate constant kb ( E ) = (1 − E */ E )s−1 kb RRK theory ( E ≥ E *) 18A.11
E = I − Eea −
4 πε 0 R Hinshelwood theory for the decomposition of the energized
This energy is zero when R is equal to some critical value R* molecule (Topic 17F), and E* is the minimum energy that
(and is negative for smaller values of R) must be accumulated in a bond in order for it to break.
The energy dependence of the rate constant given by eqn
e2 e2 18A.11 is shown in Fig. 18A.5 for various values of s. The equa-
0 = I − Eea − rearranges to R* =
4 πε 0 R * 4 πε 0 ( I − Eea ) tion can be interpreted as follows:

When the particles are at this separation, the harpoon shoots • The rate constant is smaller at a given excitation energy
across from K to Br2. The reactive cross-section can therefore if s is large, as it takes longer for the excitation energy to
be identified as σ* = πR*2. The non-reactive collision cross- migrate through all the oscillators of a large molecule

Physical interpretation
section is σ = πd2, where d = R(K) + R(Br2) is the sum of the and accumulate in the location needed for reaction.
radii of the (assumed) spherical reactants. These values of σ • As E becomes very large, however, the term in paren-
and σ* imply that the steric factor is theses approaches 1, and k b(E) becomes independent
2
of the energy and the number of oscillators in the
σ * πR * 2  e2  molecule, as there is now enough energy to accumu-
P= = = 
σ πd 2  4 πε 0d( I − Eea )  late immediately in the critical mode regardless of
the size of the molecule.
With I = 420 kJ mol−1 (corresponding to 0.70 aJ), Eea ≈ 250 kJ
mol−1 (corresponding to 0.42 aJ), and d = 400 pm, the value of 1
P is 4.2, in good agreement with the experimental value (4.8).
0.8
Rate constant, kb(E)/kb

Self-test 18A.1 Estimate the value of P for the harpoon reac-


5
tion between Na and Cl2 for which d ≈ 350 pm; take Eea ≈ 0.6
230 kJ mol−1. 10
Answer: 2.2 0.4
20

0.2

Example 18A.1 illustrates two points about steric factors. First,


0
the concept of a steric factor is not wholly useless because in 0 10 20 30 40 50
Relative energy, E/E*
some cases its numerical value can be estimated. Second, and
more pessimistically, most reactions are much more complex Figure 18A.5 The energy dependence of the rate constant given
than K + Br2, and P cannot be obtained so easily. by eqn 18A.11 for three values of s.

788 18 Reaction dynamics

TOPIC 18B Diffusion-controlled Table 18B.1 Arrhenius parameters for solvolysis reactions in
solution
pass through a shell of radius r and surface area 4πr2 centred
on A is
Fick’s first law
Solvent A/(dm3 mol−1 s−1) Ea/(kJ mol−1)

reactions (CH3)3CCl Water


Ethanol
7.1 × 1016
3.0 × 1013
100
112
vB = 4 πr 2 J B = 4 πr 2 DB
d[B](r )
dr

Chloroform 1.4 × 10 4
45 An important point to recognize is that v B is the same for
a shell of any radius greater than or equal to R*, because no
CH3CH2Br Ethanol 4.3 × 1011 90
B molecules are lost until they have reached R*. Also keep
in mind that v B is a rate expressed as amount/time, not
concentration/time.
in solution than in a gas because the encounter pair is sur-
➤ Why do you need to know this material? rounded by solvent and the energy of the entire local assembly Step 2 Use the known values of [B] for the bulk to establish an
of reactant and solvent molecules must be considered. In this diffusion-controlled limit, the rate of reaction is expression for the variation of the concentration with distance
Most chemical reactions take place in solution and for
governed by the rate at which the reactant molecules dif- For v B to be independent of r, r2d[B](r)/dr must be a constant,
a thorough grasp of chemistry it is important to under-
fuse through the solvent. Because the combination of radi- which implies that, provided r > R*, [B](r) = a + b/r. Thus,
stand what controls their rates and how those rates can (a) Classes of reaction cals involves very little activation energy, radical and atom d[B](r)/dr = −b/r2, and r2d[B](r)/dr = −b, a constant, as required.
be modified.
The complicated overall process can be divided into simpler recombination reactions are often diffusion-controlled. An You can find the values of the constants a and b by noting that
➤ What is the key idea? parts by setting up a simple kinetic scheme. Suppose the rate activation-controlled reaction arises when a substantial acti- as r → ∞, [B](r) tends to its bulk value, [B]. Therefore a = [B]
of formation of an encounter pair AB is first order in each of vation energy is involved in the reaction AB → P. Then ka[AB] and hence [B](r) = [B] + b/r. When r = R*, [B](r) = 0, which
The rate of a chemical reaction in solution is controlled
the reactants A and B: << kd′[AB] (implying ka << kd′) and implies that b = −[B]R*. It follows that
either by the rate of diffusion of the reactants or by the
activation energy of the step that leads to products. A + B → AB v = kd[A][B]  R* 
[B](r ) = [B] 1− 
ka kd K  r 
kr ≈ = ka −○− Activation-controlled limit (18B.2b)
➤ What do you need to know already? As will be seen, kd (where the d signifies diffusion) is deter- kd′ c
mined by the diffusional characteristics of A and B. The en- Figure 18B.1 illustrates the distance dependence of [B](r)
This Topic makes use of the steady-state approximation −− according to this equation. The first derivative of [B](r) with
counter pair can break up without reaction or it can go on where kd / kd′ = K / c ○ (see Topic 17C) and K is the equilibrium
(Topic 17E) and draws on Fick’s first law of diffusion (Topic respect to distance is [B]R*/r2, so
to form products P. If it is supposed that both processes are constant for A + B ! AB. In this limit, the reaction proceeds
16C). At one point it uses the Stokes–Einstein relation
pseudofirst-order reactions (with the solvent perhaps playing a at a rate that depends on the equilibrium concentration of d[B](r)⁄dr = [B]R*⁄r2
(Topic 16C).
role), then the mechanism may be written encounter pairs and the rate at which energy accumulates in d[B](r )
these pairs from the surrounding solvent. Some experimental vB = 4 πr 2 DB = 4 πR * DB[B]
dr
AB → A + B v = kd′[AB] data are given in Table 18B.1. Step 3 Write an expression for the overall rate of reaction
AB → P v = ka[AB]
To express the rate of reaction, v B must be multiplied by the
Reactions in solution are entirely different from those in gases.
No longer are there collisions of molecules hurtling through The concentration of AB can now be found by applying the
(b) Diffusion and reaction number of A molecules in the solution. If the bulk concentra-
tion of A is [A], then the number of A molecules in a solution
space; now there is the jostling of one molecule through a steady-state approximation (Topic 17E) to the equation for the The rate of a diffusion-controlled reaction is calculated by
of volume V is NA[A]V. Therefore, the rate of reaction (still as
dense but mobile collection of molecules making up the fluid net rate of change of concentration of AB: considering the rate at which the reactants diffuse together.
amount/time) is
environment.
Steady-state rate = v BNA[A]V = 4πR*D BNA[A] [B]V
d[AB] How is that done? 18B.1 Finding an expression for the
= kd[A][B] − kd′ [AB] − ka[AB] = 0
dt 1
rate constant of a diffusion-controlled reaction
18B.1 Reactions in solution This expression solves to
Concentration, [B](r)/[B]0

Suppose that molecules of A and B in solution react imme- 0.8


kd[A][B] diately when they come within some critical distance R* of
Encounters between reactants in solution occur in a very dif- [AB] =
ka + kd′ one another and the rate of reaction is controlled by the rate 0.6
ferent manner from encounters in gases. The encounters of R*
reactant molecules dissolved in a solvent are considerably The rate of formation of products is therefore of encounters between A and B molecules as they diffuse
together. As a result of the reaction, the concentration of B 0.4
less frequent than in a gas. However, because a molecule also
d[P] ka kd molecules near A is decreased and a concentration gradient
migrates only slowly away from a location, two reactant mol- = ka[AB] = kr [A][B] kr = (18B.1)
dt ka + kd′ of B molecules is established. There is a diffusive flux of B 0.2
ecules that encounter each other stay near each other for much
towards A as a result of that gradient, and the flux is constant
longer than in a gas. This lingering of one molecule near an- Two limits can now be distinguished. If the rate of separa-
while the reaction is in progress. 0
other on account of the hindering presence of solvent mol- tion of the unreacted encounter pair is much slower than the 0 2 4 6 8 10
ecules is called the cage effect. Such an encounter pair may rate at which it forms products, then kd′[AB] << ka[AB] (or, after Step 1 Consider the rate at which B molecules cross the surface Radius, r/R *
accumulate enough energy to react even though it does not cancelling the [AB], kd′ << ka), and the effective rate constant is of a sphere centred on A
Figure 18B.1 The concentration profile for reaction in solution
have enough energy to do so when it first forms. The activa- kk If the (molar) flux of B molecules towards A is JB, the rate when a molecule B diffuses towards another reactant molecule
tion energy of a reaction is a much more complicated quantity kr ≈ a d = kd Diffusion-controlled limit (18B.2a) (expressed as amount divided by time) at which B molecules
k a and reacts if it reaches R*.
18B Diffusion-controlled reactions 789 790 18 Reaction dynamics

It is unrealistic to suppose that all A molecules are station- where 1 J = 1 kg m2 s−2. This result corresponds to 2.0 × the following table of values can be constructed:
ary, so the diffusion coefficient DB is now replaced by the 1010 dm3 mol−1 s−1. The experimental value is 1.3 × 1010 dm3 mol−1 s−1,
(b) Solutions of the equation
sum of the diffusion coefficients of the two species, D = DA + so the agreement is very good considering the approximations The material-balance equation is a second-order partial differ- [J]*/(mol dm−3) at x
D B. Because it is more convenient to express rates as concen- involved. ential equation and is far from easy to solve in general. Some t 1 mm 5 mm 1 cm
tration/time, both sides of the equation are divided by the idea of how it is solved can be obtained by considering the spe- 100 s 3.72 0 0
volume V, in which case cial case in which there is no convective motion (as in an un- 1000 s 1.96 0.45 0.005
kd stirred reaction vessel): 10 000 s 0.46 0.40 0.25
!#"#$
v = 4πR*DNA [A][B] ∂[J] ∂ [J] 2
18B.2 The material-balance equation ∂t
= D 2 − kr [J]
∂x
(18B.7)
from which it follows that the diffusion-controlled rate con-
stant is The diffusion of reactants plays an important role in many As may be verified by substitution (Problem 18B.1), if the solu- Even this relatively simple example has led to an equation
(18B.3) chemical processes, such as the diffusion of O2 molecules into tion of this equation in the absence of reaction (that is, for kr = that is difficult to solve, and only in some special cases can the
kd = 4πR*DNA
Rate constant of a diffusion-controlled reaction red blood cells and the diffusion of a gas towards a catalyst. To 0) is [J](x,t), then the solution [J]*(x,t) in the presence of reac- full material-balance equation be solved analytically. Most
catch a glimpse of the kinds of calculations involved consider tion (kr > 0) is modern work on reactor design and cell kinetics uses nu-
the diffusion equation (Topic 16C) generalized to take into ac- merical methods to solve the equation, and detailed solutions
[J]*(x,t) = [J](x,t)e−krt Diffusion with reaction (18B.8)
Brief illustration 18B.1 count the possibility that the diffusing, convecting molecules for realistic environments, such as vessels of different shapes
are also reacting. An example of a solution of the diffusion equation in the ab- (which influence the boundary conditions on the solutions)
The order of magnitude of R* is 10−10 m (100 pm) and that of sence of reaction is that given in Topic 16C for a system in and with a variety of inhomogeneously distributed reactants,
D for a species in water is 10−9 m2 s −1. It follows from eqn 18B.3
which initially a layer of n0NA molecules is spread over a plane can be obtained reasonably easily.
that (a) The formulation of the equation of area A:
kd ≈ 4π ×(10−10 m) × (10−9 m2 s −1) × (6.022 × 1023 mol−1) Consider a small volume element in a chemical reactor (or 2
n0e − x /4 Dt
≈ 8 × 105 m3 mol−1 s −1 a biological cell). The net rate at which J molecules enter the [J]( x , t ) = (18B.9)
A( πDt )1/2
region by diffusion and convection is given by eqn 16C.9 of
which corresponds to about 109 dm3 mol−1 s −1. An indication

Concentration of reactant, [J]


Topic 16C: When this expression is substituted into eqn 18B.8, the result
that a reaction is diffusion-controlled is therefore that its rate is an expression for the concentration of J as it diffuses away Dt = 0.05
constant is of that order of magnitude. from its initial surface layer and undergoes reaction in the
∂[J] ∂ 2[J] ∂[J]
=D 2 −v Diffusion equation (18B.5) overlying solution (Fig. 18B.2).
∂t ∂x ∂x
Dt = 0.10

Equation 18B.3 can be taken further by incorporating where v is the velocity of the convective flow of J and [J] in
the Stokes–Einstein equation (eqn 16C.4b of Topic 16C, DJ = general depends on both position and time. If J disappears by Brief illustration 18B.3
kT/6πηRJ) for the relation between the diffusion constant and a pseudofirst-order reaction, the net rate of change of molar
the hydrodynamic radius R A and RB of each molecule in a me- concentration due to chemical reaction is Suppose 1.0 g of iodine (3.9 mmol I2) is spread over a surface of
area 5.0 cm2 under a column of hexane (D = 4.1 × 10 –9 m2 s −1).
dium of viscosity η. As this relation is approximate, little extra Distance from plane, x
As it diffuses upwards it reacts with a pseudofirst-order rate
error is introduced by writing R A = RB = 12 R*, which leads to ∂[J] Figure 18B.2 The concentration profiles for a diffusing, reacting
= − kr [J] constant kr = 4.0 × 10−5 s −1. By substituting these values into
∂t system (e.g. a column of solution) in which one reactant is initially
8 RT 2
kd = Diffusion-controlled rate constant (18B.4) n0e − x /4 Dt −k t
r
in a layer at x = 0. In the absence of reaction (grey lines) the
3η [J]*( x , t ) =
Therefore, the overall rate of change of the concentration of J is A( πDt )1/2 concentration profiles are similar to those in Fig. 16C.5.
(The R in this equation is the gas constant.) The radii have can-
celled because, although the diffusion constants are smaller Spread
Change Loss
when the radii are large, the reactive collision radius is larger due to
due to due to
and the particles need travel a shorter distance to meet. In this non-uniform

approximation, the rate constant is independent of the identi-


distribution
convection reaction
Checklist of concepts
ties of the reactants, and depends only on the temperature and !"# $
the viscosity of the solvent. ∂[J] ∂ 2[J] ∂[J] $ ☐ 1. The cage effect, the lingering of one reactant molecule ☐ 3. The rate of an activation-controlled reaction is con-
=D 2 −v − kr [J] Material-balance equation (18B.6) near another due to the hindering presence of solvent trolled by the rate at which the encounter pair accumu-
∂t ∂x ∂x
molecules, results in the formation of an encounter lates sufficient energy.
Brief illustration 18B.2 Equation 18B.6 is called the material-balance equation. If the pair of reactant molecules. ☐ 4. The material-balance equation relates the overall rate
rate constant is large, then [J] will decline rapidly. However, ☐ 2. A reaction in solution may be diffusion controlled if of change of the concentration of a species to its rates
The rate constant for the recombination of I atoms in hexane at if the diffusion constant is large, then the decline can be re- its rate is controlled by the rate at which reactant mol- of diffusion, convection, and reaction.
298 K, when the viscosity of the solvent is 0.326 cP (with 1 P = plenished as J diffuses rapidly into the region. The convection ecules encounter each other in solution.
10−1 kg m−1 s −1) is term, which may represent the effects of stirring, can sweep
8 × (8.3145JK −1 mol −1 ) × (298K) material either into or out of the region according to the signs
kd = = 2.0 ×107 m3 mol −1 s −1
3 × (3.26 ×10−4 kg m −1 s −1 ) of v and the concentration gradient ∂[J]/∂x.

18B Diffusion-controlled reactions 791

Checklist of equations TOPIC 18C Transition-state theory


Property Equation Comment Equation number
Diffusion-controlled limit kr = kd v = kd[A][B] for the encounter rate 18B.2a

Activation-controlled limit kr = ka(K/c ) K for A + B ! AB, ka for the decomposition of AB 18B.2b
Diffusion-controlled rate constant kd = 4πR*DNA D = DA + DB 18B.3
kd = 8RT/3η Assumes Stokes–Einstein relation 18B.4
➤ Why do you need to know this material?
Material-balance equation ∂[J]/∂t = D∂2[J]/∂x2 Diffusion and convection with first-order reaction 18B.6
− v∂[J]/∂x − kr[J] Transition-state theory provides a way to relate the rate
Potential energy

constant of reactions to models of the cluster of atoms Ea


supposed to form when reactants come together. It pro- Reactants
vides a link between information about the structures of
reactants and the rate constant for their reaction.
Products
➤ What is the key idea?
Reactants come together to form an activated complex,
Reaction coordinate
which decays into products.
Figure 18C.1 A potential energy profile for an exothermic
➤ What do you need to know already? reaction. The height of the barrier between the reactants and
This Topic makes use of two strands: one is the relation products is the activation energy of the reaction.
between equilibrium constants and partition functions
(Topic 13F); the other is the relation between equilib-
A and B come into contact, distort, and begin to exchange or
rium constants and thermodynamic functions, such as the
discard atoms.
Gibbs energy, enthalpy, and entropy of reaction (Topic 6A).
You need to be aware of the Arrhenius equation for the
temperature dependence of the rate constant (Topic 17D). (a) The formulation of the equation
The reaction coordinate is a representation of the atomic
displacements, such as changes in interatomic distances and
In transition-state theory (which is also widely referred to bond angles, that are directly involved in the formation of
as activated complex theory), the notion of the transition products from reactants. The potential energy rises to a maxi-
state is used in conjunction with concepts of statistical mum and the cluster of atoms that corresponds to the region
thermodynamics to provide a more detailed calculation of close to the maximum is called the activated complex. After
rate constants than collision theory provides (Topic 18A). the maximum, the potential energy falls as the atoms re-
Transition-state theory has the advantage that a quantity arrange in the cluster and reaches a value characteristic of
corresponding to the steric factor appears automatically and the products. The climax of the reaction is at the peak of the
does not need to be grafted on to an equation as an after- potential energy, which can be identified with the activation
thought; it is an attempt to identify the principal features energy Ea. However, as in collision theory, this identification
governing the size of a rate constant in terms of a model of should be regarded as approximate and is clarified later. At
the events that take place during the reaction. this peak, two reactant molecules have come to such a degree
of closeness and distortion that a small further distortion will
send them in the direction of products. This crucial configu-
ration is called the transition state of the reaction. Although
18C.1 The Eyring equation some molecules entering the transition state might revert to
reactants, if they pass through this configuration then it is in-
In the course of a chemical reaction that begins with an evitable that products will emerge from the encounter.
encounter between molecules of A and molecules of B, the
A note on good practice The terms activated complex and tran-
potential energy of the system typically changes in a man-
sition state are often used as synonyms; however, it is best to
ner shown in Fig. 18C.1. Although the illustration displays preserve the distinction, with the former referring to the cluster
an exothermic reaction, a potential barrier is also common of atoms in the vicinity of the peak of the potential energy curve,
for endothermic reactions. As the reaction event proceeds, and the latter to their critical configuration.
18C Transition-state theory 793 794 18 Reaction dynamics

ing while others are lengthening and breaking; therefore, along


(d) The rate constant
the reaction coordinate, there is a vibration-like motion of the
Activated
complex atoms in the activated complex. If this motion occurs with a fre- All the parts of the calculation can now be combined into
Potential energy

Potential energy
quency ν ‡, then the frequency with which the cluster of atoms
RT ‡ ‡ ‡ kT RT
○ k K = κν

Reactants forming the complex approaches the transition state is also ν ‡. kr = ○ K
p hν ‡ p
−− −−

Transition However, it is possible that not every oscillation along the reac-
state tion coordinate takes the complex through the transition state. At this stage the unknown frequency ν ‡ (in blue) cancels to
Products For instance, the centrifugal effect of rotations might also be an Vibrational give one version of the Eyring equation:
levels
important contribution to the break-up of the complex, and in
some cases the complex might be rotating too slowly or rotating kT RT ‡
kr = κ K Eyring equation (18C.10)
Reaction coordinate rapidly but about the wrong axis. Therefore, it is more appropri- Reaction coordinate h p−−○
Figure 18C.2 A reaction profile (for an exothermic reaction). The ate to suppose that the rate of passage of the complex through
the transition state is only proportional, not equal, to the vibra- Figure 18C.3 In an elementary depiction of the activated ‡
The equilibrium constant K , which here is expressed in terms
horizontal axis is the reaction coordinate, and the vertical axis is
tional frequency along the reaction coordinate, and to write complex close to the transition state, there is a broad, shallow dip of partial pressures, can be rewritten in terms of concentra-
potential energy. The activated complex is the region near the
in the potential energy surface along the reaction coordinate. The
potential maximum, and the transition state corresponds to the tions by using [J] = pJ/RT, and then with some rearrangement
k ‡ = κν ‡ (18C.5) complex vibrates harmonically and almost classically in this well.
maximum itself. the equilibrium constant in terms of concentrations, K c‡ (the
where κ (kappa) is the transmission coefficient. In the absence terms in blue), can be identified:
of information to the contrary, κ is assumed to be about 1.

Transition-state theory pictures a reaction between A and B This frequency ν ‡ is much lower than for an ordinary molecu- Kc
!"#
as proceeding through the formation of an activated complex, lar vibration because the oscillation corresponds to the com- p p
−−

‡[C ‡ ] p
−−

[C ‡ ]c
−−

1 p
−−

p
−−

C‡, in a rapid pre-equilibrium (Fig. 18C.2): Brief illustration 18C.1 plex falling apart (Fig. 18C.3), so the force constant is very low. K ‡ = pC p = = × −−○ × = K c‡ ○ (18C.11)
A B [A][B] RT [A][B] c RT RTc
−−

Therefore, provided that hν ‡/kT << 1, the exponential may be


pC p
−−

Typical vibrations of small molecules occur at wavenumbers
expanded and the partition function reduces to

A + B ! C‡ K‡ = (18C.1)
pA pB of the order of 103 cm−1 (C–H bends, for example, occur in the
Substitution of this relation into eqn 18C.10, gives and an al-
range 1340–1465 cm−1) and therefore occur at frequencies of
where for this gas-phase reaction the activity of each species 1 kT ternative version of the Eyring equation:
the order of 1013 Hz. Suppose that the loosely bound cluster q= ≈
has been replaced by p/p . The development of femtosecond

vibrates at one or two orders of magnitude lower frequency, 1− (1− hν ‡ / kT +!) hν ‡


(and even attosecond) pulsed lasers has made it possible to kT ‡ Eyring equation
then ν ‡ ≈ 1011–1012 Hz. These figures suggest that k ‡ ≈ 1011– kr = κ ○ Kc (18C.12)
make observations on species that have such short lifetimes It follows that the partition function for the activated complex hc
−−
[alternative version]
1012 s −1, with κ perhaps reducing that value further.
that in a number of respects they resemble an activated com- is
plex, which often survive for only a few picoseconds. The equilibrium constant K ‡ can be computed from the parti-
kT
When the partial pressures, pJ, are expressed in terms of qC = −
○−
q −○−
(18C.8) tion functions of A, B, and C‡, so in principle the Eyring equa-
hν ‡ C
‡ ‡

the molar concentrations, [J], by using pJ = RT[J], the concen- tion is an explicit expression for calculating the second-order
tration of activated complex is related to the (dimensionless)
(c) The concentration of the activated complex where the bar in q C denotes that the partition function is for

○−
rate constant for a bimolecular reaction in terms of the molec-

equilibrium constant by Topic 13F explains how to calculate equilibrium constants all the other modes of the complex. The constant K‡ is therefore ular parameters for the reactants and the activated complex,
from structural data. Equation 13F.10b of that Topic (which and the quantity κ.
RT
[C ‡ ] = −−○ K ‡[A][B] (18C.2) expresses K in terms of the standard molar partition functions kT ‡ N q
−○−
The partition functions for the reactants can normally be
p K K ‡ = A C e − ∆ E /RT

K‡ = 0
(18C.9)
q J ) can be used directly, which in this case gives

hν ‡ qA q B −
○− −
○−
calculated quite readily by using either spectroscopic informa-
The activated complex falls apart by unimolecular decay into tion about their energy levels or the approximate expressions
products, P, with a rate constant k‡: NA q C −
○−
with K ‡ a kind of equilibrium constant, but with one vibra- set out in the Checklist at the end of Topic 13B. The difficulty
e − ∆ E /RT

K‡ = 0
(18C.6)
tional mode of C‡ discarded. with the Eyring equation, however, lies in the calculation of
C‡ → P v = k ‡[C ‡ ] (18C.3) qA q B

○− −
○−

the partition function of the activated complex: C‡ is difficult


It follows that with to investigate spectroscopically, and in general it is necessary
RT ‡ ‡ to make assumptions about its size, shape, and structure.
ΔE 0 = E 0(C‡) − E 0(A) − E 0(B) (18C.7)
v = kr [A][B] kr = ○ k K (18C.4) Brief illustration 18C.2
p
−−

Note that the units of NA and the q J are mol−1, so K ‡ is dimen-


Consider the case of two structureless particles A and B col-


The next task is the calculation of the unimolecular rate con- sionless (as is appropriate for an equilibrium constant).
liding to give an activated complex that resembles a diatomic
stant k‡ and the equilibrium constant K‡. The focus of the final step of this part of the calculation is
molecule. The activated complex is a diatomic cluster. It has Example 18C.1 Analysing the collision of structureless
the partition function of the activated complex. For the spe- one vibrational mode, but that mode corresponds to motion
cial vibration of the activated complex C‡ that tips it through particles
(b) The rate of decay of the activated complex the transition state and has frequency ν ‡ the partition function
along the reaction coordinate and therefore does not appear
in q C ‡ . It follows that the standard molar partition function
−○−
Consider the case of two structureless (and different) particles
An activated complex forms products only if it passes through may be written from eqn 13B.15 of Topic 13B as of the activated complex has only rotational and translational A and B colliding to give an activated complex that resembles
the transition state. As the reactant molecules approach the ac- 1 contributions. a diatomic molecule. Deduce an expression for the rate con-
q= ‡
tivated complex region, some bonds are forming and shorten- 1− e − hν /kT stant of the reaction A + B → P.

18C Transition-state theory 795 796 18 Reaction dynamics

1/2
Collect your thoughts Because the reactants are structureless  8kT  that is valid only at T = 0; at higher temperatures upper levels In summary:
kr = N A  σ *e − ∆ E /RT
 πµ 
0
‘atoms’, the only contribution to their partition functions is of all the species are occupied and contribute additional terms (a) Δ‡H = Ea − 2RT
from translation. The activated complex is a diatomic cluster of the order of RT (a value suggested by the equipartition prin- (bimolecular gas-phase reaction) (18C.17)
of mass mC‡ = mA + mB and moment of inertia I. It has one Self-test 18C.1 What additional contributions would there be ciple). Secondly, for gas-phase processes (but not for those in
(b) Δ‡H = Ea − RT Relations between
vibrational mode but, as explained in Brief illustration 18C.2, to the partition functions of the reactants and of the activated solution), ∆‡H differs from Δ‡U by another contribution RT. ∆‡H and Ea
that mode corresponds to motion along the reaction coordi- (bimolecular reaction in solution)
complex if the reaction were AB + C → P, with a linear acti- These additional contributions need to be identified.
nate. It follows that the standard molar partition function of vated complex? It now follows that
the activated complex has only rotational and translational symmetric stretch of the activated complex.
contributions. Expressions for the relevant partition func- Answer: Rotation and vibration of AB, bends and
How is that done? 18C.1 Relating the enthalpy of ‡
Rate constant
tions are given at the end of Topic 13B. kr = e 2 Be∆ S/R e − Ea /RT [transition-state theory, bimolecular (18C.18a)
activation to the activation energy gas-phase reaction]
The solution The translational partition functions are and
The relation between the enthalpy of activation and the
Vm−○− h RT activation energy depends on two equations. One is the ‡
Rate constant
qJ = ΛJ = Vm○ = kr = eBe∆ S/R e − Ea /RT

○− −−

Λ3J (2πmJ kT )1/2 p


−−

18C.2 Thermodynamic aspects expression for the temperature dependence of the ‘equilib-
rium constant’ K ‡, which is eqn 6B.2 of Topic 6B in the form
[transition-state theory,
bimolecular reaction in solution]
(18C.18b)


with J = A, B, and C , and with mC‡ = mA + mB. The expression d ln K ‡ /dT = ∆‡H/RT 2, and the second is the definition of the where, from eqn 18C.16, B = (kT/h)(RT/p ). The Arrhenius

The statistical thermodynamic version of transition-state the-


for the partition function of the activated complex is activation energy, which is eqn 17D.3 of Topic 17D in the frequency factors can be identified as
ory rapidly runs into difficulties because only in some cases is
form d ln kr /dT = Ea /RT 2. The link between the two expres-
anything known about the structure of the activated complex. Frequency factor
rotation translation sions is the alternative version of the Eyring equation, eqn A = e 2 Be∆ S/R

[transition-state theory, bimolecular (18C.19a)
" " However, the concepts it introduces, principally that of an ‡
18C.12, kr = (κ kT /hc )K c . Differentiation of the correspond-
−−

gas-phase reaction]
2 IkT V −○− equilibrium between the reactants and the activated complex,
qC = −
○−
× m3 ing expression for ln kr with respect to T gives and
!2 ΛC‡

have motivated a more general, empirical approach in which
the activation process is expressed in terms of thermodynamic d ln kr 1 d ln K c‡ Frequency factor
= + ‡
[transition-state theory, bimolecular
where the high-temperature form of the rotational partition functions. dT T dT A = eBe∆ S/R (18C.19b)
reaction in solution]
function has been used (Topic 13B). From eqn 18C.9, the con- 2
Then, by using d ln kr /dT = Ea /RT , it follows that
stant K ‡ is expressed in terms of the partition functions as The entropy of activation is negative because throughout
(a) Activation parameters d lnK c‡ the system reactant species are combining to form reactive
Ea = RT + RT 2 pairs. However, if there is a reduction in entropy below what
q C−○−‡ If K ‡ is taken as an equilibrium constant (despite one mode of dT
"$$ $#$$$ % C‡ having been discarded), then it can be expressed in terms of would be expected for the simple encounter of A and B, then
N (2 IkT / ! 2 ) Vm /ΛC3 −∆E /RT  N A Λ3A Λ3B  2IkT −∆E /RT
−− At this point it is necessary to distinguish between gas-phase the frequency factor A will be reduced further. Indeed, that
a Gibbs energy of activation, Δ‡G, through the definition

K ‡ = A −−○ 3

e =  3 −−○  2 e 0 0
and solution-phase reactions of the form A + B ! C ‡ . For additional reduction in entropy, Δ‡S steric, can be identified as
(V /ΛA ) (Vm /Λ3B )  ΛC V m  !
−−

Gibbs energy of activation
&$m'$ (& $'$ ( ‡
∆‡ G = − RT ln K ‡ [definition] (18C.13) the latter, the equilibrium constant is expressed in terms of the origin of the steric factor P of collision theory (Topic 18A),
qA −
○−
qB −
○− concentrations and the second term in the preceding equa- so that
All the Δ‡X in this section are standard thermodynamic quan-
tion can be identified with ∆‡H without further calculation. ‡
P-factor
tities, Δ‡X , but the standard state sign will be omitted to avoid P = e∆ S steric / R

It follows that [transition-state theory]


(18C.20)
It follows from eqn 18C.10 that overburdening the notation. Then from eqn 18C.10 the expres-
sion for the rate constant becomes For a solution-phase reaction: Ea = RT + ∆‡H Thus, the more complex the steric requirements of the

One further step is needed for a gas-phase reaction because encounter, the more negative the value of Δ‡S steric, and the
"$$$$#$$$$%
K
kT RT − ∆ G/RT ‡
kr = κ ○ e (18C.14) K c‡ is not the same as K ‡(which is expressed in terms of partial smaller the value of P.
h p
−−
kT RT  N A Λ3A Λ3B  2 IkT − ∆ E0 /RT
kr = κ ○ 
−−  2 e pressures). According to eqn 18C.11 the two are related by
h p  Λ3C‡ Vm−○−  ! Because ΔG = ΔH − TΔS, the Gibbs energy of activation can be K ‡ = ( p /RTc )K c‡ , which implies that
−−
○ −−

Brief illustration 18C.3


kT  ΛA ΛB  2 IkT − ∆ E0 /RT
3 divided into an entropy of activation, Δ‡S, and an enthalpy of
=κ N e activation, Δ‡H, by writing
h A  ΛC‡  ! 2
1/T Δ‡H/RT2
!#"#$ !"$ The reaction of propylxanthate ion in ethanoic acid buffer
d lnK c‡ d lnK ‡ 1 ∆‡H
−−
d RTc

Entropy and enthalpy of activation solutions can be represented by the equation A− + H+ → P.


∆‡G = ∆‡H − T∆‡S (18C.15) = ln −−○ + = +
The moment of inertia of a diatomic molecule of bond length [definition] dT dT p dT T RT 2 Near 30 °C, A = 2.05 × 1013 dm3 mol−1 s −1. To evaluate the
r is μr2, where μ = mAmB/(mA + mB), so after introducing the entropy of activation at 30 °C, use eqn 18C.19b, rearranged as
When eqn 18C.15 is used in eqn 18C.14 and κ is absorbed into Substitution of this expression into the previous expression
expressions for the thermal wavelengths Λ and cancelling A kT RT
common terms, the result is
the entropy term, the result is for Ea in terms of d lnK c‡ /dT gives ∆‡ S = R ln with B = ○ = 1.592 × 10
14
dm3 mol −1 s −1
eB h p
−−

‡ ‡ kT RT
1/2 kr = Be∆ S/R e − ∆ H /RT B= (18C.16) d lnK c‡  1 ∆‡H  Therefore,
 8kT  h p
−−
○ 2
Ea = RT + RT = RT + RT 2  + = RT + RT + ∆‡ H
kr = κ N A 
 πµ 
πr 2e − ∆ E0 /RT dT  T RT 
2

To develop this expression further it is necessary to find a re- 2.05 ×1013 dm3 mol −1s −1
It therefore follows that ∆‡ S = R ln = R ln 0.0473…
Finally, by identifying κπr as the reactive cross-section σ*, 2
lation between the enthalpy of activation and the activation e ×1.592 ×1014 dm3 mol −1s −1
the resulting expression is the same as that obtained from energy. The two are not the same, for two main reasons. One For a gas-phase reaction: Ea = 2 RT + ∆‡H = − 25.4 JK −1 mol −1
simple collision theory (eqn 18A.9): is that although it might be tempting to identify Ea with ∆ ‡U ,
18C Transition-state theory 797 798 18 Reaction dynamics

If k °r is the rate constant when the activity coefficients are 1 (i.e. 0.6 0
Δ‡G(a) 2+ 2+
k °r = k‡K/c ), then
−−

a
Δ‡G 0.4
b 2+ +
Gibbs energy, G

–0.1
Δ‡G(b) k°r
kr = log kr = log k°r − log Kγ (18C.21b)

log(kr/kr°)
|ΔrG| Kγ 0.2

log(kr/kr°)
+ +
+ 0 –0.2
At low concentrations the activity coefficients can be ex- 0 or
ΔrG(a) 0 –
ΔrG(b) pressed in terms of the ionic strength, I, of the solution by + –
using the Debye–Hückel limiting law (Topic 5F, particularly –0.2
+ 2– –0.3
eqn 5F.27, log γ± = − A |z +z −|I1/2). However, the expressions 2+ 2–
needed are those for the individual ions rather than the –0.4
Reaction coordinate 0 0.1 0.2 0 0.1 0.2
I1/2
mean value, and so it is more appropriate to write log γJ = I1/2
Figure 18C.4 For a related series of reactions here denoted a and
− A z 2J I1/2 and Figure 18C.5 Experimental tests of the kinetic salt effect for Figure 18C.6 The experimental ionic strength dependence
b, as the standard reaction Gibbs energy becomes more negative
reactions in water at 298 K. The ion types are shown as spheres, of the rate constant of a hydrolysis reaction: the slope gives
on going from a to b, the activation Gibbs energy decreases and
the rate constant increases. The approximate linear correlation
log γA = − A zA2I1/2 log γB = − A zB2I1/2 (18C.22a) and the slopes of the lines are those given by the Debye–Hückel information about the charge types involved in the activated
⦵ limiting law and eqn 18C.23. complex of the rate-determining step. The data plotted are
between Δ‡G and ΔrG is the origin of ‘linear free energy
relations’. with A = 0.509 in aqueous solution at 298 K and zA and zB the from Example 18C.2.
(signed) charge numbers of A and B, respectively. Because the
activated complex forms from reaction of one of the ions of A I 0.0050 0.0100 0.0150 0.0200 0.0250 0.0300
Example 18C.2 Analysing data in terms of the kinetic
Gibbs energies, enthalpies, and entropies of activation with one of the ions of B, the charge number of the activated kr/k°r 0.847 0.791 0.750 0.717 0.690 0.666
(and volumes and heat capacities of activation) are widely complex is zA + zB where zJ is positive for cations and negative salt effect Answer: −1
used to report experimental reaction rates, especially for for anions. Therefore The rate constant (at 298 K) for the hydrolysis of [CoBr(NH3)5]2+
organic reactions in solution. They are encountered when under basic conditions in aqueous solution varies with ionic
relationships between equilibrium constants and rates strength as in the following table. What can be deduced about
of reaction are explored by using correlation analysis, in log γ C = − A (zA + zB)2I1/2
‡ (18C.22b) the charge of the activated complex in the rate-determining
which ln K (which is equal to −ΔrG /RT) is plotted against

step? What might this deduction imply about the mechanism? 18C.3 The kinetic isotope effect
ln kr (which is proportional to −Δ‡G/RT). In many cases the When these expressions are inserted into eqn 18C.21b the re-
I 0.0050 0.0100 0.0150 0.0200 0.0250 0.0300
correlation is linear, signifying that as the reaction becomes sult is The postulation of a plausible reaction mechanism requires
kr/k°r 0.718 0.631 0.562 0.515 0.475 0.447
thermodynamically more favourable, its rate constant in- careful analysis of many experiments designed to deter-
creases (Fig. 18C.4). This linear correlation is the origin of Collect your thoughts According to eqn 18C.23, a plot of mine the fate of atoms during the formation of products.
log kr = log k°r − A {zA2 + zB2 − (zA + zB)2}I1/2 log(kr/kr°) against I1/2 should have a slope of 2 AzA zB. Because
the alternative name linear free energy relation (LFER). Observation of the kinetic isotope effect, a decrease in the
= log k°r + 2 A zA zBI1/2 (18C.23) A = 0.509 for aqueous solutions at 298 K, the slope will be rate of a chemical reaction upon replacement of one atom in
1.02zA zB. From the slope you can infer the charges of the ions a reactant by a heavier isotope, facilitates the identification of
(b) Reactions between ions Equation 18C.23 expresses the kinetic salt effect, the varia- involved in the formation of the activated complex. bond-breaking events in the rate-determining step. A primary
The full statistical thermodynamic theory is very complicated tion of the rate constant of a reaction between ions with the The solution Form the following table: kinetic isotope effect is observed when the rate-determining
for reactions involving ions in solution because the solvent ionic strength of the solution (Fig. 18C.5). The equation is in- step requires the scission of a bond involving the isotope. A
plays a role in the activated complex. The thermodynamic ver- terpreted as follows: I 0.0050 0.0100 0.0150 0.0200 0.0250 0.0300 secondary kinetic isotope effect is the reduction in reaction
sion of transition-state theory simplifies the discussion and is I1/2 0.071 0.100 0.122 0.141 0.158 0.173 rate even though the bond involving the isotope is not bro-
• If the reactant ions have the same sign (as in a
applicable to non-ideal systems. In the thermodynamic ap- log(kr/k°) –0.14 –0.20 –0.25 –0.29 –0.32 –0.35 ken to form product. In both cases, the effect arises from the
reaction between cations or between anions), then r

proach, the rate law change in activation energy that accompanies the replacement
increasing the ionic strength by the addition of inert These values are plotted in Fig. 18C.6. The slope of the (least
of an atom by a heavier isotope on account of changes in the
d[P] ‡ ‡ ions increases the rate constant. squares) straight line is −2.04, indicating that zA zB = −2. A
= k [C ] zero-point vibrational energies. What follows is a description
dt • The formation of a single, highly charged ionic com- possible explanation for this conclusion is that the two spe-
Physical interpretation

cies involved in the formation of the activated complex are of the primary kinetic isotope effect.
is combined with the thermodynamic equilibrium constant plex from two less highly charged ions is favoured Consider a reaction in which a C–H bond is cleaved. If scis-
[CoBr(NH3)5]2+, which has z = +2, and OH−, which has z = −1.
(Topic 6A) by a high ionic strength because the new ion has sion of this bond is the rate-determining step (Topic 17E),
The product of the charges is therefore −2.
aJ = γJ[J]/c ⦵
a denser ionic atmosphere and interacts with that then the reaction coordinate corresponds to the stretching
atmosphere more strongly. Comment. Although the point is not pursued here, you of the C–H bond and the potential energy profile is shown
aC‡ [C ‡ ]c
−−

γC should be aware that the rate constant is also influenced by
• Conversely, ions of opposite charge react more in Fig. 18C.7. On deuteration, the dominant change is the

K= = Kγ Kγ =
aAaB [A][B] γ Aγ B the relative permittivity of the medium.
slowly in solutions of high ionic strength. Now the reduction of the zero-point energy of the bond (because the
Then charges cancel and the complex has a less favour- Self-test 18C.2 An ion of charge number +1 is known to be deuterium atom is heavier). The whole reaction profile is not
able interaction with its atmosphere than the sepa- involved in the activated complex of a reaction. Deduce the lowered, however, because the relevant vibration in the acti-
d[P] k‡ K
= kr [A][B] kr = (18C.21a) rated ions. charge number of the other ion from the following data, vated complex has a very low force constant, so there is little
dt Kγ c
−−

recorded at 298 K in aqueous solution: zero-point energy associated with the reaction coordinate in

18C Transition-state theory 799 800 18 Reaction dynamics

where R = NAk. After using eqn 18C.24 for Ea(C–D) − Ea(C–H) of tunnelling through a barrier decreases as the mass of the
Potential energy and wavefunction

in this expression, the result is particle increases (Topic 7D), so deuterium tunnels less effi-
ciently through a barrier than hydrogen and its reactions are
Ea(C–D)

Ea(C–H)
Potential energy, V

!ω (C–H)   µCH  
1/2 Wavefunction
kr (C–D) −ζ correspondingly slower. Quantum mechanical tunnelling
=e with ζ = 1−
C–H kr (C–H) 2kT   µCD   can be the dominant process in reactions involving hydrogen
 
C–D atom or proton transfer when the temperature is so low that
Effect of deuteration on the rate constant (18C.25)
very few reactant molecules can overcome the activation en-
Note that ζ > 0 (ζ is zeta) because μCD > μCH and so it follows ergy barrier.
that kr(C–D)/kr(C–H) < 1. As expected from Fig. 18C.7, the
rate constant decreases upon deuteration.
Reaction coordinate Reaction coordinate

Figure 18C.7 Changes in the reaction profile when a C–H bond Figure 18C.8 A proton can tunnel through the activation energy
undergoing cleavage is deuterated. In this illustration the C–H barrier that separates reactants from products, so the effective
Brief illustration 18C.4
and C–D bonds are modelled as harmonic oscillators. The only height of the barrier is reduced and the rate of the proton transfer
significant change is in the zero-point energy of the reactants, From infrared spectra, the fundamental vibrational wave- reaction increases. The effect is represented by drawing the
wavefunction of the proton near the barrier. Proton tunnelling is
which is lower for C–D than for C–H. As a result, the activation number ν! for stretching of a C–H bond is about 3000 cm−1. To
important only at low temperatures, when most of the reactants
energy is greater for C–D cleavage than for C–H cleavage. convert this wavenumber to an angular frequency, ω = 2πν,
are trapped on the left of the barrier.
use ω = 2πcν!, so that
ω = 2π × (2.998 × 1010 cm s −1) × (3000 cm−1)
either form of the activated complex. From these considera- = 5.65… × 1014 s −1
tions it is possible to investigate the effect of deuteration on
the activation energy.
The ratio of effective masses is Checklist of concepts
µCH  mCmH   mC + mD 
= ×
µCD  mC + mH   mCmD  ☐ 1. In transition-state theory, it is supposed that an acti- ☐ 4. The kinetic salt effect is the effect of an added inert salt
vated complex is in equilibrium with the reactants. on the rate constant of a reaction between ions.
 12.01×1.0078   12.01+ 2.0140 
= ×
 12.01+1.0078   12.01× 2.0140  ☐ 2. The rate at which the activated complex forms products ☐ 5. The kinetic isotope effect is the decrease in the rate
How is that done? 18C.2 Exploring the primary kinetic depends on the rate at which it passes through a transi- constant of a chemical reaction upon replacement of
= 0.539…
isotope effect tion state. one atom in a reactant by a heavier isotope.
Now use eqn 18C.25 to calculate ☐ 3. The rate constant may be parametrized in terms of the
Consider the cleavage of a C–H bond in a larger molecule. For
such a reaction it is reasonable to assume that motion along (1.055 ×10−34 J s) × (5.65…×1014 s −1 ) Gibbs energy, entropy, and enthalpy of activation.
the reaction coordinate is dominated by stretching and com- ζ= × (1− 0.539…1/2 )
2 × (1.381×10−23 J K −1 ) × (298 K)
pression of the C–H fragment. Therefore, to a good approxi- = 1.92 …
mation, a change in the activation energy arises only from the
change in zero-point energy of the stretching vibration, 12 !ω . and
It follows from Fig. 18C.7 that Checklist of equations
kr (C–D) −1.92…
=e = 0.146
kr (C–H)
Ea (C–D) − Ea (C–H) = NA{ !ω (C–H) − !ω (C–D)}1
2
1
2
Property Equation Comment Equation number
Therefore at room temperature cleavage of the C–H bond
= 12 NA !{ω (C–H) − ω (C–D)} ‘Equilibrium constant’ for activated K ‡ = ( NA q C−○‡− / q A○ q B○ )e − ∆ E0 /RT Assume equilibrium; one vibrational mode of C‡ 18C.9
−− −−

should be about seven times faster than cleavage of the C–D complex formation discarded
bond, other conditions being equal. Experimental values of −−
kr = κ (kT / h)( RT / p )K ‡

Eyring equation Transition-state theory 18C.10
Then from Topic 11C ω (C–D) = (μCH/μCD)1/2ω (C–H), where μ kr(C–D)/kr(C–H) can differ significantly from those predicted −−

is the relevant effective mass and therefore that kr = κ (kT / hc )K c‡ 18C.12
by eqn 18C.25 on account of the severity of the assumptions
in the model. Gibbs energy of activation ∆‡G = − RT ln K ‡ Definition 18C.13
 µ   1/2

Ea (C–D) − Ea (C–H) = NA !ω (C–H) 1−  CH  


1 Enthalpy and entropy of activation ∆‡G = ∆‡H − T∆‡S Definition 18C.15
  µCD  
2

Parametrization kr = en Be∆ S/Re − Ea /RT n = 2 for bimolecular gas-phase reactions; n = 1 18C.18
Effect of deuteration on the activation energy (18C.24) ‡ for solution 18C.19
In some cases, substitution of deuterium for hydrogen re- A-factor A = en Be ∆ S/R

If the Arrhenius frequency factor does not change upon deu- sults in values of kr(C–D)/kr(C–H) that are too low to be ac- P-factor

P = e∆ S steric /R 18C.20
teration, the rate constants for the two species should be in counted for by eqn 18C.25, even when more complete models Kinetic salt effect log kr = log k°r + 2AzAzBI1/2 Assumes Debye–Hückel limiting law valid 18C.23
the ratio are used to predict ratios of rate constants. Such abnormal Primary kinetic isotope effect kr (C–D)/ kr (C–H) = e −ζ Cleavage of a C–H/D bond in the rate- 18C.25
kinetic isotope effects are evidence for a path in which quan- ζ = (!ω (C–H)/2kT ) × determining step
kr (C–D) − { Ea (C–D)−Ea (C–H)}/RT
=e = e − { Ea (C–D)−Ea (C–H)}/NAkT tum mechanical tunnelling of hydrogen atoms takes place {1 − ( µCH / µCD )1/2 }
kr (C–H)
through the activation barrier (Fig. 18C.8). The probability
802 18 Reaction dynamics

TOPIC 18D The dynamics of molecular Maxwell–Boltzmann


distribution Supersonic
nozzle
with a sensitive pressure gauge, a ‘bolometer’ (a detector that
responds to the incident energy by making use of the temper-
ature-dependence of resistance), or an ionization detector, in

collisions

Intensity
which the incoming molecule is first ionized and then detected
electronically. The rotational and vibrational state of the scat-
tered molecules may also be determined spectroscopically.

(b) Experimental results


Molecular speed
The primary experimental information from a molecular
Figure 18D.2 The shift in the mean speed and the width of the
beam experiment is the fraction of the molecules in the inci-
(a) Techniques distribution brought about by use of a supersonic nozzle.
➤ Why do you need to know this material? dent beam that are scattered into a particular direction. The
The basic arrangement of a molecular beam experiment is fraction is normally expressed in terms of dI, the number of
Chemists are interested in the details of chemical reac-
shown in Fig. 18D.1. Atoms or molecules emerge from a source the molecules in the beam (Fig. 18D.2), which may be as low molecules in a given time divided by the length of the inter-
tions, and there is no more detailed approach than that
chamber (which may be heated if the species is not already as 1 K. Such jets are called supersonic because the mean speed val, scattered into a cone (described by a solid angle dΩ) that
involved in the study of the dynamics of reactive encoun-
a gas) through a pinhole and out into a vacuum chamber. If of the molecules in the jet is much greater than the speed of represents the area covered by the ‘eye’ of the detector (Fig.
ters, when one molecule collides with another and atoms
the pressure of vapour in the source is increased so that the sound in the jet. 18D.4). This rate is reported as the differential scattering
exchange partners.
mean free path of the molecules in the emerging beam is much A supersonic jet can be converted into a more parallel super- cross-section, σ, the constant of proportionality between the
➤ What is the key idea? shorter than the diameter of the pinhole, many collisions take sonic beam if it is ‘skimmed’ in the region of hydrodynamic value of dI and the intensity, I, of the incident beam, the num-
place even outside the source. The net effect of these collisions, flow and the excess gas pumped away. A skimmer consists of ber density of target molecules, N, and the infinitesimal path
The rates of reactions in the gas phase can be investigated
which give rise to hydrodynamic flow, is to transfer momen- a conical nozzle shaped to avoid any supersonic shock waves length dx through the sample:
by exploring the trajectories of molecules on potential
tum into the direction of the beam. The molecules in the beam spreading back into the gas and so increasing the translational
energy surfaces. Differential scattering cross-section
then travel with very similar speeds, so further downstream temperature (Fig. 18D.3). A jet or beam may also be formed by dI = σINdx (18D.1)
[definition]
➤ What do you need to know already? few collisions take place between them. This condition is using helium or neon as the principal gas, and injecting mol-
called molecular flow. If necessary, atoms or molecules mov- ecules of interest into it in the hydrodynamic region of flow. The value of σ (which has the dimensions of area) depends on
This Topic builds on the concept of rate constant (Topic
ing with a given speed can be selected by using a velocity selec- As well as having a low translational temperature, the mol- the impact parameter, b, the initial perpendicular separation
17A) and in one part of the discussion uses the concept of
tor, as depicted in Fig. 18D.1. ecules in the beam also have low rotational and vibrational of the paths of the colliding molecules (Fig. 18D.5), and the
partition function (Topic 13B). The discussion of potential
In a molecular beam of this kind the spread of speeds is temperatures. In this context, a rotational or vibrational tem- details of the intermolecular potential.
energy surfaces is qualitative, but the underlying calcula-
much smaller than that predicted by the Maxwell–Boltzmann perature means the temperature that should be used in the The role of the impact parameter is most easily seen by
tions are those of self-consistent field theory (Topic 9E).
distribution. The unexpected narrowness of the distribution Boltzmann distribution to reproduce the observed popula- considering the impact of two hard spheres (Fig. 18D.6). If
is interpreted by assigning a low translational temperature to tions of the states. However, as rotational states equilibrate b = 0, the projectile is on a trajectory that leads to a head-on
more slowly than translational states, and vibrational states collision, so the only scattering intensity is detected when the
equilibrate even more slowly, the rotational and vibrational detector is at θ = π. When the impact parameter is so great
populations of the species correspond to somewhat higher that the spheres do not make contact (b > R A + RB), there is no
The investigation of the dynamics of the collisions between re- Source temperatures, of the order of 10 K for rotation and 100 K for
actant molecules is the most detailed level of the examination Detector vibrations. dΩ
of the factors that govern the rates of reactions. There are two The target gas may be either a bulk sample or another mo-
approaches: an experimental one that uses molecular beams lecular beam. The detectors may consist of a chamber fitted
Vacuum
and a theoretical one that uses the results of computations. Selector chamber θ

To pump Pinhole
Source

18D.1 Molecular beams Gas flow


Figure 18D.4 The definition of the solid angle, dΩ, for scattering.
Figure 18D.1 The basic arrangement of a molecular beam
Molecular beams, which consist of collimated, narrow apparatus. The atoms or molecules emerge from a source, and
pass through a velocity selector, such as that discussed in Topic
streams of molecules travelling through an evacuated vessel,
1B. The scattering occurs from the target gas (which might take
allow collisions between molecules in preselected states (e.g. Oven Skimmer
the form of another beam), and the flux of particles entering
specific rotational and vibrational states) to be studied, and the detector set at some angle is recorded. In a crossed-beam
can be used to identify the states of the products of a reactive experiment, state-selected molecules are generated in two b
collision. Information of this kind is essential if a full picture Collimator
separate sources, and are directed perpendicular to one another.
of the reaction is to be built, because the rate constant is an The detector responds to molecules (which may be product Figure 18D.3 A supersonic beam is generated by using a
average over events in which reactants in different initial states molecules if a chemical reaction occurs) scattered into a chosen skimmer to remove some of the molecules from the beam, so Figure 18D.5 The definition of the impact parameter, b, as the
evolve into products in their final states. direction. leading to a greater degree of collimation. perpendicular separation of the initial paths of the particles.

18D The dynamics of molecular collisions 803 804 18 Reaction dynamics

Interfering rainbow angle, θr, is the angle for which dθ/db = 0 and the v 7
paths 6
RA b > RA + RB scattering is strong. 5
RB
b=0 Another phenomenon that can occur in certain beams is the 4
capturing of one species by another. The vibrational tempera- 3
(a) (b) ture in supersonic beams is so low that van der Waals mol-
ecules may be formed, which are complexes of the form AB 2
in which A and B are held together by van der Waals forces or
Figure 18D.8 Two paths leading to the same destination will
hydrogen bonds. Large numbers of such molecules have been 1
0 < b < RA + RB
interfere quantum mechanically; in this case they give rise to studied spectroscopically, including ArHCl, (HCl)2, ArCO2, Frequency, ν
quantum oscillations in the forward direction. and (H2O)2. The study of their spectroscopic properties gives 0
detailed information about the intermolecular interactions in-
Figure 18D.10 Infrared chemiluminescence from CO produced
volved.
in the reaction O + CS → CO + S arises from the non-equilibrium
jectile molecule from source to detector, and then considering
(c) populations of the vibrational states of CO. The horizontal bars
the effects of interference between them. indicate the relative populations of the vibrational states and the
Figure 18D.6 Three typical cases for the collisions of two hard Two quantum mechanical effects are of great importance. blue arrows indicate the observed transitions.
spheres: (a) b = 0, giving backward scattering; (b) b > RA + RB, A molecule with a certain impact parameter might approach 18D.2 Reactive collisions
giving forward scattering; (c) 0 < b < RA + RB, leading to scattering the attractive region of the potential in such a way that it is
into one direction on a ring of possibilities. (The target molecule deflected towards the repulsive core (Fig. 18D.8), which then Detailed experimental information about the intimate processes ionization (REMPI), in which one or more photons promote a
is taken to be so heavy that it remains virtually stationary.) repels it out through the attractive region to continue its flight that occur during reactive encounters comes from molecular molecule to an electronically excited state and then additional
in the forward direction. Some molecules, however, also travel beams, especially crossed molecular beams (see Fig. 18D.1). The photons are used to generate ions from the excited state. The
in the forward direction because they have impact parameters detector for the products of the collision of molecules in the two power of REMPI lies in the fact that the experimenter can
scattering and the scattering cross-section is zero at all angles so large that they are undeflected. The wavefunctions of the beams can be moved to different angles to observe the angular choose which reactant or product to study by tuning the laser
except θ = 0. Glancing blows, with 0 < b ≤ R A + RB, lead to scat- molecules that take the two types of path interfere, and the in- distribution of the products. Because the molecules in the in- frequency to the electronic absorption band of a specific mol-
tering intensity in cones around the forward direction. tensity in the forward direction is modified. The effect is called coming beams can be prepared with different energies (e.g. with ecule.
The scattering pattern of real molecules, which are not hard quantum oscillation. The same phenomenon accounts for the different translational energies by using rotating sectors and The angular distribution of ions can be determined by reac-
spheres, depends on the details of the intermolecular potential optical ‘glory effect’, in which a bright halo can sometimes be supersonic nozzles, with different vibrational energies by using tion product imaging. In this technique, product ions are ac-
energy and molecular shape. The scattering also depends on seen surrounding an illuminated object. (The coloured rings selective excitation with lasers, and with different orientations celerated by an electric field towards a phosphorescent screen
the relative speed of approach of the two molecules: a very fast around the shadow of an aircraft cast on clouds by the Sun, by using electric fields), it is possible to study the dependence and the light emitted from specific spots where the ions struck
molecule might pass through the interaction region without and often seen in flight, are an example of an optical glory.) of the success of collisions on these variables and to study how the screen is imaged by a charge-coupled device (CCD).
much deflection, whereas a slower one on the same path might The second quantum effect is the observation of a strongly they affect the properties of the product molecules.
be temporarily captured and undergo considerable deflection enhanced scattering in a non-forward direction. This effect is
(Fig. 18D.7). The variation of the scattering cross-section with called rainbow scattering because the same mechanism ac-
(b) State-to-state reaction dynamics
the relative speed of approach gives information about the counts for the appearance of an optical rainbow. The origin
(a) Probes of reactive collisions The concept of collision cross-section is introduced in con-
strength and range of the intermolecular potential. of the phenomenon is illustrated in Fig. 18D.9. As the impact One method for examining the energy distribution in the nection with collision theory in Topic 18A, where it is shown
A further point is that the outcome of collisions is deter- parameter decreases, there comes a stage at which the scat- products is infrared chemiluminescence, in which vibration- that the second-order rate constant, kr, can be expressed as a
mined by quantum, not classical, mechanics. The wave nature tering angle passes through a maximum and the interference ally excited molecules emit infrared radiation as they return to Boltzmann-weighted average of the reactive cross-section and
of the molecules can be taken into account, at least to some between the paths results in a strongly scattered beam. The their ground states. By studying the intensities of the infrared the relative speed of approach of the colliding reactant mole-
extent, by drawing all classical trajectories that take the pro- emission spectrum, the populations of the vibrational states cules. Equation 18A.7 of that Topic (kr = N A ∫ 0∞ σ (ε )vrel f (ε )dε )
of the products may be determined (Fig. 18D.10). Another may be written as
method makes use of laser-induced fluorescence. In this tech-
kr = 〈σvrel 〉NA (18D.2)
Slow nique, a laser is used to excite a product molecule from a spe-
molecule Decreasing b cific vibration–rotation level; the intensity of the fluorescence where the angle brackets denote a Boltzmann average.
from the upper state is monitored and interpreted in terms of Molecular beam studies provide a more sophisticated version
Fast the population of the initial vibration–rotation state. When of this quantity, for they provide the state-to-state cross-sec-
molecule the molecules being studied do not fluoresce efficiently, ver- tion, σnn′, and hence the state-to-state rate constant, knn′, for
sions of Raman spectroscopy (Topic 11A) can be used to mon- the reactive transition from initial state n of the reactants to
itor the progress of reaction. final state n′ of the products:
Maximum deflection angle, θr Multiphoton ionization (MPI) techniques are also good
knn′ = 〈σ nn′vrel 〉 N A State-to-state rate constant (18D.3)
Figure 18D.9 The interference of paths leading to rainbow alternatives for the study of weakly fluorescing molecules. In
Figure 18D.7 The extent of scattering may depend on the scattering. The rainbow angle, θr, is the maximum scattering MPI, the absorption by a molecule of several photons from The rate constant kr is the sum of the state-to-state rate constants
relative speed of approach as well as the impact parameter. The angle reached as b is decreased. Interference between the one or more pulsed lasers results in ionization if the total pho- over all final states (because a reaction is successful whatever
dark central zone represents the repulsive core; the fuzzy outer numerous paths at that angle modifies the scattering intensity ton energy is greater than the ionization energy of the mol- the final state of the products) and over a Boltzmann-weighted
zone represents the long-range attractive potential. markedly. ecule. An important variant of MPI is resonant multiphoton sum of initial states (because the reactants are initially present
18D The dynamics of molecular collisions 805 806 18 Reaction dynamics

with a characteristic distribution of populations at a tempera- Potential energy RBC is very large, a section through the surface is the molecular Potential energy
ture T): potential energy curve of an isolated HAHB molecule.

kr = ∑knn′ (T ) fn (T ) (18D.4)
n ,n′
Brief illustration 18D.2
where fn(T) is the Boltzmann factor at a temperature T. It fol-
The bimolecular reaction H + O2 → OH + O plays an impor-
lows that if the state-to-state cross-sections can be determined
tant role in combustion processes. The reaction can be char-
or calculated for a wide range of approach speeds and initial
acterized in terms of the HO2 potential energy surface and the
and final states, then there is a route to the calculation of the RAB RBC
two distances for collinear approach RHOA and ROAOB . When
rate constant for the reaction. RBC RAB
RHOA is very large, the variation of the HO2 potential energy Figure 18D.14 The transition state is a set of configurations (here,
with ROAOB is that of an isolated dioxygen molecule as its bond marked by the red line across the saddle point) through which
Brief illustration 18D.1 Figure 18D.11 The potential energy surface for the H + H2 → H2 + length is changed. Similarly, when ROAOB is very large, a section successful reactive trajectories must pass.
H reaction when the atoms are constrained to be collinear. through the potential energy surface is the molecular potential
Suppose a harmonic oscillator collides with another oscillator energy curve of an isolated OH radical.
of the same effective mass and force constant. If the state-to-
state rate constant for the excitation of the latter’s vibration is to form a bond with HB. The HB–HC bond relaxes at the de-
kvv′ = k°r δ vv′ for all the states v and v′, implying that an excitation along the HB–HC axis requires less energy for reaction than mand of the incoming atom, and the potential energy climbs
can flow only from any level to the same level of the second any other approach, so initially it is convenient to confine The actual path of the atoms in the course of the encoun- only as far as the saddle-shaped region of the surface, to the
oscillator, then at a temperature T, when fv(T) = e −vhν /kT /q, where attention to that collinear approach. Two parameters are re- ter depends on their total energy, the sum of their kinetic and saddle point marked C‡. The encounter that requires the least
q is the molecular vibrational partition function (Topic 13B), quired to define the nuclear separations: the HA–HB separa- potential energies. However, an initial idea of the paths avail- increase in potential energy is the one in which the atoms take
the overall rate constant is tion R AB and the HB–HC separation RBC. able to the system can be obtained by identifying paths that route C up the floor of the valley, through the saddle point,
q
At the start of the encounter RAB is effectively infinite and RBC correspond to least potential energy. For example, consider and down the floor of the other valley as HC recedes and the
k°r k°r
!#"#
$ is the H2 equilibrium bond length. At the end of a successful the changes in potential energy as HA approaches HBHC. If the new HA–HB bond achieves its equilibrium length. This path is
kr =
q ∑δ vv′ e − vhv/kT =
q ∑e − v′hv/kT
= k°r reactive encounter RAB is equal to the equilibrium bond length HB–HC bond length is constant during the initial approach of the reaction coordinate.
v ,v ′ v′
and RBC is infinite. The total energy of the three-atom system HA, then the potential energy of the H3 cluster rises along the It is now possible to make contact with the transition-state
depends on their relative separations, and can be found by path marked A in Fig. 18D.13. The potential energy reaches theory of reaction rates (Topic 18C). In terms of trajectories
doing an electronic structure calculation. The plot of the total a high value as HA is pushed into the molecule and then de- on potential surfaces with a total energy close to the saddle
energy of the system against RAB and RBC gives the potential en- creases sharply as HC breaks off and separates to a great dis- point energy, the transition state can be identified with a criti-
18D.3 Potential energy surfaces ergy surface of this collinear reaction (Fig. 18D.11). This surface tance. An alternative reaction path can be imagined (B) in cal range of configurations such that every trajectory that goes
is normally depicted as a contour diagram (Fig. 18D.12). which the HB–HC bond length increases while HA is still far through this configuration goes on to react (Fig. 18D.14). Most
One of the most important concepts for discussing beam re- When R AB is very large, the variation in potential energy away. Both paths, although feasible if the molecules have suf- trajectories on potential energy surfaces do not go directly
sults and calculating the state-to-state collision cross-section represented by the surface as RBC changes is that of an isolated ficient initial kinetic energy, take the three atoms to regions of over the saddle point and therefore, to result in a reaction, they
is the potential energy surface of a reaction, the potential H2 molecule as its bond length is altered. A section through high potential energy in the course of the encounter. require a total energy significantly higher than the saddle-
energy as a function of the relative positions of all the atoms the surface at R AB = ∞, for example, is the same as the H2 bond- The path of least potential energy is the one marked C, cor- point energy. As a result, the experimentally determined acti-
taking part in the reaction. Potential energy surfaces may be ing potential energy curve. At the edge of the diagram where responding to RBC lengthening as HA approaches and begins vation energy is often significantly higher than the calculated
constructed from experimental data and from results of quan- saddle-point energy.
tum chemical calculations (Topic 9E). The theoretical method
requires the systematic calculation of the energies of the sys-
tem in a large number of geometrical arrangements. Special
computational techniques, such as those described in Topic Re
9E, are used to take into account electron correlation, which Some results from experiments
18D.4
RBC
arises from interactions between electrons as they move closer and calculations
to and farther from each other in a molecule or molecular B
RBC
C
cluster. Techniques that incorporate electron correlation ac- Although quantum mechanical tunnelling can play an im-
curately are very time consuming and, consequently, the most C‡ A portant role in reactivity, particularly in hydrogen atom
reliable results are for reactions between relatively simple par- Re
0
and electron transfer reactions, this discussion begins with
ticles, such as the reactions H + H2 → H2 + H and H + H2O → RAB consideration of the classical trajectories of particles over
OH + H2. An alternative is to use semi-empirical methods, in Figure 18D.13 Various trajectories through the potential energy
surfaces. From this viewpoint, to travel successfully from
RAB
which results of calculations and experimental parameters are surface shown in Fig. 18D.12. Path A corresponds to a route reactants to products, the incoming molecules must possess
used to construct the potential energy surface. Figure 18D.12 The contour diagram (with contours of equal in which RBC is held nearly constant as HA approaches; path B enough kinetic energy to be able to climb to the saddle point
To illustrate the features of a potential energy surface, con- potential energy) corresponding to the surface in Fig. 18D.11. corresponds to a route in which RBC lengthens at an early stage of the potential surface. Therefore, the shape of the surface can
sider the collision between an H atom and an H2 molecule. Re marks the equilibrium bond length of an H2 molecule (strictly, during the approach of HA; path C is the route along the floor of be explored experimentally by changing the relative speed of
Detailed calculations show that the approach of an atom HA it relates to the arrangement when the third atom is at infinity). the potential valley. approach (by selecting the beam velocity) and the degree of

18D The dynamics of molecular collisions 807 808 18 Reaction dynamics

H however, the same amount of energy is present solely as transla-


RBC
tional kinetic energy, then the system moves along C* and travels
RAB smoothly over the saddle point into products. Therefore, reac-
H
tions with attractive potential energy surfaces proceed more effi-
ciently if the energy is in relative translational motion. Moreover,
C the potential energy surface shows that once past the saddle
C 1* C2* point the trajectory runs up the steep wall of the product valley,
H and then rolls from side to side as it falls to the foot of the valley
(a) (b) C* ‡
as the products separate. In other words, the products emerge in
a vibrationally excited state.
Figure 18D.16 An indication of how the anisotropy of the Figure 18D.18 An attractive potential energy surface. A Now consider the repulsive surface. On trajectory C the col-
potential energy changes as H approaches H2 with different successful encounter (C*) involves high translational kinetic lisional energy is largely in translation. As the reactants ap-
angles of attack. The collinear attack has the lowest potential energy and results in a vibrationally excited product. proach, the potential energy rises. Their path takes them up
barrier to reaction. The surface indicates the potential energy the opposing face of the valley, and they are reflected back into
profile along the reaction coordinate for each configuration. the reactant region. This path corresponds to an unsuccessful
C4 encounter, even though the energy is sufficient for reaction.
C3 (b) Attractive and repulsive surfaces
On C* some of the energy is in the vibration of the reactant
(c) (d)
If the collision is sticky, so that when the reactants collide Some reactions are very sensitive to whether the energy has molecule and the motion causes the trajectory to weave from
they orbit around each other, the products can be expected to been deposited into a vibrational mode or left as the relative side to side up the valley as it approaches the saddle point. This
Figure 18D.15 Some successful (*) and unsuccessful encounters.
emerge in random directions because all memory of the ap- translational kinetic energy of the colliding molecules. For ex- motion may be sufficient to tip the system round the corner
(a) C1* corresponds to the path along the foot of the valley; (b)
proach direction has been lost. A rotation takes about 1 ps, so ample, if two HI molecules are hurled together with more than to the saddle point and then on to products. In this case, the
C2* corresponds to an approach of A to a vibrating BC molecule,
and the formation of a vibrating AB molecule as C departs. (c) C3
if the collision is over in less than that time the complex will twice the activation energy of the reaction, then no reaction product molecule is expected to be in an unexcited vibrational
corresponds to A approaching a non-vibrating BC molecule, but not have had time to rotate and the products will be thrown off occurs if all the energy is solely translational. For F + HCl → state. Reactions with repulsive potential surfaces can therefore
with insufficient translational kinetic energy; (d) C4 corresponds to in a specific direction. In the collision of K and I2, for example, Cl + HF, for example, the reaction is about five times more ef- be expected to proceed more efficiently if the excess energy is
A approaching a vibrating BC molecule, but still the energy, and most of the products are thrown off in the forward direction ficient when the HCl is in its first vibrational excited state than present as vibrations. This is the case with the H + Cl2 → HCl +
the phase of the vibration, is insufficient for reaction. (‘forward’ and ‘backward’ refer to directions in a centre-of- when it is in its vibrational ground state, although HCl has the Cl reaction, for instance.
mass coordinate system with the origin at the centre of mass of same total energy.
the colliding reactants and collision occurring when molecules The origin of these requirements can be found by examining
vibrational excitation and observing whether reaction occurs are at the origin). This product distribution is consistent with the potential energy surface. Figure 18D.18 shows an attractive Brief illustration 18D.3
and whether the products emerge in a vibrationally excited the harpoon mechanism (Topic 18A) because the transition surface in which the saddle point occurs early in the reaction
The reaction H + Cl2 → HCl + Cl has a repulsive potential
state (Fig. 18D.15). For example, one question that can be an- takes place at long range. In contrast, the collision of K with coordinate. Figure 18D.19 shows a repulsive surface in which
surface. Of the following four reactive processes, H + Cl2(v) →
swered is whether it is better to smash the reactants together CH3I leads to reaction only if the molecules approach each the saddle point occurs late. A surface that is attractive in one
HCl(v′) + Cl, denoted (v,v′), all at the same total energy, (a)
with a lot of translational kinetic energy or to ensure instead other very closely. In this mechanism, K effectively bumps into direction is repulsive in the reverse direction.
(0,0), (b) (2,0), (c) (0,2), (d) (2,2), reaction (b) is most probable
that they approach in highly excited vibrational states. Thus, is a brick wall, and the KI product bounces out in the backward Consider first the attractive surface. If the original molecule is
with reactants vibrationally excited and products vibrationally
trajectory C2*, where the HBHC molecule is initially vibration- direction. The detection of this anisotropy in the angular dis- vibrationally excited, then a collision with an incoming molecule
unexcited.
ally excited, more efficient at leading to reaction than the tra- tribution of products gives an indication of the distance and takes the system along C. This path is bottled up in the region of
jectory C1*, in which the total energy is the same but reactants orientation of approach needed for reaction, as well as showing the reactants, and does not take the system to the saddle point. If,
have a high translational kinetic energy? that the event is complete in less than about 1 ps.
(c) Quantum mechanical scattering theory
(a) The direction of attack and separation A picture of the reaction event can be obtained by using classi-
Figure 18D.16 shows the results of a calculation of the potential cal mechanics to calculate the trajectories of the atoms taking
energy as an H atom approaches an H2 molecule from differ- I H
place in a reaction from a set of initial conditions, such as veloc-
ent angles, the H2 bond being allowed to relax to the optimum Cl ‡ ities, relative orientations, and internal energies of the reacting
Successful C*
length in each case. The potential barrier is least for collinear attack particles. However, classical trajectory calculations do not rec-
attack, as assumed earlier. (But be aware that other lines of at- ognize the fact that the motion of atoms, electrons, and nuclei
tack are feasible and contribute to the overall rate.) In contrast, C is governed by quantum mechanics. The concept of trajectory
Fig. 18D.17 shows the potential energy changes that occur as a Cl then fades and is replaced by the unfolding of a wavefunction
atom approaches an HI molecule. The lowest barrier occurs for Unsuccessful that represents initially the reactants and finally products.
attack
approaches within a cone of half-angle 30° surrounding the H Figure 18D.19 A repulsive potential energy surface. A successful Complete quantum mechanical calculations of trajectories
atom. The relevance of this result to the calculation of the steric Figure 18D.17 The potential energy barrier for the approach of encounter (C*) involves initial vibrational excitation and the and rate constants are very onerous because it is necessary to
factor of collision theory should be noted: not every collision is Cl to HI. In this case, successful encounters occur only when Cl products have high translational kinetic energy. A reaction that is take into account all the allowed electronic, vibrational, and ro-
successful, because they do not all lie within the reactive cone. approaches almost directly towards the H atom. attractive in one direction is repulsive in the reverse direction. tational states populated by each atom and molecule in the sys-
18D The dynamics of molecular collisions 809

tem at a given temperature. It is common to define a ‘channel’


as a group of molecules in well-defined quantum mechanically
allowed states. Then, at a given temperature, there are many
P ( E ) = ∑Pij ( E )
i,j
Cumulative reaction probability (18D.5)
TOPIC 18E Electron transfer in
channels that represent the reactants and many channels that
represent possible products, with some transitions between
channels being allowed but others not allowed. Furthermore,
where Pij(E) is the probability for a transition between a react-
ing channel i and a product channel j and the summation is
over all possible transitions that lead to product. It is then pos-
homogeneous systems
not every transition leads to a chemical reaction. For example, sible to show that the rate constant is given by
the process H2* + OH → H2 + OH*, where the asterisk denotes an

excited state, amounts to energy transfer between H2 and OH,
kr (T ) =
∫ 0
P ( E )e − E/kT dE
whereas the process H*2 + OH → H2O + H represents a chemi- hQ R(T )
Rate constant (18D.6)
cal reaction. What complicates a quantum mechanical calcula- Suppose that in the complex D and A are separated by d, the
tion of rate constants even in this simple four-atom system is where Q R(T) is the partition function density (the partition ➤ Why do you need to know this material? distance between their outer surfaces. Then electron transfer
that many reacting channels present at a given temperature can function divided by the volume) of the reactants at the tem- occurs within the DA complex to yield D+A–:
Electron transfer reactions between protein-bound cofac-
lead to the desired products H2O + H, which themselves may perature T. The significance of eqn 18D.6 is that it provides a
tors or between proteins play an important role in a variety ket
be formed as many distinct channels. The cumulative reaction direct connection between an experimental quantity, the rate
of biological processes. Electron transfer is also important DA ! D+A− (18E.2b)
probability, P ( E ), at a fixed total energy E is then written as constant, and a theoretical quantity, P ( E ). ket′
in homogeneous, non-biological catalysis.
+ −
➤ What is the key idea? The complex D A can also break apart and the ions diffuse
through the solution:
The rate constant of electron transfer in a donor–acceptor
complex depends on the distance between electron
Checklist of concepts donor and acceptor, the standard reaction Gibbs energy, D+A− → D+ + A−
kd
(18E.2c)
and the energy needed to reach a particular arrangement
☐ 1. A molecular beam is a collimated, narrow stream of tiphoton ionization (REMPI), and reaction product Now the techniques described in Topic 17E can be used to
of atoms.
molecules travelling through an evacuated vessel. imaging. write an expression for the rate constant for the overall reac-
➤ What do you need to know already? tion D + A → D+ + A−.
☐ 2. In a molecular beam, the scattering pattern of mol- ☐ 5. A potential energy surface maps the potential energy
ecules depends on quantum mechanical effects and the as a function of the relative positions of all the atoms This Topic makes use of transition-state theory (Topic 18C).
details of the intermolecular potential. taking part in a reaction. It also uses the concept of tunnelling (Topic 7D), the
☐ 3. A van der Waals molecule is a complex of the form AB ☐ 6. In an attractive surface, the saddle point (the highest steady-state approximation (Topic 17E), and the Franck–
Condon principle (Topic 11F). How is that done? 18E.1 Deriving an expression for the
in which A and B are held together by van der Waals point on the valley between reactants and products)
forces or hydrogen bonds. occurs early on the reaction coordinate. rate constant for electron transfer in solution
☐ 4. Techniques for the study of reactive collisions include ☐ 7. In a repulsive surface, the saddle point occurs late on Identify the rate of the overall reaction (eqn 18E.1) with the
infrared chemiluminescence, laser-induced fluores- the reaction coordinate. rate of the step described by eqn 18E.2c because the products
cence, multiphoton ionization (MPI), resonant mul- Concepts of transition-state theory and quantum theory of the reaction are the separated ions:
can be applied to the study of a deceptively simple process, v = kd[D+A−]
electron transfer between molecules in homogeneous sys-
tems. The approach developed here can be used to predict Then follow these steps.
the rates of electron transfer of this kind with reasonable
Checklist of equations accuracy.
Step 1 Apply the steady-state approximation to both inter-
mediates
Property Equation Comment Equation number There are two reaction intermediates, DA and D+A−, so apply
the steady-state approximation (Topic 17E) to both. From
Rate of molecular scattering dI = σINdx σ is the differential scattering cross-section 18D.1
Rate constant kr = 〈σvrel 〉NA 18D.2 18E.1 The rate law d[D+ A − ]
= ket[DA] − ket′ [D+ A − ] − kd [D+ A − ] = 0
State-to-state rate constant knn′ = 〈σ nn′vrel 〉 NA 18D.3 dt
Overall rate constant kr = ∑knn′ (T ) fn (T ) 18D.4 Consider electron transfer from a donor species D to an accep-
it follows that
n ,n ′
tor species A in solution. The overall reaction is
Cumulative reaction probability P ( E ) = ∑Pij ( E ) 18D.5
D + A → D+ + A− v = kr[D][A] ket′ + kd + −
i,j (18E.1) [DA] = [D A ]
∞ ket
Rate constant kr (T ) = ∫ P ( E )e − E/kT d E / hQ R(T ) Q R (T ) is the partition function density 18D.6
0
In the first step of the mechanism, D and A must diffuse
through the solution and on meeting form a complex DA: The steady-state expression for DA is
ka
D + A ! DA (18E.2a) d[DA]
= ka [D][A] − ka′[DA] − ket[DA] + ket′ [D+ A − ] = 0
ka′ dt

18E Electron transfer in homogeneous systems 811 812 18 Reaction dynamics

Next, replace the terms in blue by the expression for [DA] • When ket[DA] >> ka′[DA], which implies that kr ≈ ka, DA
from the preceding equation to give the rate of product formation is controlled by diffu-
Physical interpretation

Potential energy
sion of D and A in solution. V
(k ′ + k )(k ′ + k )
ka [D][A] − et d a et [D+ A − ] + ket′ [D+ A − ] • When ket[DA] << ka′[DA], it follows that kr ≈ (ka/ka′)ket
Gibbs energy

ket
= Kket, where K is the equilibrium constant for the dif-
= ka [D][A] − [D+ A − ] { (ket′ + kd )(ka′ + ket )
ket
− ket′ } fusive encounter. The process is controlled by ket and
therefore the activation energy of electron transfer in
DA
Activation
barrier
(a) q0R
D+A–
(b)
Electron
transfer q‡
(k ′ + k )(k ′ + k ) − ket′ ket
+ − the DA complex. ΔrG ⦵
= ka [D][A] − [D A ] et d a et =0 Electron Nuclear
ket displacement displacement
D+A–
It follows that
Potential energy

q‡
Displacement, q
ka ket 18E.2 The role of electron tunnelling
[D+ A − ] = [D][A]
(ket′ + kd )(ka′ + ket ) − ket′ ket Figure 18E.1 The Gibbs energy surfaces of the complexes DA
and D+A− involved in an electron transfer process are represented
ka ket This analysis can be taken further by introducing the implica-
= [D][A] by parabolas characteristic of harmonic oscillators, with the d
ket′ ka′ + kd ka′ + kd ket (c) q0P
tion from transition-state theory that, at a given temperature, displacement coordinate q corresponding to the changing

ket ∝ e − ∆ G/RT , where ∆‡G is the Gibbs energy of activation. The geometries of the system. Figure 18E.2 (a) At the nuclear configuration denoted by q 0R, the
Step 2 Write an expression for the rate constant remaining task, therefore, is to write expressions for the pro- electron to be transferred in DA is in the HOMO; the LUMO of D+A−
The overall rate of reaction is v = kd[D+A−], which now becomes portionality constant and ∆‡G. The discussion concentrates on is too high in energy for efficient electron transfer. (b) As the nuclei
the following two key aspects of the theory of electron trans- The proportionality constant in the expression for k et is a rearrange to a configuration represented by q ‡, the HOMO of DA
ka ket
v = kd [D][A] fer processes, which was developed independently by R.A. measure of the rate at which the system converts from re- and LUMO of D+A− become similar in energy and electron transfer
ket′ ka′ + kd ka′ + kd ket
Marcus, N.S. Hush, V.G. Levich, and R.R. Dogonadze: actants (DA) to products (D+A−) at q‡ by electron transfer occurs by tunnelling. (c) The system relaxes to the equilibrium
within the thermally excited DA complex. To understand the nuclear configuration of D+A− denoted by q 0P, in which the LUMO of
This expression is of the form v = kr[D][A], with kr given by • Electrons are transferred by tunnelling through a poten- DA is higher in energy than the HOMO of D+A−. Adapted from R.A.
process, consider the effect that the rearrangement of nuclear
tial energy barrier, the height of which is partly deter- coordinates has on the electronic energy levels of DA and Marcus and N. Sutin, Biochim. Biophys. Acta 811, 265 (1985).
ka ket kd ka ket kd
kr = =
ka′ket′ + kd ka′ + kd ket ka′(ket′ + kd ) + kd ket
mined by the ionization energies of the DA and D+A– D+A− for a given distance d between D and A (Fig. 18E.2).
complexes. Electron tunnelling influences the magnitude Initially, the HOMO of DA is lower than the LUMO of D+A−
of the proportionality constant in the expression for ket. (Fig. 18E.2a). As the nuclei rearrange into a configuration ponential dependence on the width of the barrier (Topic 7D),
Step 3 Rearrange the preceding expression
• The complex DA and the solvent molecules surrounding represented by q‡ in Fig. 18E.2b, the HOMO of DA and the suggesting that
You can obtain a more convenient form of the preceding
it undergo structural rearrangements prior to electron LUMO of D+A− become similar in energy and electron trans-
expression by dividing the numerator and denominator on Het(d)2 = Het°2e−βd (18E.4)
transfer. The energy associated with these rearrange- fer becomes feasible. Over reasonably short distances d, the
the right-hand side by kdket to obtain
ments and the standard reaction Gibbs energy determine main mechanism of electron transfer is tunnelling through where d is the edge-to-edge distance between D and A, β is a pa-
ka ∆‡G. the potential energy barrier depicted in Fig. 18E.2b. After rameter that measures the sensitivity of the electronic coupling
kr =
ka′(ket′ + kd )/ ket kd +1 an electron moves from the HOMO of DA to the LUMO of matrix element to distance, and Het° is the value of the electronic
According to the Franck–Condon principle (Topic 11F), D+A−, the system relaxes to the configuration represented coupling matrix element at d = 0. The value of β depends on the
The reciprocal of each side then gives electronic transitions are so fast that they can be regarded as by q 0P in Fig. 18E.2c. As shown in the illustration, now the medium through which the electron must travel from donor to
taking place in a stationary nuclear framework. This princi- energy of D+A− is lower than that of DA, reflecting the ther- acceptor. In a vacuum, 28 nm−1 < β < 35 nm−1, whereas β ≈ 9 nm−1
1 1 ka′ ple also applies to an electron transfer process in which an modynamic tendency for A to remain reduced (as A−) and for when the intervening medium is a molecular link between donor
= + (k ′ + k )
kr ka ka ket kd et d electron migrates from one energy surface, representing the D to remain oxidized (as D+). and acceptor.
dependence of the energy of DA on its geometry, to another The tunnelling event responsible for electron transfer is
and therefore representing the energy of D+A−. The potential energy (and similar to that described in Topic 7D, except that in this case
1 1 k′  k′  (18E.3) the Gibbs energy) surfaces of the two complexes (the reactant the electron tunnels from an electronic level of DA, with wave-
= + a 1 + et 
kr ka ka ket  kd  Electron transfer rate constant
complex, DA, and the product complex, D+A−) can be repre- function ψDA, to an electronic level of D+A−, with wavefunction 18E.3 The rate constant
sented by the parabolas characteristic of harmonic oscilla- ψD+A−. The rate of an electronic transition from a level de-

tors, with the displacement coordinate corresponding to the scribed by the wavefunction ψDA to a level described by ψD+A− is The proportionality constant in ket ∝ e − ∆ G/RT is proportional to
To gain insight into this equation and the factors that de- changing geometries (Fig. 18E.1). This coordinate represents a proportional to the square of the integral Het(d)2, as expressed by eqn 18E.4. A detailed calculation (not
termine the rate of electron transfer reactions in solution, as- collective mode of the donor, acceptor, and solvent. reproduced here) shows that the full expression for ket is
sume that the main decay route for D+A− is dissociation of the According to the Franck–Condon principle, the nuclei do not H et = ∫ψ DAĥψ D A dτ 1/2
1  π3 
+ −

complex into separated ions, and therefore that kd[D+A−] >> have time to move when the system passes from the reactant ket =  H et (d )2 e − ∆ G/RT (18E.5)
h  RT∆ER 
ket′[D+A−], which implies that kd >> ket′. It follows that to the product surface as a result of the transfer of an electron. where ĥ is a hamiltonian that describes the coupling of the
Therefore, electron transfer can occur only after thermal fluc- electronic wavefunctions. The quantity Het is often referred where ΔER is the reorganization energy, the energy change
1 1 k′ 
≈ 1+ a  tuations bring the geometry of DA to q‡ in Fig. 18E.1, the value to as the ‘electronic coupling matrix element’. The probability associated with the molecular rearrangement that must take
kr ka  ket 
of the nuclear coordinate at which the two parabolas intersect. of tunnelling through a potential barrier typically has an ex- place so that DA can take on the equilibrium geometry of
18E Electron transfer in homogeneous systems 813 814 18 Reaction dynamics

D+A−. These molecular rearrangements include the relative re- After rearrangement, the expression for q‡ is which implies that a plot of ln ket against d should be a straight 10
orientation of the D and A molecules in DA and the relative line of slope −β.

○−
reorientation of the solvent molecules surrounding DA. To use ∆ ER + ∆rG
q‡ = 9
eqn 18E.5 it is necessary to find an expression for the Gibbs 2keq0P
energy of activation in terms of a simple model of the reaction.

log ket
Step 2 Use the expression for q‡ to find an expression for ∆‡G
8
Because Gibbs energies are measured from the minimum the A
How is that done? 18E.2 Establishing an expression for the reactant parabola, where q = 0, it follows from the expression 1
Gibbs energy of activation for q‡ that 7
2
The dependence of ket on the standard reaction Gibbs en-
The simplest way to derive an expression for the Gibbs energy  ∆E + ∆ G−○−  −

(∆ER + ∆rG )2

∆‡ G = keq‡2 = ke  R rP  = ergy has been investigated in systems where the edge-to-edge 0 0.5 1 1.5 2 2.5
of activation of electron transfer is to construct a model in  2keq0  4 keq0P2

–ΔrG /eV
! distance and the reorganization energy are constant for a
which the energy surfaces of DA (the ‘reactant complex’, ‡
ΔER series of reactions. Then, because ket ∝e − ∆ G/RT implies that Figure 18E.5 Variation of log ket with −ΔrG for a series of

denoted R) and D+A− (the ‘product complex’, denoted P) are
ln ket = − ∆‡ G/RT + constant, it follows from eqn 18E.6 that compounds with the structures given in (1) and as described in
plotted against the reaction coordinate q and assumed to be which simplifies to
identical parabolic curves with displaced minima (Fig. 18E.3). Brief illustration 18E.1. Based on J.R. Miller et al., J. Am. Chem. Soc.
For simplicity, q can be set equal to 0 at the minimum of the ( ∆ rG− + ∆ER )2
○ 106, 3047 (1984).

○−
( ∆ rG + ∆ER )2 (18E.6) ln ket = − + constant
reactant parabola, and Gibbs energies measured from that ∆‡ G = 4 RT∆ER
4∆ER Gibbs energy of activation
minimum. Then
A constant
!###"### $ linked covalently to a molecular spacer of known and fixed
Gm,R(q) = ke q2 and Gm,P(q) = ke(q − q0P)2 + ΔrG

( ∆ rG− )2 ∆ rG− ∆ER


○ ○
size (Fig. 18E.5).
=− − − + constant
P
where q is the location of the minimum of the product

This equation shows that ∆ G = 0 when ∆rG = −∆ER, and ⦵
4 RT∆ER 2 RT 4 RT
0
parabola, ke is a constant describing the curvature of the the reorganization energy is cancelled by the standard reac-
parabola, and ΔrG is the standard reaction Gibbs energy for

tion Gibbs energy. If this condition holds, the reaction is not and therefore
Brief illustration 18E.1
the electron transfer process DA → D+A−. slowed down by an activation barrier.
2
Write ∆ER = Gm,R(q0P) = keq0P2 as the difference in the Gibbs Equation 18E.6 has some limitations. For instance, it de- RT  ∆ rG−  ○
 ∆ G−  ○
Kinetic measurements were conducted in 2-methyltetrahy-
ln ket = − − 12  r  + constant (18E.8)
energy of R when the coordinate changes from q = 0 to the equi- scribes processes with weak electronic coupling between donor 4∆ER  RT   RT  drofuran and at 296 K for a series of compounds with the
librium value for P, q0P . The Gibbs energy of activation, ∆‡G = and acceptor. Weak coupling is observed when the electroactive structures given in (1) with A the following groups:
Gm,R(q‡) = keq‡2 , is the change in Gibbs energy of R when the species are sufficiently far apart (d > 1 nm) that the tunnelling A plot of ln ket (or log ket = ln ket/ln 10) against ∆rG (or −∆rG )
⦵ ⦵

coordinate changes from q = 0 to q‡. Then follow these steps. is an exponential function of distance. The weak coupling limit should therefore be a downward parabola (a curve of the
Step 1 Identify the location of the activated complex applies to a large number of electron transfer reactions, includ- form y = ax2 + bx + c; Fig. 18E.4). Equation 18E.8 implies that
ing those between proteins during metabolism. Strong coupling the rate constant increases as ∆rG decreases but only up to

The activated complex occurs at the point where the two


is observed when the wavefunctions ψA and ψD overlap very ex- −∆rG = ∆ER. Beyond that, the reaction enters the inverted

parabolas intersect, which is where


tensively and, as well as other complications, the tunnelling rate region, in which the rate constant decreases as the reac-

○−
keq‡2 = ke (q‡ − q0P )2 + ∆rG is no longer a simple exponential function of distance. Examples tion becomes more exergonic (∆rG becomes more negative).

(a) (b) (c)


of strongly coupled systems are mixed-valence, binuclear The inverted region has been observed in a series of special
!
ΔER d-metal complexes with the general structure LmMn+–B–Mp+Lm, compounds in which the electron donor and acceptor are O O O
= keq − 2k q q + keq0P2 + ∆rG
‡2 P ‡
e 0

○− in which the electroactive metal ions are separated by a bridging R1
ligand B. In these systems, d < 1.0 nm.
constant + ΔER/4RT R2
O O O

R P 18E.4 Experimental tests of the theory constant


(d) (e) (f) R1 = H, R2 = H
(g) R1 = H, R2 = Cl
ΔER keq 2
Gibbs energy

(h) R1 = Cl, R2 = Cl
The most meaningful experimental tests of the dependence of

ln ket
ΔrG‡ ket on d are those in which the same donor and acceptor are
The distance between donor (the reduced biphenyl group)
0 positioned at a variety of distances, perhaps by covalent at-
and the acceptor is constant for all compounds in the series
ΔrG ⦵ ke(q – q0P)2 + ΔrG ⦵ tachment to molecular linkers (see 1 for an example, in which
because the molecular linker remains the same. Each acceptor
the biphenyl group is the donor and A are various acceptors). ΔER
‡ has a characteristic standard potential, so it follows that the
Under these conditions, the term e − ∆ G/RT becomes a constant standard Gibbs energy for the electron transfer process is dif-
0 q‡ q0P and, after taking the natural logarithm of eqn 18E.5 and using 0 –ΔrG

ferent for each compound in the series. The line in Fig. 18E.5 is
Displacement, q
eqn 18E.4, the result is a fit to a version of eqn 18E.8 and the maximum of the parabola

Figure 18E.3 The model system used in the calculation of the Figure 18E.4 The parabolic dependence of ln ket on −ΔrG occurs at −ΔrG = ΔER = 1.4 eV = 1.4 × 102 kJ mol−1.

Gibbs energy of activation for an electron transfer process. ln ket = −βd + constant (18E.7) predicted by eqn 18E.8.

18E Electron transfer in homogeneous systems 815 816 18 Reaction dynamics

Checklist of concepts FOCUS 18 Reaction dynamics


☐ 1. Electron transfer can occur through tunnelling only ☐ 2. The reorganization energy is the energy change asso-
after thermal fluctuations bring the nuclear coordinate ciated with molecular rearrangements that must take
to the point at which the donor and acceptor have the place so that DA can acquire the equilibrium geometry TOPIC 18A Collision theory
same configuration. of D+A−.
Discussion questions
D18A.1 Discuss how the collision theory of gas-phase reactions builds on the D18A.3 Describe the essential features of the harpoon mechanism.
kinetic molecular theory.
D18A.4 Explain how the complexity of the reacting molecules affects the value
Checklist of equations D18A.2 How might collision theory change for real gases? of the rate constant.

Property Equation Comment Equation number Exercises


Electron transfer rate constant 1/ kr = 1/ ka + (ka′ / ka ket )(1 + ket′ / kd ) Steady-state assumption 18E.3 E18A.1(a) Calculate the collision frequency, z, and the collision density, ZAA, in D2+ Br2 → DBr + DBr at 450 K. Take the collision cross-section as 0.30 nm2,
2 2 −βd ammonia, d = 380 pm, at 30 °C and 120 kPa. What is the percentage increase the reduced mass as 3.930mu, and the activation energy as 200 kJ mol−1.
Tunnelling probability Het(d) = Het° e Assumed 18E.4
when the temperature is raised by 10 K at constant volume? (Assume a steric factor of 1.)
Rate constant 3 1/2 2 − ∆‡ G / RT E18A.1(b) Calculate the collision frequency, z, and the collision density, ZAA, in
ket = (1/ h)(π / RT∆ER ) H et (d ) e Transition-state theory 18E.5 E18A.5(a) For the gaseous reaction A + B → P, the reactive cross-section
carbon monoxide, d = 360 pm, at 30 °C and 120 kPa. What is the percentage
obtained from the experimental value of the frequency factor is 9.2 ×
Gibbs energy of activation



∆‡ G = ( ∆ rG + ∆ER )2 /4∆ER Assumes parabolic potential energy 18E.6 increase when the temperature is raised by 10 K at constant volume?
10−22 m2. The collision cross-sections of A and B estimated from the transport
E18A.2(a) Collision theory depends on knowing the fraction of molecular properties are 0.95 nm2 and 0.65 nm2, respectively. Calculate the steric factor,
collisions having at least the kinetic energy Ea along the line of flight. What is P, for the reaction.
this fraction when (i) Ea = 20 kJ mol−1, (ii) Ea = 100 kJ mol−1 at (1) 350 K and E18A.5(b) For the gaseous reaction A + B → P, the reactive cross-section
(2) 900 K? obtained from the experimental value of the frequency factor is 8.7 ×
E18A.2(b) Collision theory depends on knowing the fraction of molecular 10−22 m2. The collision cross-sections of A and B estimated from the transport
collisions having at least the kinetic energy Ea along the line of flight. What is properties are 0.88 nm2 and 0.40 nm2, respectively. Calculate the steric factor,
this fraction when (i) Ea = 15 kJ mol−1, (ii) Ea = 150 kJ mol−1 at (1) 300 K and P, for the reaction.
(2) 800 K?
E18A.6(a) Consider the unimolecular decomposition of a nonlinear molecule
E18A.3(a) Calculate the percentage increase in the fractions in Exercise containing five atoms according to RRK theory. If kb(E)/kb = 3.0 × 10−5, what is
E18A.2(a) when the temperature is raised by 10 K in each case. the value of E*/E?
E18A.3(b) Calculate the percentage increase in the fractions in Exercise E18A.6(b) Consider the unimolecular decomposition of a linear molecule
E18A.2(b) when the temperature is raised by 10 K in each case. containing four atoms according to RRK theory. If kb(E)/kb = 0.025, what is the
value of E*/E?
E18A.4(a) Use the collision theory of gas-phase reactions to calculate the
−1
theoretical value of the second-order rate constant for the elementary E18A.7(a) Suppose that an energy of 250 kJ mol is available in a collision but
reaction H2 + I2 → HI + HI at 650 K. The collision cross-section is 0.36 nm2, 200 kJ mol−1 is needed to break a particular bond in a molecule with s = 10.
the reduced mass is 3.32 × 10−27 kg, and the activation energy is 171 kJ mol−1. Use the RRK model to calculate the ratio kb(E)/kb.
−1
(Assume a steric factor of 1.) E18A.7(b) Suppose that an energy of 500 kJ mol is available in a collision but
E18A.4(b) Use the collision theory of gas-phase reactions to calculate the 300 kJ mol−1 is needed to break a particular bond in a molecule with s = 12.
theoretical value of the second-order rate constant for the elementary reaction Use the RRK model to calculate the ratio kb(E)/kb.

Problems
P18A.1 In the dimerization of methyl radicals at 25 °C, the experimentally sample of ethane of volume 1.0 dm3 at 298 K and 100 kPa is dissociated into
determined frequency factor is 2.4 × 1010 dm3 mol−1 s−1. What are (a) the reac- methyl radicals. What is the minimum time for 90 per cent recombination?
tive cross-section, and (b) the steric factor for the reaction given that the C–H
P18A.4 The reactive cross-sections for reactions between alkali metal atoms
bond length is 154 pm?
and halogen molecules are given in the table below (R.D. Levine and R.B.
P18A.2 Nitrogen dioxide reacts bimolecularly in the gas phase according to Bernstein, Molecular reaction dynamics, Clarendon Press, Oxford, 72 (1974)).
NO2 + NO2 → NO + NO + O2. The temperature dependence of the second- Assess the data in terms of the harpoon mechanism.
order rate constant for the rate law d[P]/dt = kr[NO2]2 is given below. What
are the steric factor and the reactive cross-section for the reaction? σ*/nm2 Cl2 Br2 I2
Na 1.24 1.16 0.97
T/K 600 700 800 1000
K 1.54 1.51 1.27
kr/(cm3 mol−1 s−1) 4.6 × 102 9.7 × 103 1.3 × 105 3.1 × 106
Rb 1.90 1.97 1.67
Take σ = 0.60 nm2.
Cs 1.96 2.04 1.95
P18A.3 The diameter of the methyl radical is about 308 pm. What is the
2
maximum rate constant in the expression d[C2H6]/dt = kr[CH3] for second- Electron affinities are approximately 1.3 eV (Cl2), 1.2 eV (Br2), and 1.7 eV (I2), and
order recombination of radicals at 298 K? It is reported that 10 per cent of a ionization energies are 5.1 eV (Na), 4.3 eV (K), 4.2 eV (Rb), and 3.9 eV (Cs).
Exercises and problems 817 818 18 Reaction dynamics

P18A.5‡ R. Atkinson (J. Phys. Chem. Ref. Data 26, 215 (1997)) has reviewed R = C2H5 and 8.4 × 109 dm3 mol−1 s−1 for R = cyclohexyl. Assuming no energy D18C.3 Explain the physical origin of the kinetic salt effect. How might the size D18C.4 How do kinetic isotope effects provide insight into the mechanism of
a large set of rate constants relevant to the atmospheric chemistry of volatile barrier, compute the steric factor, P, for each reaction. Hint: Obtain collision of the effect be altered by a change in the relative permittivity of the medium? a reaction?
organic compounds. The recommended rate constant for the bimolecular diameters from collision cross-sections of similar molecules in the Resource
reaction of O2 with an alkyl radical R at 298 K is 4.7 × 109 dm3 mol−1 s−1 for section.
Exercises

E18C.1(a) The reaction of propylxanthate anion, A , in ethanoic acid buffer so- E18C.5(a) The frequency factor for the second-order gas-phase
lutions has the mechanism A− + H+ → P. Near 30 °C the rate constant is given decomposition of ozone at low pressures is 4.6 × 1012 dm3 mol−1 s−1 and its
by the empirical expression kr = Ae−(8681 K)/T with A = 2.05 × 1013 dm3 mol−1 s−1. activation energy is 10.0 kJ mol−1. What are (i) the entropy of activation,
TOPIC 18B Diffusion-controlled reactions Evaluate the enthalpy and entropy of activation at 30 °C.
− +
E18C.1(b) The reaction A + H → P has a rate constant given by the empirical
(ii) the enthalpy of activation, (iii) the Gibbs energy of activation at 298 K?
Assume κ = 1.
expression kr = Ae−(5925 K)/T with A = 6.92 × 1012 dm3 mol−1 s−1. Evaluate the E18C.5(b) The frequency factor for a second-order gas-phase decomposition
Discussion questions enthalpy and entropy of activation at 25 °C. of a species at low pressures is 2.3 × 1013 dm3 mol−1 s−1 and its activation
D18B.1 Distinguish between a diffusion-controlled reaction and an activation- D18B.2 Describe the role of the encounter pair in the cage effect.
energy is 30.0 kJ mol−1. What are (i) the entropy of activation, (ii) the
E18C.2(a) When the reaction in Exercise E18C.1(a) occurs in a dioxane/
controlled reaction. Do both have activation energies? enthalpy of activation, (iii) the Gibbs energy of activation at 298 K?
water mixture which is 30 per cent dioxane by mass, the rate constant fits
Assume κ = 1.
kr = Ae−(9134 K)/T with A = 7.78 × 1014 dm3 mol−1 s−1 near 30 °C. Calculate Δ‡G
− +
for the reaction at 30 °C; assume κ = 1. E18C.6(a) The rate constant of the reaction H2O2(aq) + I (aq) + H (aq) →
Exercises E18C.2(b) A rate constant is found to fit the expression kr = Ae
−(4972 K)/T
with A = H2O(l) + HIO(aq) is sensitive to the ionic strength of the aqueous solution
E18B.1(a) A typical diffusion constant for small molecules in aqueous solution E18B.3(b) Calculate the magnitude of the diffusion-controlled rate constant at 4.98 × 1013 dm3 mol−1 s−1 near 25 °C. Calculate Δ‡G for the reaction at 25 °C; in which the reaction occurs. At 25 °C, kr = 12.2 dm6 mol−2 min−1 at an ionic
at 25 °C is 6 × 10−9 m2 s−1. If the critical reaction distance is 0.5 nm, what value is 320 K for the recombination of two atoms in benzene, for which η = 0.601 cP. assume κ = 1. strength of 0.0525. Use the Debye–Hückel limiting law to estimate the rate
expected for the second-order rate constant for a diffusion-controlled reaction? Assuming the concentration of the reacting species is 2.0 mmol dm−3 initially, constant at zero ionic strength.
E18C.3(a) The gas-phase reaction between F2 and IF5 is first order in each 6 −2 −1
E18B.1(b) Suppose that the typical diffusion coefficient for a reactant in how long does it take for the concentration of the atoms to fall to half that E18C.6(b) At 25 °C, kr = 1.55 dm mol min at an ionic strength of 0.0241 for
of the reactants. The energy of activation for the reaction is 58.6 kJ mol−1.
aqueous solution at 25 °C is 5.2 × 10−9 m2 s−1. If the critical reaction distance value? Assume the reaction is elementary. a reaction in which the rate-determining step involves the encounter of two
At 65 °C the rate constant is 7.84 × 10−3 kPa−1 s−1. Calculate the entropy of
is 0.4 nm, what value is expected for the second-order rate constant for the singly charged cations. Use the Debye–Hückel limiting law to estimate the rate
E18B.4(a) Two neutral species, A and B, with diameters 655 pm and 1820 pm, activation at 65 °C.
diffusion-controlled reaction? constant at zero ionic strength.
respectively, undergo the diffusion-controlled reaction A + B → P in a solvent E18C.3(b) A certain gas-phase reaction is first order in each of the reactants.
E18B.2(a) Calculate the magnitude of the diffusion-controlled rate constant of viscosity 2.93 × 10−3 kg m−1 s−1 at 40 °C. Use eqn 18B.3 to calculate the initial The energy of activation for the reaction is 39.7 kJ mol−1. At 65 °C the rate E18C.7(a) Estimate the magnitude of the primary kinetic isotope effect on the
at 298 K for a species in (i) water, (ii) pentane. The viscosities are 1.00 × rate d[P]/dt, given that the initial concentrations of A and B are 0.170 mol constant is 0.35 m3 mol−1 s−1. Calculate the entropy of activation at 65 °C. relative rates of displacement of 1H and 3H in a C–H bond. Will raising the
10−3 kg m−1 s−1 and 2.2 × 10−4 kg m−1 s−1, respectively. dm−3 and 0.350 mol dm−3, respectively. Then repeat the calculation by using temperature enhance the difference? Take kf(C–H) = 450 N m−1.
E18C.4(a) Calculate the entropy of activation for a collision between two
E18B.2(b) Calculate the magnitude of the diffusion-controlled rate constant eqn 18B.4. Comment on the validity of the approximation that leads to eqn E18C.7(b) Estimate the magnitude of the primary isotope effect on the
structureless particles at 300 K, taking M = 65 g mol−1 and σ = 0.35 nm2. Hint:
at 298 K for a species in (i) decylbenzene, (ii) concentrated sulfuric acid. The 18B.4. relative rates of displacement of 16O and 18O in a C–O bond. Will raising the
Refer to Example 18C.1.
viscosities are 3.36 cP and 27 cP, respectively. E18A.4(b) Two neutral species, A and B, with diameters 421 pm and 945 pm, temperature enhance the difference? Take kf(C–O) = 1750 N m−1.
E18C.4(b) Calculate the entropy of activation for a collision between two
respectively, undergo the diffusion-controlled reaction A + B → P in a solvent structureless particles at 450 K, taking M = 92 g mol−1 and σ = 0.45 nm2.
E18B.3(a) Calculate the magnitude of the diffusion-controlled rate constant
of viscosity 1.35 cP at 20 °C. Use eqn 18B.3 to calculate the initial rate d[P]/
at 320 K for the recombination of two atoms in water, for which η = 0.89 cP.
dt, given that the initial concentrations of A and B are 0.155 mol dm−3 and
Assuming the concentration of the reacting species is 1.5 mmol dm−3 initially,
0.195 mol dm−3, respectively. Then repeat the calculation by using eqn 18B.4. Problems
how long does it take for the concentration of the atoms to fall to half that
Comment on the validity of the approximation that leads to eqn 18B.4.
value? Assume the reaction is elementary.
P18C.1‡ For the gas-phase reaction A + A → A2, the experimental rate con- the activated complex removes much of the ambiguity of theoretical
stant, kr, has been fitted to the Arrhenius equation with a frequency factor predictions. Consider the attack of H on D2, which is one step in the H2 +
Problems A = 4.07 × 105 dm3 mol−1 s−1 at 300 K and an activation energy of 65.4 kJ mol−1. D2 reaction. (a) Suppose that the H approaches D2 from the side and forms
Calculate Δ‡S, Δ‡H, Δ‡U, and Δ‡G for the reaction. a complex in the form of an isosceles triangle. Take the H–D distance as
P18B.1 Confirm that eqn 18B.8 is a solution of eqn 18B.7, where [J] is a solu- Base you answer on eqn 18B.7. 30 per cent greater than in H2 (74 pm) and the D–D distance as 20 per
P18C.2 The rates of thermal decomposition of a variety of cis- and trans-
tion of the same equation but with kr = 0 and the same initial conditions. cent greater than in H2. Let the critical coordinate be the antisymmetric
P18B.4‡ The compound α-tocopherol, a form of vitamin E, is a powerful azoalkanes have been measured over a range of temperatures in order to
P18B.2 Use mathematical software or a spreadsheet to explore the effect of antioxidant that may help to maintain the integrity of biological membranes. stretching vibration in which one H–D bond stretches as the other shortens.
settle a controversy concerning the mechanism of the reaction. In ethanol an
varying the value of the rate constant kr on the spatial variation of [J]* (see R.H. Bisby and A.W. Parker (J. Amer. Chem. Soc. 117, 5664 (1995)) studied Let all the vibrations be at about 1000 cm−1. Estimate kr for this reaction
unstable cis-azoalkane decomposed at a rate that was followed by observing
eqn 18B.8 with [J] given in eqn 18B.9) for a constant value of the diffusion the reaction of photochemically excited duroquinone with the antioxidant in at 400 K using the experimental activation energy of about 35 kJ mol−1.
the N2 evolution, and this led to the rate constants listed below (P.S. Engel
coefficient D. ethanol. Once the duroquinone was photochemically excited, a bimolecular (b) Now change the model of the activated complex in part (a) and make
and D.J. Bishop, J. Amer. Chem. Soc. 97, 6754 (1975)). Calculate the enthalpy,
reaction took place at diffusion-limited rate. (a) Estimate the rate constant it linear. Use the same estimated molecular bond lengths and vibrational
P18B.3 Confirm that if the boundary condition is [J] = [J]0 at t > 0 at all points
entropy, energy, and Gibbs energy of activation at −20 °C.
for a diffusion-limited reaction in ethanol. (b) The reported rate constant was frequencies to calculate kr for this choice of model. (c) Clearly, there is
on the yz-plane, and the initial condition is [J] = 0 at t = 0 everywhere else, much scope for modifying the parameters of the models of the activated
2.77 × 109 dm3 mol−1 s−1. Estimate the critical reaction distance if the sum of θ/°C −24.82 −20.73 −17.02 −13.00 −8.95
then the solutions [J]* in the presence of a first-order reaction that removed J complex. Use mathematical software to vary the structure of the complex
diffusion coefficients is 1 × 10−9 m2 s−1.
are related to those in the absence of reaction, [J], by 104 × kr/s−1 1.22 2.31 4.39 8.50 14.3 and the parameters in a plausible way, and look for a model (or more than
t one model) that gives a value of kr close to the experimental value,
[J]* = kr ∫ [J]e − kr t
dt +[J]e − kr t
0 P18C.3 Derive the expression for kr given in Example 18C.1 starting from the 4 × 105 dm3 mol−1 s−1.
point at which the thermal wavelengths are substituted. P18C.6‡ M. Cyfert et al. (Int. J. Chem. Kinet. 28, 103 (1996)) examined the
P18C.4‡ Show that bimolecular reactions between nonlinear molecules are much oxidation of tris(1,10-phenanthroline)iron(II) by periodate in aqueous
slower than between atoms even when the activation energies of both reactions solution. To assess the kinetic salt effect, they measured rate constants at a
TOPIC 18C Transition-state theory are equal. Use transition-state theory and make the following assumptions. variety of concentrations of Na2SO4 far in excess of reactant concentrations
and reported the following data at 298 K:
(1) All vibrational partition functions are close to 1; (2) all rotational partition
functions are approximately 1 × 101.5; (3) the translational partition function for
Discussion questions each species is 1 × 1026. Hint: Equation 18C.9 is a good starting point. [Na2SO4]/(mol kg−1) 0.2 0.15 0.1 0.05 0.025 0.0125 0.005
D18C.1 Which mode would be discarded for a reaction A + BC in which the D18C.2 Describe in outline the formulation of the Eyring equation. kr/(dm3/2 mol−1/2 s−1) 0.462 0.430 0.390 0.321 0.283 0.252 0.224
P18C.5 This exercise gives some familiarity with the difficulties involved
activated complex is modelled as a linear triatomic cluster? in predicting the structure of activated complexes. It also demonstrates What can be inferred about the charge of the activated complex of the
the importance of femtosecond spectroscopy for understanding chemical rate-determining step? The ionic strength of a solution of Na2SO4 is

These problems were provided by Charles Trapp and Carmen Giunta. dynamics because direct experimental observation of a cluster resembling 3[Na2SO4]/(mol kg−1).

Exercises and problems 819 820 18 Reaction dynamics

− −
P18C.7 The study of conditions that optimize the association of proteins in P18C.9 The rate constant of the reaction I (aq) + H2O2(aq) → H2O(l) + IO (aq)
solution guides the design of protocols for formation of large crystals that
are amenable to analysis by X-ray diffraction techniques. It is important to
varies weakly with ionic strength, even though use of the Debye–Hückel
limiting law predicts no effect. Assume that the rate-determining step involves
TOPIC 18E Electron transfer in homogeneous systems
characterize protein dimerization because the process is considered to be the a reaction between I− and H2O2 and use the following data from 25 °C to find
rate-determining step in the growth of crystals of many proteins. Consider the the dependence of log kr on the ionic strength: Discussion questions
variation with ionic strength of the rate constant at 298 K of dimerization in
D18E.1 Discuss how the following factors affect the rate of electron transfer in D18E.2 What role does tunnelling play in electron transfer?
aqueous solution of a cationic protein P: I 0.0207 0.0525 0.0925 0.1575
homogeneous systems: the distance between electron donor and acceptor, the
3 −1
kr/(dm mol min ) −1
0.663 0.670 0.679 0.694 D18E.3 Explain why the rate constant for electron transfer decreases as the
standard Gibbs energy of the process, and the reorganization energy of the
reaction becomes more exergonic in the inverted region.
I 0.0100 0.0150 0.0200 0.0250 0.0300 0.0350 redox active species and the surrounding medium.
Evaluate the limiting value of kr at zero ionic strength. What does the result
kr/k°r 8.10 13.30 20.50 27.80 38.10 52.00
suggest for the dependence of log γ on ionic strength for a neutral molecule
in an electrolyte solution? Exercises
What can be deduced about the charge of P?
P18C.10 Use the Debye–Hückel limiting law to show that changes in ionic 2
E18E.1(a) By how much does Het(d) change when d is increased from 1.0 nm −0.975 eV when a substituent is added to the electron acceptor and the rate
P18C.8 In an experimental study of a bimolecular reaction in aqueous strength can affect the rate of reaction catalysed by H+ from the deprotonation to 2.0 nm, with β ≈ 9 nm–1? constant for electron transfer changes to ket = 3.33 × 106 s−1. Assume that the
solution, the second-order rate constant was measured at 25 °C and at a of a weak acid. Consider the mechanism: H+ + B → P, where H+ is supplied by 2
E18E.1(b) By how much does Het(d) change when d is increased from 1.0 nm distance between donor and acceptor is the same in both experiments and
variety of ionic strengths; the results are tabulated below. It is known that a the weak acid, HA, which has a fixed concentration. First show that log [H+] to 2.0 nm, with β ≈ 30 nm–1? estimate the values of Het(d) and ΔER.
singly charged ion is involved in the rate-determining step. What is the charge depends on the activity coefficients of ions and thus depends on the ionic
−1 5 −1
on the other ion involved? strength. Then find the relationship between log(rate) and log [H+] to show E18E.2(a) For an electron donor/acceptor pair at 298 K, Het(d) = 0.04 cm , E18E.3(a) For an electron donor/acceptor pair, ket = 2.02 × 10 s when d =
⦵ ⦵
that the rate also depends on the ionic strength. ΔrG = −0.185 eV, and ket = 37.5 s−1. Use mathematical software to estimate the 1.11 nm and ket = 4.51 × 104 s−1 when r = 1.23 nm. Assume that ΔrG and ΔER
I 0.0025 0.0037 0.0045 0.0065 0.0085 value of the reorganization energy. are the same in both experiments and estimate the value of β.
P18C.11 The bromination of a deuterated hydrocarbon at 298 K proceeds 5 −1
E18E.2(b) For an electron donor/acceptor pair at 298 K, ket = 2.02 × 10 s and E18E.3(b) Refer to Exercise E18E.3(a). Estimate the value of ket when d =
kr/(dm3 mol−1 s−1) 1.05 1.12 1.16 1.18 1.26 6.4 times more slowly than the bromination of the undeuterated material. What ⦵ ⦵
ΔrG = −0.665 eV. The standard reaction Gibbs energy changes to ΔrG = 1.59 nm.
value of the force constant for the cleaved bond can account for this difference?

Problems
+ −
P18E.1 Consider the reaction D + A → D + A . The rate constant kr may be P18E.4 A useful strategy for the study of electron transfer in proteins consists

TOPIC 18D The dynamics of molecular collisions determined experimentally or may be predicted by the Marcus cross-relation
kr = (kDDkAAK)1/2 f, where kDD and kAA are the experimental rate constants for the
of attaching an electroactive species to the protein’s surface and then
measuring ket between the attached species and an electroactive protein
electron self-exchange processes *D + D+ → *D+ + D and *A + A+ → *A+ + A, cofactor. J.W. Winkler and H.B. Gray (Chem. Rev. 92, 369 (1992)) summarize
Discussion questions respectively, and f is a function of K = [D+][A−]/[D][A], kDD, kAA, and κν ‡. data for cytochrome c modified by replacement of the haem (heme) iron by
Derive the approximate form of the Marcus cross-relation by following these a zinc ion, resulting in a zinc–porphyrin (ZnP) group in the interior of the
D18D.1 Describe how the following techniques are used in the study of chemi- D18D.3 Consider a reaction with an attractive potential energy surface.
steps. (a) Use eqn 18E.6 to write expressions for Δ‡G, Δ‡GDD, and Δ‡GAA, keep- protein, and by attachment of a ruthenium ion complex to a surface histidine
cal dynamics: infrared chemiluminescence, laser-induced fluorescence, mul- Discuss how the initial distribution of reactant energy affects how efficiently ⦵
ing in mind that ΔrG = 0 for the electron self-exchange reactions. (b) Assume amino acid. The edge-to-edge distance between the electroactive species was
tiphoton ionization, resonant multiphoton ionization, and reaction product the reaction proceeds. Repeat the discussion for a repulsive potential energy that the reorganization energy ΔER,DA for the reaction D + A → D+ + A− is the thus fixed at 1.23 nm. A variety of ruthenium ion complexes with different
imaging. surface. average of the reorganization energies ΔER,DD and ΔER,AA of the electron self- standard potentials was used. For each ruthenium-modified protein, either

D18D.2 Discuss the relationship between the saddle-point energy and the D18D.4 Describe how molecular beams are used to investigate intermolecular exchange reactions. Then show that in the limit of small magnitude of ΔrG , Ru2+ → ZnP+ or ZnP* → Ru3+, in which the electron donor is an electronically
⦵ ⦵ ⦵
activation energy of a reaction. forces. or |ΔrG | << ΔER,DA, Δ‡G = 12 (Δ‡GDD + Δ‡GAA + ΔrG ), where ΔrG is the stand- excited state of the zinc–porphyrin group formed by laser excitation, was
ard Gibbs energy for the reaction D + A → D+ + A−. (c) Use an equation of monitored. This arrangement leads to different standard reaction Gibbs
the form of eqn 18E.5 to write expressions for kDD and kAA. (d) Use eqn 18E.5 energies because the redox couples ZnP+/ZnP and ZnP+/ZnP* have different
Exercises and the result you have derived to write an expression for kr. (e) Complete the

standard potentials, with the electronically excited porphyrin being a more
derivation by using the results from part (c), the relation K = e−ΔrG /RT), and powerful reductant. Use the following data to estimate the reorganization
E18D.1(a) The interaction between an atom and a diatomic molecule is de- E18D.2(a) If the cumulative reaction probability was independent of energy, assuming that all κν ‡ terms are identical. energy for this system:
scribed by an ‘attractive’ potential energy surface. What distribution of vibra- what would be the temperature dependence of the rate constant predicted by + −
P18E.2 Consider the reaction D + A → D + A . The rate constant kr may be ⦵
tional and translational energies among the reactants is most likely to lead to a the numerator of eqn 18D.6? −ΔrG /eV 0.665 0.705 0.745 0.975 1.015 1.055
successful reaction? Describe the distribution of vibrational and translational E18D.2(b) If the cumulative reaction probability equalled 1 for energies less
determined experimentally or may be predicted by the Marcus cross-relation
(see Problem P18E.1). It is common to make the assumption that f ≈ 1. Use ket/(106 s−1) 0.657 1.52 1.12 8.99 5.76 10.1
energies among the products for these most successful reactions. than a barrier height V and vanished for higher energies, what would be the
E18D.1(b) The interaction between an atom and a diatomic molecule is temperature dependence of the rate constant predicted by the numerator of the approximate form of the Marcus relation to estimate the rate constant for
the reaction [Ru(bpy)3]3+ + [Fe(OH2)6]2+ → [Ru(bpy)3]2+ + [Fe(OH2)6]3+, where P18E.5 The photosynthetic reaction centre of the purple photosynthetic
described by a ‘repulsive’ potential energy surface. What distribution of eqn 18D.6?
bpy stands for 4,4′-bipyridine. Use the following data: bacterium Rhodopseudomonas viridis contains a number of bound cofactors
vibrational and translational energies among the reactants is most likely to
that participate in electron transfer reactions. The following table shows data
lead to a successful reaction? Describe the distribution of vibrational and ⦵
[Ru(bpy)3]3+ + e− → [Ru(bpy)3]2+ E = 1.26 V compiled by Moser et al. (Nature 355, 796 (1992)) on the rate constants for
translational energies among the products for these most successful reactions.
⦵ electron transfer between different cofactors and their edge-to-edge distances:
[Fe(OH2)6]3+ + e− → [Fe(OH2)6]2+ E = 0.77 V
3+
*[Ru(bpy)3] + [Ru(bpy)3] → 2+
kRu = 4.0 × 108 dm3 mol−1 s−1 Reaction BChl− → BPh BPh− → BChl2+ BPh− → QA cyt c559 → BChl2+
Problems *[Ru(bpy)3]2+ + [Ru(bpy)3]3+
d/nm 0.48 0.95 0.96 1.23
P18D.1 Show that the intensities, I, of a molecular beam before and after P18D.3 Suppose a harmonic oscillator collides with another oscillator of *[Fe(OH2)6]3+ + [Fe(OH2)6]2+ → kFe = 4.2 dm3 mol−1 s−1 −1 12 9 9
ket/s 1.58 × 10 3.98 × 10 1.00 × 10 1.58 × 108
passing through a chamber of length L containing inert scattering atoms are the same effective mass and force constant. Evaluate kr by assuming that the *[Fe(OH2)6]2+ + [Fe(OH2)6]3+
related by I = I0e−NσL, where σ is the collision cross-section and N the number state-to-state rate constant for the excitation of the latter’s vibration is Reaction QA− → QB QA− → BChl2+
density of scattering atoms. kvv′ = k°δ –λv P18E.3 Some data in the inverted region on a series of donor–linker–acceptor
r vv′e , implying that the transfer becomes less efficient as the d/nm 1.35 2.24
vibrational quantum number increases. Hint: Refer to Brief illustration complexes are as follows:
P18D.2 In a molecular beam experiment to measure collision cross-sections it ket/s−1 3.98 × 107 63.1
18D.1. ⦵
was found that the intensity of a CsCl beam was reduced to 60 per cent of its –ΔrG /eV 0.20 0.60 1.0 1.3 1.6 2.0 2.4
intensity on passage through CH2F2 at 10 μTorr, but that when the target was P18D.4 Use the approach in Brief illustration 18D.2 to analyse the reaction log ket 8.2 9.7 10.2 10.1 9.4 7.7 5.1
Ar at the same pressure the intensity was reduced only by 10 per cent. What H + OD → OH + D.
are the relative cross-sections of the two types of collision? Why is one much Evaluate the reorganization energy.
larger than the other?
Exercises and problems 821

(BChl, bacteriochlorophyll; BChl2, bacteriochlorophyll dimer, functionally P18E.6 The rate constant for electron transfer between a cytochrome c and
distinct from BChl; BPh, bacteriophaeophytin; QA and QB, quinone molecules the bacteriochlorophyll dimer of the reaction centre of the purple bacterium
bound to two distinct sites; cyt c559, a cytochrome bound to the reaction centre Rhodobacter sphaeroides (Problem P18E.5) decreases with decreasing
complex). Are these data in agreement with the behaviour predicted by eqn temperature in the range 300 K to 130 K. Below 130 K, the rate constant
18E.7? If so, evaluate the value of β. becomes independent of temperature. Account for these results.

FOCUS 18 Reaction dynamics


Integrated activities
I18.1 According to the RRK model (see A deeper look 12 on the website of this I18.2 Estimate the orders of magnitude of the partition functions involved in
book) a rate expression. State the order of magnitude of q mT/NA, q R, q V, q E for typical
molecules. Check that in the collision of two structureless molecules the
n! (n − n * + s −1) order of magnitude of the pre-exponential factor is of the same order as that
P= predicted by collision theory. Go on to estimate the steric factor for a reaction
(n − n*)!(n + s −1)
in which A + B → P, and A and B are nonlinear triatomic molecules.
Use Stirling’s approximation of the form ln x! ≈ x ln x − x to deduce that P ≈ I18.3 Discuss the factors that govern the rates of electron transfer according
{(n − n*)/n}s−1 when s − 1 << n − n*. Hint: Replace terms of the form n − n* + to Marcus theory and that govern the rates of resonance energy transfer
s − 1 by n − n* inside logarithms but retain n − n* + s − 1 when it is a factor of according to Förster theory (Topic 17G). Can you find similarities between
a logarithm. the two theories?

You might also like